Está en la página 1de 143

NOTAS DE TRABAJO, 30

ARITMTICA Y COMBINATORIA
(I. S UCESIONES RECURRENTES )

Pascual Jara Martnez

Departamento de lgebra. Universidad de Granada


Granada, 20042013

Primera redaccin: Julio 2004.


Segunda redaccin: Marzo 2006.
Tercera redaccin: Febrero 2007.
Cuarta redaccin: Mayo 2009.
Quinta redaccin: Enero 2013.

Introduction
This text is a compilation of Discrete Mathematics.

ndice general
Introduction

Sucesiones recurrentes

Introduccin

Sucesiones
1
Progresiones aritmticas . . . . . . . . . .
2
Progresiones aritmticas de order superior
3
Progresiones geomtricas . . . . . . . . . .
4
Ejercicios con solucin . . . . . . . . . . .

.
.
.
.

.
.
.
.

.
.
.
.

.
.
.
.

.
.
.
.

.
.
.
.

.
.
.
.

.
.
.
.

.
.
.
.

.
.
.
.

.
.
.
.

.
.
.
.

.
.
.
.

.
.
.
.

.
.
.
.

.
.
.
.

.
.
.
.

.
.
.
.

.
.
.
.

5
. 6
. 7
. 13
. 14

II

Sucesiones recurrentes
5
Sucesiones recurrentes homogneas .
6
Sucesiones recurrentes no homogneas
7
Funciones generatrices . . . . . . . . .
8
Ejercicios con solucin . . . . . . . . .

.
.
.
.

.
.
.
.

.
.
.
.

.
.
.
.

.
.
.
.

.
.
.
.

.
.
.
.

.
.
.
.

.
.
.
.

.
.
.
.

.
.
.
.

.
.
.
.

.
.
.
.

.
.
.
.

.
.
.
.

.
.
.
.

.
.
.
.

.
.
.
.

.
.
.
.

.
.
.
.

III

Ejercicios de repaso
47
9
Ejercicios de repaso . . . . . . . . . . . . . . . . . . . . . . . . . . . . . . . . . 47

IV

Ejercicios avanzados
83
10 Ejercicios avanzados . . . . . . . . . . . . . . . . . . . . . . . . . . . . . . . . . 83

Ejemplos
91
11 Ejemplos . . . . . . . . . . . . . . . . . . . . . . . . . . . . . . . . . . . . . . . 91

VI

Miscelnea
12 Nmeros naturales. El principio de induccin
13 Progresiones aritmticas . . . . . . . . . . . .
14 Progresiones geomtricas . . . . . . . . . . . .
15 Sucesiones recurrentes . . . . . . . . . . . . .
16 Ejercicios resueltos. Seleccin . . . . . . . . .
17 Recurrencia en combinatoria . . . . . . . . . .
18 Ampliacin de nmeros combinatorios . . . .

Bibliografa

.
.
.
.

.
.
.
.

.
.
.
.
.
.
.

.
.
.
.
.
.
.

.
.
.
.
.
.
.

.
.
.
.
.
.
.

.
.
.
.
.
.
.

.
.
.
.
.
.
.

.
.
.
.
.
.
.

.
.
.
.
.
.
.

.
.
.
.
.
.
.

.
.
.
.
.
.
.

.
.
.
.
.
.
.

.
.
.
.
.
.
.

.
.
.
.
.
.
.

.
.
.
.
.
.
.

.
.
.
.
.
.
.

.
.
.
.
.
.
.

.
.
.
.
.
.
.

.
.
.
.
.
.
.

19
20
31
35
41

111
111
114
117
118
121
122
131
135

2
ndice alfabtico

28 de enero de 2013

NDICE GENERAL
137

Curso 20122013. NOTAS DE TRABAJO, 30

Parte I
Sucesiones recurrentes

Introduccin
Pretendemos en este pequeo panfleto resumir algunas ideas elementales sobre el estudio
de las sucesiones recurrentes, esto es, sucesiones recurrentes, lineales o no, con coeficientes
constantes. Ejemplos de esta teora abundan en mltiples aplicaciones y adems son una
buena excusa para tratar sobre la aritmtica de nmeros: enteros, reales y complejos, y tambin de polinomios.
No hemos pretendido ser exhaustivos en el tratamiento de la teora, de hecho hemos adoptado una presentacin ms bien light, pero hemos procurado que los argumentos utilizados
queden claros y sirvan de ejemplo para posteriores desarrollos. Es evidente que quedan fuera
algunos temas que iremos incluyendo en sucesivas versiones de este texto.
En el desarrollo que hemos hecho comenzamos con una presentacin de las progresiones
aritmticas de orden uno y superior; de forma que tenemos una fcil introduccin a las sucesiones definidas por recurrencia lineal con coeficientes constantes. Luego, para tratar de
hacer un estudio completo de estas ltimas sucesiones, introducimos las progresiones geomtricas y el polinomio caracterstico que stas definen. En este punto un poco de lgebra
Lineal sera necesaria, sobre todo para ver las sucesiones definidas por recurrencia como elementos de un espacio vectorial, y calcular una base del mismo.
El resto del texto se centra en el estudio de ejemplos y ejercicios relacionados con la teora
expuesta. Tal vez se eche en falta una ordenacin ms racional de los ejercicios y problemas
que se presentan, pero dejamos esto al lector. Tambin hemos realizado una recopilacin de
problemas que presentamos sin solucin; la no inclusin de la solucin podemos achacarla
a la falta material de tiempo para completar el texto.
Vamos a hacer responsable a esta falta de tiempo de las numerosas erratas que sin duda
salpicarn el texto por aqu y por all. Por esto agradeceramos al lector interesado en esta
materia cualquier comentario o informacin sobre las mismas con objeto de corregir algunas
de stas en prximas versiones del texto, aunque no somos muy optimistas sobre su total
eliminacin; ni cuartel sin ratas ni libro sin erratas" dice un viejo proverbio castellano.
Hemos preferido hacer uso de la red de internet y de los numerosos textos que en ella viven para documentarnos, no vamos a citar ninguno de ellos en particular, pero desde aqu
agradecemos a los autores su generosidad al compartir con todos su inestimable trabajo. Sin
embargo nos gustara citar dos textos publicados en papel por gente prxima a la organizacin de la Olimpada Matemtica Espaola que nos han ayudado mucho en la elaboracin
de este trabajo.

4
[1] Manual de matemticas para preparacin olmpica de Cristobal Snchez Rubio y Manuel
Ripolls Amela, publicado por la Universita Jaume I de Castelln en 2000.
[2] Sessions de preparaci per a lolimpada matemtica. Publicado por la Societat Catalana
de Matematiques en el ao 1999.

28 de enero de 2013

Curso 20122013. NOTAS DE TRABAJO, 30

Captulo I
Sucesiones
1
2
3
4

Progresiones aritmticas . . . . . . . . . . .
Progresiones aritmticas de order superior
Progresiones geomtricas . . . . . . . . . .
Ejercicios con solucin . . . . . . . . . . . .

.
.
.
.

.
.
.
.

.
.
.
.

.
.
.
.

.
.
.
.

.
.
.
.

.
.
.
.

.
.
.
.

.
.
.
.

.
.
.
.

.
.
.
.

.
.
.
.

.
.
.
.

.
.
.
.

.
.
.
.

.
.
.
.

.
.
.
.

.
.
.
.

6
7
13
14

Introduccin
Sea a0 , a1 , a2 , a3 , . . . una sucesin de nmeros (enteros, racionales, reales o complejos). Vamos a representar esta sucesin por {an }nN , {an }n o simplemente por a si necesitamos
abreviar.
Estamos interesados en aquellas sucesiones en las que cada trmino es construido en funcin de los anteriores mediante frmulas o reglas, que ms adelante determinaremos.
Los primeros ejemplos de esta teora son las progresiones aritmticas y geomtricas.

C AP. I. S UCESIONES

1.

Progresiones aritmticas

Un tipo especial de sucesiones son las progresiones aritmticas. stas son sucesiones {an }n
para las que existe un nmero d, la diferencia, tal que el trmino an+1 est definido como
an+1 = an + d, para cada n 0.
Es claro que en una progresin aritmtica {an }n de diferencia d se tiene
an = a0 + n d.
sta es la expresin del trmino general de la progresin.
Dada una sucesin {an }n , llamamos msima suma parcial a
Sm = a0 + + am .
Podemos calcular los valores de las sumas parciales de forma sencilla; basta hacer induccin
sobre m; se obtiene
(a0 + am )(m + 1)
Sm =
.
2
D EMOSTRACIN . El resultado es cierto para m = 0. Suponemos t 1, que el resultado es
cierto para m = t y vamos a ver qu ocurre para m = t + 1. Tenemos
St+1 = St + at+1 =

(a0 +at )(t+1)


2

+ at+1

a0 (t+1)+a0 (t+1)+td(t+1)+2a0 +2(t+1)d


2

(a0 +a0 +(t+1)d)(t+2)


2

a0 2(t+2)+(t+1)(t+2)d
2

(a0 +at+1 )(t+2)


.
2


Observa que tambin se tiene Sm = (m + 1)a0 +
Ver el ejercicio (4.1.).
Otros ejercicios relacionados: (4.1.).

28 de enero de 2013

m(m+1)
d.
2

Curso 20122013. NOTAS DE TRABAJO, 30

S EC . 2. P ROGRESIONES ARITMTICAS DE ORDER SUPERIOR

2.

Progresiones aritmticas de order superior

Diferencias
Dada una sucesin {an }n llamamos diferencia de orden uno de {an }n a la sucesin {an }n ,
en donde
an = an+1 an .
Podemos representar esta sucesin tambin por {an }n .
Por recurrencia se define la diferencia de orden k > 1 de {an }n como la sucesin
k {an }n = k1 {an }n .
La definicin se completa tomando 0 {an }n = {an }n .
La expresin de los trminos de k {an }, en funcin de los an , se calcula fcilmente por induccin sobre k. En efecto, se tiene
k

an =

k
X
i=0

 
k
(1)
a
.
i n+ti
i

D EMOSTRACIN . Hacemos la demostracin para el trmino a0 ; para otros valores basta sumar una constante a cada uno de los ndices.
Tenemos a0 = a1 a0 , entonces 2 a0 = a1 a0 = a2 2a1 + a0 . Supongamos que t 1
y hacemos la hiptesis
 
t
X
i t
t
(1)
a0 =
a .
i ti
i=0

Entonces:
t+1 a0 = t a1 t a0


P
P
= ti=0 (1)i ti at+1i ti=0 (1)i ti ati


P
P
i t a
t
= at+1 + ti=1 (1)i ti at+1i t1
(1)
i=0
i ti (1) a0

Pt
P
t
t
t
= at+1 + i=1 (1)i i at+1i i=1 (1)i1 i1 at+1i (1)t a0
 
P
t 
= at+1 + ti=1 (1)i ti + i1
at+1i (1)t a0

Pt
t
= at+1 + i=1 (1)i t+1
i at+1i (1) a0

Pt+1
= i=0 (1)i t+1
i at+1i

La sucesin {an }n es una progresin aritmtica de grado k 0 si la sucesin k {an }n es
constante.
Las progresiones aritmticas de la seccin (1) son progresiones aritmticas de grado uno.
ARITMTICA Y COMBINATORIA
(I. Sucesiones recurrentes)

P. Jara

C AP. I. S UCESIONES

8
Si {an }n es una progresin aritmtica de grado k, podemos escribir:
a0
2 a0
3 a0
..
.

= a1 a0
= a2 2a1 + a0
= a3 3a2 + 3a1 a0


k a0 = ak k1 ak1 + + (1)k1

k
k+1 a0 = ak+1 k+1
1 ak + + (1)
En resumen:

k
X

a0 =

i=0

k 
k1a1
k+1
k a1

+ (1)k a0 = cte.
+ (1)k+1 a0 = 0

 
k
a .
(1)
i ki
i

(I.1)

Surge ahora la pregunta de si el valor de ak+1 se puede calcular en funcin de los j a0 , con
0 j k + 1. La respuesta es s.
D EMOSTRACIN . Consideramos las sucesiones de las diferencias por columnas:
a0
a1
a2
a3
..
.

2 a0
2 a1
2 a2
2 a3
..
.

a0
a1
a2
a3
..
.

3 a0 . . .
3 a1 . . .
3 a2 . . .
3 a3 . . .
..
.

Es claro que esta tabla se puede tambin formar, a partir de la primera fila, haciendo sumas,
en vez de diferencias. Veamos
esos clculos.
como hacer

1a
a
=
a
+

1
1
Se tiene a1 = a0 +1 a0 , y 1 a1 = 1 a0 + 2 a0 , luego a2 = a0 +21 a0 +2 a0 . Suponemos

a1 = a0 + 1 a0


P
P
que para t 1 se verifica at = ti=0 ti i a0 , y anlogamente 1 at = ti=0 ti i a0 , y vamos
a probar que el mismo resultado se tiene para at+1 .
at+1 = at + 1 at
=

P

P

P

t
n i
i=0 i a0

P

t 0
0 a0

Pt

t 0
0 a0

Pt

=
28 de enero de 2013

t
n i
i=0 i a0

i=1
i=1

t
t  i+1
a0
i=0 i

t+1
t  i
i=1 i1 a0

t
t  i
+
i
i1
t+1 i
i

t  t+1
a0
t

t  t+1
a0
t

Pt+1

t+1 i
i=0
i a0 .

Curso 20122013. NOTAS DE TRABAJO, 30

S EC . 2. P ROGRESIONES ARITMTICAS DE ORDER SUPERIOR


En resumen
an =

n  
X
n
i=0

i a0 .

(I.2)


Observa que la expresin (I.2) es un polinomio en n. Como consecuencia de este desarrollo


tenemos el siguiente resultado:
Teorema. 2.1.
Una sucesin {an }n es una progresin aritmtica de grado k si, y slo si, existe un polinomio
F de grado k tal que an = F(n) para cada entero natural n.
D EMOSTRACIN . La condicin suficiente es consecuencia del hecho expresado en la relaPn n i
Pk
n i
cin (I.2) de la pgina 9. El trmino an es: an =
i=0 i a0 =
i=0 i a0 , que es un
polinomio de grado k en n, ya que i a0 = 0, si i > k, y ni = 0, si i > n.
Para la condicin necesaria basta comprobar que para cada ndice n, si F(n) es un polinomio
en n de grado k, entonces el polinomio F(n + 1) F(n) es de grado menor o igual que k 1
en n.


El trmino general de una progresin aritmtica


Sea {an }n una sucesin de grado k. Como se tiene k+1 an = 0 para cada ndice n, la relacin
(I.3) de la pgina 9, escrita para el ndice k + 1 es:
0=

k+1
X

(1)

i=0


k+1
ak+1i ,
i

y por tanto el trmino ak+1 se escribe:


ak+1



k+1
X
i+1 k + 1
=
(1)
ak+1i .
i

(I.3)

i=1

La misma relacin es vlida para ndices mayores que k + 1.


Por otro lado, de la relacin (I.2) se tiene que el trmino an se expresa
n  
X
n
an =
i a0 ,
i

(I.4)

i=0

que como ya conocemos es un polinomio en n de grado como mximo k.


ARITMTICA Y COMBINATORIA
(I. Sucesiones recurrentes)

P. Jara

C AP. I. S UCESIONES

10

Ejemplo. 2.2.
Consideramos la sucesin aritmtica definida por
a0 = 1, con diferencia d = 3,
esto es, la sucesin: 1, 4, 7, 10, 13, . . .
Se verifica:
n

0 an

1 an

2 an

0
1
2
3
4

1
4
7
10
13

3
3
3
3

0
0
0

Tenemos una progresin aritmtica de grado uno, esto es, k = 1, por lo que la frmula (I.3)
se escribe
 
 
2
2
a2 =
a1
a0 = 2a1 a0 ,
1
2
y podemos cambiar los ndices obteniendo an+1 = 2an+1 an .
La frmula que se obtiene de (I.4) es:
 
 
n
n
0
an =
a0 +
1 a0 = 1 + 3n.
0
1

Sucesiones de potencias
Un caso especial de progresin aritmtica de grado superior lo proporcionan las progresiones
de potencias: {nk }n . Vamos a estudiar en este caso como calcular las sumas parciales.
Llamamos Snk a la suma 0 + 1 + 2k + 3k + + nk .
Si k = 0 este valor es conocido. Veamos el caso general k 1.
Se verifica, por el desarrollo del binomio:
k1  
X
k
k
k
(m + 1) m =
mi .
i
i=0

Sumando en m, desde 1 hasta n, resulta:


(n + 1)k 1 =
=

Pn

k
m=1 [(m + 1)

Pk1
i=0

=n+
28 de enero de 2013

mk ] =

k Pn
i
m=1 m
i

Pk1
i=1

Pn

Pk1

Pk1

k i
i Sn

m=1

i=0

i=0

k i
i m

k i
i Sn .

Curso 20122013. NOTAS DE TRABAJO, 30

S EC . 2. P ROGRESIONES ARITMTICAS DE ORDER SUPERIOR

11

Se obtiene entonces la relacin:




k2  
X
k i
k
k
k1
S .
Sn = (n + 1) (n + 1)
i n
k1

(I.5)

i=1

Los primeros ejemplos son:


Sn1 = 21 ((n + 1)2 (n + 1)) =
Sn2 =
=
=
=
=

n2 +n
2

P1
3 i
1
3
i=1 i Sn )
3 ((n + 1) (n + 1)
1 3
2
1
3 (n + 3n + 3n + 1 n 1 3Sn )
1 3
3 2
2
3 (n + 3n + 2n 2 (n + n))
1
3
2
6 (2n + 3n + n)
n(n+1)(2n+1)
6

(I.6)


P
Sn3 = 14 ((n + 1)4 (n + 1) 2i=1 4i Sni )
= 14 (n4 + 4n3 + 6n2 + 3n (4Sn1 + 6Sn2 ))
2

= 41 (n4 + 2n3 + n2 ) = n(n+1)
2
El clculo explcito de Sni se hace fcilmente a partir de aqu de forma recursiva.
Ver el problema (9.14.).

Aplicacin al clculo del trmino general de una progresin aritmtica


Consideramos una progresin aritmtica {an }n de grado k > 1.
Supongamos que no conocemos el trmino general; vamos a dar un algoritmo para calcularlo
determinando previamente los trminos generales de las sucesiones de menor grado que
aparecen asociadas a {an }n .
Construimos la tabla siguiente:
a0
a1
a2
a3
a4

1 a0
1 a1
1 a2
1 a3
1 a4

2 a0
2 a1
2 a2
2 a3
2 a4

...
...
...
...
...

k2 a0
k2 a1
k2 a2
k2 a3
k2 a4

k1 a0
k1 a1
k1 a2
k1 a3
k1 a4

k a0
k a1
k a2
k a3
k a4

Por hiptesis la sucesin k {an }n es constante, por lo tanto su trmino general es sencillo de
calcular.
La sucesin k1 {an }n es de grado uno, su primer trmino es k1 a0 y su diferencia es exactamente la constante k a0 , luego su trmino general es:
k1 an = k1 a0 + nk a0 .
ARITMTICA Y COMBINATORIA
(I. Sucesiones recurrentes)

P. Jara

C AP. I. S UCESIONES

12

Para calcular el trmino general de la sucesin k2 {an }n consideramos la suma de los trminos de la sucesin k1 {an }n , sea sta
k1
Sm
= k1 a0 + + k1 am , para m 0.

Observa que se tiene


k1
Sm
k1 am = k1 a0 + + k1 am1 = k2 am k2 a0 .

De aqu, cambiando el ndice de m a n, se tiene


k2 an = Snk1 + k2 a0 k1 an ,
que nos da el trmino general de la sucesin k2 {an }n . Adems el clculo de Snk1 se realiza
sumando las potencias de n; en este caso el trmino general de k1 {an }n est dado por un
polinomio de grado uno, k1 an = k1 a0 + nk a0 , y por tanto
Pn

k1 a

i


Pi=0
n
k1
k
= i=0 a0 + i a0
Pn  k
= (n + 1)k1 a0 +
i=0 i a0
n(n+1) k
k1
= (n + 1) a0 + 2 a0 .

Snk1 =

Una vez conocidos los trminos generales de las sucesiones k {an }n , k1 {an }n , . . . , ki {an }n ,
podemos calcular el trmino general de ki1 {an }n siguiendo el mismo procedimiento, as
se obtendr
ki1 an = Snki + ki1 a0 ki1 an ,
y el clculo de Snki se realiza conociendo las sumas de las primeras potencias de los primeros
n + 1 enteros naturales.
Ver el problema (4.2.).

28 de enero de 2013

Curso 20122013. NOTAS DE TRABAJO, 30

S EC . 3. P ROGRESIONES GEOMTRICAS

3.

13

Progresiones geomtricas

Un segundo tipo de sucesin son las progresiones geomtricas; stas son sucesiones {an }n
para las que existe un nmero r, al que vamos a llamar la razn tal que el trmino an+1 est
definido como
an+1 = an r,
para cada n N. Podemos probar por induccin que an = a0 r n .
Dada una progresin geomtrica {an }n , consideramos la msima suma parcial a
Sm = a0 + + am .
Los valores de las sumas parciales podemos calcularlos de forma sencilla: En el caso en que
r = 1 se tiene que an = a0 para cada ndice n, luego Sm = (m + 1)a0 . En el caso en que r 6= 1
tenemos:
Sm = a0 + + am
= a0 (1 + r + + r m )
m

= a0 (1+r++r
1r

ARITMTICA Y COMBINATORIA
(I. Sucesiones recurrentes)

)(1r)

m+1

= a0 1r
1r .

P. Jara

C AP. I. S UCESIONES

14

4.

Ejercicios con solucin

Sucesiones
Ejercicio. 4.1.
Se considera un cuadrado 5 5 con algunas casillas marcadas.

74
186
103
0
Es posible completar el cuadrado sabiendo que cada fila y cada columna forma una progresin aritmtica (de grado uno)?
[From Erds to Kiev. Problems of Olympiad Caliber. MAA, 1996, pag. 93]
SOLUCIN
S OLUCIN . Ejercicio (4.1.)
Consideramos la primera columna, sea 4d, 3d, 2d, d, 0, entonces podemos completar con x
la cuarta fila, comenzando sta por d, x, 103, y la tercera fila que es: 2d, y, , , 186. Tenemos
entonces parte de la segunda columna, 74, y, x.
4d
3d
2d

74
186

103
x
d
0
Escribiendo las relaciones tenemos:
Cuarta fila. x d = 103 x, equivalentemente x =

103+d
2 .

Segunda columna. 74 y = y x, o equivalentemente y =

74+x
2

251+d
4 .

Tercera fila. 186 = 2d + 4d0 = y + 3d0 .


La ltima relacin da las siguientes:
d0 = y 2d =

251 7d
4

2d + 4d0 = 186.

Se obtiene entonces d = 13, x = 103+13


= 58 e y = 251+13
= 66. Entonces la diferencia en la
2
4
progresin de la segunda columna es: 8 y esto permite completar el cuadrado.
El resultado es:
28 de enero de 2013

Curso 20122013. NOTAS DE TRABAJO, 30

S EC . 4. E JERCICIOS CON SOLUCIN


30

52

35

82
74

112

142

172

109

144

26

66

146

13
0

58

106
103

179
186

148

193

100
3

150
2

200
7

39

40
45
50

15

13

50
8


Ejercicio. 4.2.
Determinar el trmino general de la sucesin 1, -7, -45, -23, 389, 1881, 5623, 13385, 27657,
51769.
SOLUCIN
S OLUCIN . Ejercicio (4.2.)
Primero comprobamos si son trminos de una progresin aritmtica calculando las diferencias sucesivas:
n
an an 2 an 3 an 4 an 5 an
0
1
8
30
90
240
120
1
7 38
60
330
360
120
2 45
22
390
690
480
120
3 23 412 1080 1170
600
120
4 389 1492 2250 1770
720
120
5 1881 3742 4020 2490
840
6 5623 7762 6510 3330
7 13385 14272 9840
8 27657 24112
9 51769
Es pues una progresin aritmtica de orden cinco. Se trata ahora de averiguar cul es su trmino general, ver pgina 9 y siguiente.
El trmino general de 5 {an }n es 5 an = 120.
El trmino general de 4 {an }n es: 4 an = 4 a0 + n5 a0 = 240 + 120n = 120(n + 2).
Para calcular el trmino general de 3 {an }n tenemos que calcular
Sn4 =

n
X
i=0

4 ai =

n
X

120(i + 2) = 120

i=0

n
X
(2 + n + 2)(n + 1)
= 60(n + 4)(n + 1).
(i + 2) = 120
2
i=0

A partir de aqu calculamos 3 an .


3 an = Sn4 + 3 a0 4 an = 60(n + 4)(n + 1) + 90 120(n + 2) = 30(2n2 + 6n + 3).
ARITMTICA Y COMBINATORIA
(I. Sucesiones recurrentes)

P. Jara

C AP. I. S UCESIONES

16
Para calcular el trmino general de 2 {an }n tenemos que calcular
Sn3 =

n
X
i=0

3 ai =

n
X

(30(2i2 + 6i + 3)) = 30 2

n
X

i2 + 6

n
X

i+3

n
X

!
1

i=0
i=0
i=0

i=0
n(n + 1)
n(n + 1)(2n + 1)
+6
+ 3(n + 1) = 10(2n3 + 12n2 + 19n + 9)
= 30 2
6
2

Ahora calculamos 2 an .
2 an = Sn3 +2 a0 3 an = 10(2n3 +12n2 +19n+9)3030(2n2 +6n+3) = 10(2n3 +6n2 +n3).
Para calcular el trmino general de {an }n tenemos que calcular
Sn2

n
X

n
X
ai =
(10(2i3 + 6i2 + i 3))
2

i=0

i=0

= 10 2

n
X

i3 + 6

i=0


= 10 2

n(n + 1)
2

2

n
X

i2 +

i=0

n
X

i3

i=0

n
X

!
1

i=0

!
n(n + 1)(2n + 1) n(n + 1)
+6
+
3(n + 1)
6
2
= 5(n4 + 6n3 + 8n2 3n 6).

Ahora calculamos an .
an = Sn2 + a0 2 an
= 5(n4 + 6n3 + 8n2 3n 6) 8 10(2n3 + 6n2 + n 3)
= 5n4 + 10n3 20n2 25n 8.
Para calcular el trmino general de {an }n tenemos que calcular
Sn1 =

n
X

ai =

i=0

n
X

(5i4 + 10i3 20i2 25i 8)

i=0

=5

n
X
i=0

i + 10

n
X
i=0

i 20

n
X
i=0

i 25

n
X
i=0

i8

n
X

i=0



2
n(n + 1)
n(n + 1)(2n + 1)
n(n + 1)(2n + 1)(3n2 + 3n 1)
+ 10
20
=5
30
2
6
n(n + 1)
25
8(n + 1) = n5 + 5n4 20n2 24n 8.
2
2
P
+3n1)
En donde hemos utilizado que ni=0 i4 = n(n+1)(2n+1)(3n
.
30


28 de enero de 2013

Curso 20122013. NOTAS DE TRABAJO, 30

S EC . 4. E JERCICIOS CON SOLUCIN

17

Ahora calculamos an .
an = 0 an = Sn1 + 0 a0 1 an
= n5 + 5n4 20n2 24n 8 + 1 (5n4 + 10n3 20n2 25n 8)
= n5 10n3 + n + 1.
En consecuencia el trmino general de la sucesin {an }n es:
an = n5 10n3 + n + 1.
Otra solucin.
De la ecuacin (I.2) se deduce que el trmino general de la sucesin {an }n es:
an =

n
X
i=0

 
n
=
i a0 ,
i

por lo tanto en este caso resulta:


 
 
 
 
 
 
n
n
n
n
n
n
1
2
3
4
an =
a0 +
a0 +
a0 +
a0 +
a0 +
5 a0
0
1
2
3
4
5
n(n 1) 2
n(n 1)(n 2) 3
n(n 1)(n 2)(n 3) 4
a0 +
a0 +
a0
= a0 + na0 +
2
6
24
n(n 1)(n 2)(n 3)(n 4) 5
+
a0
120
= 18n15n(n1)+15n(n1)(n2)+10n(n1)(n2)(n3)+n(n1)(n2)(n3)(n4)
= n5 10n3 + n + 1.
El trmino general de la sucesin es:
an = n5 10n3 + n + 1.


ARITMTICA Y COMBINATORIA
(I. Sucesiones recurrentes)

P. Jara

18

28 de enero de 2013

C AP. I. S UCESIONES

Curso 20122013. NOTAS DE TRABAJO, 30

Captulo II
Sucesiones recurrentes
5
6
7
8

Sucesiones recurrentes homogneas . .


Sucesiones recurrentes no homogneas
Funciones generatrices . . . . . . . . . .
Ejercicios con solucin . . . . . . . . . .

.
.
.
.

.
.
.
.

.
.
.
.

.
.
.
.

.
.
.
.

.
.
.
.

.
.
.
.

.
.
.
.

.
.
.
.

.
.
.
.

.
.
.
.

.
.
.
.

.
.
.
.

.
.
.
.

.
.
.
.

.
.
.
.

.
.
.
.

.
.
.
.

.
.
.
.

.
.
.
.

20
31
35
41

Introduction
La frmula (I.3) de la pgina 9




k+1
k+1 k + 1
ak+1 =
ak + + (1)
a1 + (1)k+2 a0
1
k
nos indica que los trminos de una sucesin aritmtica de orden superior se pueden construir de forma recurrente. En este captulo vamos a adentrarnos en el estudio de este tipo
ms general de sucesiones.

C AP. II. S UCESIONES RECURRENTES

20

5.

Sucesiones recurrentes homogneas

Una sucesin {an }n se llama recurrente de orden t si existen nmeros c1 , . . . , ct , con ct 6= 0, y


un ndice m 0 tal que para cada n m se verifica:
an+t = an+t1 c1 + + an ct .

(II.1)

Podemos suponer, prescindiendo de los trminos necesarios, que en la definicin se verifica


m = 0.
Una sucesin recurrente de orden t est definida por dos t-uplas,
- una los elementos c1 , . . . , ct , con ct 6= 0, y
- otra los t primeros trminos de la sucesin a0 , . . . , at1 , a los que llamaremos condiciones iniciales de la sucesin. El resto de los elementos se define por:
an+t = an+t1 c1 + + an ct ,

(n 0).

(II.2)

Llamamos a (II.1) a (II.2) la ecuacin de recurrencia de la sucesin.


Ejemplo. 5.1.
Cada progresin aritmtica de orden k es una sucesin recurrente de orden k + 1.
Ver pgina 9.
Ejemplo. 5.2.
Cada progresin geomtrica es una sucesin recurrente de orden 1.
La sucesin
a0 = a, a1 = ar, a2 = ar 2 , . . . , an = ar n . . .
cumple con la ecuacin recurrente an+1 = an r, para n 0.
Ejemplo. 5.3. (Sucesin de Fibonacci.)
El prototipo de sucesin recurrente es la sucesin de Fibonacci definida:
a0 = 0,
a1 = 1,
an+2 = an+1 + an , la ecuacin de recurrencia para n 0.
El problema que estudia Fibonacci es la evolucin de una poblacin de conejos sometida a
las siguientes reglas:
(1) Se inicia el proceso con una pareja recin nacida.
(2) Cada pareja es frtil al cabo de un mes, y se reproduce cada mes dando lugar a una nueva
pareja.
(3) En el momento cero no hay ninguna pareja.
28 de enero de 2013

Curso 20122013. NOTAS DE TRABAJO, 30

S EC . 5. S UCESIONES RECURRENTES HOMOGNEAS

21

La evolucin es la siguiente: Al inicio del primer periodo se introduce una pareja. Al inicio
del segundo periodo solamente tenemos una pareja. Al inicio del tercer periodo tenemos dos
parejas. Al inicio del cuarto periodo tenemos tres parejas, y as sucesivamente.
La siguiente es una representacin grfica de este proceso:
Periodo

Parejas

Nm. Parejas

(0

1 JJJ

JJ
JJ
JJ
JJ

JJ
JJ
1 AA
JJ
JJ
AA
JJ
AA
JJ
AA
JJ
JJ
AA

$
AA
13
1AA
33
-AA
AA
33
-A
33
AA

 -33
33
1
1
1 --33
-33
-33
-





(1

(1

(2

(3

(5

En este caso an es el nmero de parejas, de nuestra poblacin de conejos, al inicio del periodo
n.
La sucesin es: 0, 1, 1, 2, 3, 5, 8, 13, 21, 34, 55, . . .

El espacio vectorial
Para cada t nmeros: c1 , . . . , ct , ct 6= 0, el conjunto de las sucesiones recurrentes con ecuacin
recurrente
an+t = an+t1 c1 + + an ct .
(n 0)
(II.3)
tiene estructura de espacio vectorial de dimensin t, ya que tenemos en principio t +1 grados
de libertad, uno para cada ai , i = n, n + 1, . . . , n + t, y una restriccin, justo la que da la
ecuacin (II.3).
Podemos calcular una base de este espacio, pero vamos primero a ver qu progresiones geomtricas podemos encontrar en este espacio vectorial.
Consideramos una progresin geomtrica, por ejemplo an = an1 r = a0 r n , con a0 6= 0 y r 6= 0
para evitar casos triviales.
Se verifican entonces las igualdades
a0 r n+t = a0 r n+t1 c1 + + a0 r n ct .
ARITMTICA Y COMBINATORIA
(I. Sucesiones recurrentes)

(II.4)
P. Jara

C AP. II. S UCESIONES RECURRENTES

22

r t = r t1 c1 + + rct1 + ct .

(II.5)

Llamamos a sta la ecuacin caracterstica de la sucesin.


Cada solucin de esta ecuacin determina una progresin geomtrica {1, , 2 , . . .} que
pertenece al espacio vectorial.
Cuando tiene multiplicidad al menos dos tenemos en este mismo espacio otra sucesin
recurrente, ahora no geomtrica, la definida por 0, , 22 , 33 , . . .. Para comprobarlo, como
es tambin raz de la derivada de r t r t1 c1 ct , entonces es raz del polinomio:
tr t1 (t 1)r t2 c1 2rct2 ct1 ,

(II.6)

y al multiplicar por r, tambin es raz de


tr t (t 1)r t1 c1 2r 2 ct2 rct1 .

(II.7)

De esta forma la sucesin 0, , 22 , 33 , . . . tambin verifica la ecuacin (II.4) y pertenece el


espacio de soluciones..
Cuando la multiplicidad de es tres o superior podemos seguir un proceso similar para encontrar nuevas sucesiones. Veamos el caso de tres: Tras derivar tenemos que es raz de
t(t 1)r t2 (t 1)(t 2)r t3 c1 3 2rct3 2ct2 ,

(II.8)

multiplicando por r 2 se obtiene que es raz de:


t(t 1)r t (t 1)(t 2)r t1 c1 3 2r 3 ct3 2r 2 ct2 .

(II.9)

Ahora es fcil comprobar que (II.7)+(II.9) nos da


t 2 r t (t 1)2 r t1 c1 32 r 3 ct3 2r 2 ct2 r 2 ct1 ,

(II.10)

y que es raz de este polinomio, tenemos entonces la solucin: 0, , 22 2 , 32 3 , . . .


Para multiplicidades superiores se obtienen resultados anlogos, ya que basta desarrollar los
productos t(t 1)(t 2), t(t 1)(t 2)(t 3), etc., y obtener la mxima potencia de t en
trminos de las restantes.
De esta forma podemos obtener nuevas sucesiones recurrentes en el mismo espacio vectorial. De hecho si la multiplicidad de es s, obtenemos la siguientes sucesiones recurrentes,
que son linealmente independientes:

28 de enero de 2013

1:
2:
3:
..
.

1,
0,
0,

,
,
,

2 ,
22 ,
22 2 ,

3 , . . .
33 , . . .
32 3 , . . .

s:

0,

2s1 2 ,

3s1 3 , . . .

Curso 20122013. NOTAS DE TRABAJO, 30

S EC . 5. S UCESIONES RECURRENTES HOMOGNEAS

23

Si las races del polinomio r t r t1 c1 ct son 1 , . . . , h con multiplicidades s1 , . . . , sh ,


respectivamente, uniendo las construcciones para cada una de ellas obtenemos una base del
espacio vectorial formada por s1 + + sh sucesiones recurrentes.
Ejemplo. 5.4. (Sucesin de Fibonacci.)
Vamos a calcular el trmino general de la sucesin de Fibonacci dada por la ecuacin de
recurrencia an+2 = an+1 + an , para n 0 con las condiciones iniciales a0 = 0 y a1 = 1. Ver
ejemplo (5.3.).
En este caso t = 2, y c1 = 1 = c2 .
2
La ecuacin caracterstica
es: r r 1 = 0.
Las races son: 1 = 1+2 5 y 2 = 12 5 , y son races simples.
Una base del espacio de las sucesiones recurrentes est formado por las dos progresiones
geomtricas: {1n }n y {2n }n . Entonces el trmino general an se escribe
an = 1 1n + 2 2n , para cada entero no negativo n.
En donde 1 y 2 son nmeros que vamos a determinar utilizando las condiciones iniciales.
Para n = 0 se verifica: 0 = 1 + 2 , y para n = 1 se tiene: 1 = 1 1 + 2 2 .

1 + 2 = 0
Tenemos pues un sistema lineal
cuyas soluciones son:
1 1 + 2 2 = 1

5
1 =
,
5

2 =

El trmino general de la sucesin de Fibonacci es:

!n
5 1+ 5
5
an =

5
2
5

5
.
5

!n
1 5
.
2

Esta expresin se conoce como la Frmula de Binet,y para cada entero no negativo da siempre valores enteros.
Ver tambin otro ejemplo en el problema (8.5.).

ARITMTICA Y COMBINATORIA
(I. Sucesiones recurrentes)

P. Jara

C AP. II. S UCESIONES RECURRENTES

24

Construccin de sucesiones recurrentes. Producto


Supongamos que tenemos dos sucesiones recurrentes, {an }n y {bn }n dadas por las ecuaciones de recurrencia
an+t = an+t1 c1 + + an ct
bm+s = bm+s1 d1 + + bm ds
podemos suponer que tenemos trminos generales dados por expresiones del tipo siguiente:
P
an = hi=1 pi in ,
P
bn = kj=1 qj jn
en donde pi y qj son polinomios en n, cuyo grado depende de la multiplicidad de la raz i
j , respectivamente en la ecuacin caracterstica.
Definimos una nueva sucesin mediante:
xn = an bn , para cada ndice n.
El trmino general de la nueva sucesin {xn }n es el producto de los trminos de las sucesiones factores.
h
k
X
X
X
n
pi qj (i j )n ,
pi i
qj jn =
xn =
i=1

j=1

i,j

entonces el problema es el clculo de la ecuacin caracterstica de {xn }n ; sta va a depender


de los polinomios pi y qj que aparezcan. Vamos a estudiar dos ejemplos.
Proposicin. 5.5. (Caso cuadrtico)
Consideramos las sucesiones recurrentes
an+2 = an+1 c1 + an c2 y condiciones iniciales a0 , a1 ;
bn+2 = bn+1 d1 + bn d2 y condiciones iniciales b0 , b1 .
Si 1 , 2 son las races de la ecuacin caracterstica r 2 = rc1 + c2 y si 1 , 2 son las races de
la ecuacin caracterstica r 2 = rd1 + d2 , entonces tenemos:
an = 1 1n + 2 2n ,
bn = 1 1n + 2 2n ,
xn = 1 1 (1 1 )n + 1 2 (1 2 )n + 2 1 (2 1 )n + 2 2 (2 2 )n
(Los casos en que existen races mltiples se tratan de la misma manera.)
Como consecuencia el polinomio caracterstico de {xn }n es:
(r 1 1 )(r 1 2 )(r 2 1 )(r 2 2 ),
la cual se puede expresar en funcin de los polinomios caractersticos originales ya que sus
coeficientes son polinomios simtricos en los i y polinomios simtricos en los j .
28 de enero de 2013

Curso 20122013. NOTAS DE TRABAJO, 30

S EC . 5. S UCESIONES RECURRENTES HOMOGNEAS

25

En nuestro caso tenemos la siguiente expresin:


(r 1 1 )(r 1 2 )(r 2 1 )(r 2 2 )


2 2 2 2
2
2
2 2
2
2
2
2
= 1 2 1 2 1 2 1 2 + 1 2 1 2 + 1 2 1 2 + 1 2 1 2 r


+ 1 2 12 + 12 1 2 + 21 2 1 2 + 22 1 2 + 1 2 22 r 2
(1 1 + 2 1 + 1 2 + 2 2 ) r 3 + r 4
= c22 d22 c1 c2 d1 d2 r (c12 d2 + c2 d12 + 2c2 d2 )r 2 c1 d1 r 3 + r 4
Ejercicio. 5.6.
Se consideran las sucesiones recurrentes definidas por
a0 = 0, a1 = 1 y la frmula an+2 = 2an+1 an para n 0 y
b0 = 1, b1 = 2 y la frmula bn+2 = 3an+1 2an para n 2.
Se define la nueva sucesin recurrente con xn = an bn . Vamos a calcular el trmino general
de xn y su ecuacin caracterstica.
Primero el polinomio caracterstico es:
c22 d22 c1 c2 d1 d2 r (c12 d2 + c2 d12 + 2c2 d2 )r 2 c1 d1 r 3 + r 4 ,
y como c1 = 2, c2 = 1, d1 = 3, d2 = 2, tenemos el valor:
r 4 6r 3 + 13r 2 12r + 4.
Las races de este polinomio son 1 = 1, 2 = 1, 3 = 2 y 4 = 2. Los valores iniciales son:
x0
x1
x2
x3

= a0 b0
= a1 b1
= a2 b2
= a3 b3

= 0 1 = 0,
= 1 2 = 2,
= 2 4 = 8,
= 3 8 = 24.

La solucin general es:


xn = (1 + n2 ) + (3 + n4 )2n .
Para averiguar el trmino general de la solucin al problema, basta resolver el sistema de
ecuaciones lineales:

1
+3
= 0,

1 +2 +23 + 24 = 2,
1 +22 +43 + 84 = 8,

1 +32 +83 + 244 = 24


que tiene la solucin 1 = 2 = 3 = 0, 4 = 1.
ARITMTICA Y COMBINATORIA
(I. Sucesiones recurrentes)

P. Jara

C AP. II. S UCESIONES RECURRENTES

26
El trmino general de la solucin es:
xn = n2n .

La comprobacin de que el resultado es correcto es fcil, ya que el trmino general de {an }n


es an = n y el de {bn }b es bn = 2n , y por lo tanto el trmino general del producto es n2n .
Ejercicio. 5.7.
Se considera las sucesiones recurrentes definidas por
a0 = 1, a1 = 1 y la frmula an+2 = 2an+1 an para n 0 y
b0 = 1, b1 = 0 y la frmula bn+2 = 5bn+1 6bn para n 0.
Se define una nueva sucesin recurrente con xn = an bn . Vamos a averiguar el trmino
general de xn y su ecuacin caracterstica.
Primero el polinomio caracterstico es: c22 d22 c1 c2 d1 d2 r (c12 d2 + c2 d12 + 2c2 d2 )r 2 c1 d1 r 3 + r 4 ,
y como c1 = 2, c2 = 1, d1 = 5, d2 = 6, tenemos el valor:
r 4 + 10r 3 + 37r 2 + 60r + 36.
Las races de este polinomio son 1 = 3, 2 = 3, 3 = 2 y 4 = 2. Los valores iniciales
son:
x0 = a0 b0 = 1 1 = 1,
x1 = a1 b1 = 1 0 = 0,
x2 = a2 b2 = 1 (6) = 6,
x3 = a3 b3 = 1 (30) = 30.
Para calcular el trmino general de la solucin al problema calcular una solucin general,
xn = (1 + n2 )(3)n + (3 + n4 )(2)n ,
e imponemos que verifique las condiciones iniciales. Tenemos que resolver el sistema de
ecuaciones lineales:

1
+3
= 1,

(1 + 2 )(3) (3 + 4 )(2) = 0,
(1 + 22 )(3)2 (3 + 24 )(2)2 = 6,

(1 + 32 )(3)3 (3 + 34 )(2)2 = 30
que tiene la solucin 1 = 2, 2 = 0, 3 = 3, 4 = 0. As pues tenemos el trmino general
xn = 2 (3)n + 3 (2)n .
La comprobacin es fcil, ya que el trmino general de {an }n es an = (1)n y el de {bn }b es
bn = 3 2n 2 3n .
28 de enero de 2013

Curso 20122013. NOTAS DE TRABAJO, 30

S EC . 5. S UCESIONES RECURRENTES HOMOGNEAS

27

Ejercicio. 5.8.
Se considera las sucesiones recurrentes definidas por
a0 = 1, a1 = 3 y la frmula an+2 = 2an+1 + an para n 0 y
b0 = 1, b1 = 0 y la frmula bn+2 = 5bn+1 6bn para n 0.
Se define la nueva sucesin recurrente con xn = an bn . Vamos a averiguar el trmino general
de xn y su ecuacin caracterstica.
Primero el polinomio caracterstico es: c22 d22 c1 c2 d1 d2 r (c12 d2 + c2 d12 + 2c2 d2 )r 2 c1 d1 r 3 + r 4 ,
y como c1 = 2, c2 = 1, d1 = 5, d2 = 6, tenemos el valor:
r 4 + 10r 3 + 11r 2 60r + 36.

Las polinomio
races
de
este
son

=
2(1

2),

=
3(1

2),

=
2(1
+
2) y
1
2
3

4 = 3(1 + 2). Los valores iniciales son:


= 1 1 = 1,
= 3 0 = 0,
= 5 (6) = 30,
= 13 (30) = 390.
n
n
n
2))
+

(3(1

2))
+

(2(1
+
2)) +
Para calcular
el
trmino
general,
x
=

(2(1

n
2
3
1

4 (3(1 + 2))n , basta resolver el sistema de ecuaciones lineales:

1
+
+
+
=
1
2
3
4

1 (2(1 2)) +2 (3(1 2)) +3 (2(1 + 2)) +4 (3(1 + 2)) = 0


1 (2(1 2))2 +2 (3(1 2))2 +3 (2(1 + 2))2 +4 (3(1 + 2))2 = 30

1 (2(1 2))3 +2 (3(1 2))3 +3 (2(1 + 2))3 +4 (3(1 + 2))3 = 390


x0
x1
x2
x3

= a0 b0
= a 1 b1
= a 2 b2
= a 3 b3

que tiene la solucin


1 =

3
3
(1 2 2), 2 = 1 + 2 2, 3 = (1 + 2 2), 4 = 1 2 2.
2
2

As pues tenemos el trmino general

n
xn = 32 (1 2
2)(2(1
2)) + (1 + 2
2)(3(1
2))
3
n
+ 2 (1 + 2 2)(2(1 + 2)) + (1 2 2)(3(1 + 2))n .
La comprobacin es fcil, ya que el trmino general de {an }n es
an =

1
1
(1 2)(1 2 2)n + (1 + 2 2)(1 + 2)n
2
2

y el de {bn }b es

ARITMTICA Y COMBINATORIA
(I. Sucesiones recurrentes)

bn = 3 2n 2 3n .
P. Jara

C AP. II. S UCESIONES RECURRENTES

28

Proposicin. 5.9. (Caso cbico)


Consideramos las sucesiones recurrentes
an+3 = an+2 c1 + an+1 c2 + an c3 y condiciones iniciales a0 , a1 , a2 ;
bn+2 = bn+1 d1 + bn d2 y condiciones iniciales b0 , b1 .
Y se define la nueva sucesin recurrente con xn = an bn . Si 1 , 2 , 3 son las races de la
ecuacin caracterstica r 3 = r 2 c1 + rc2 + c3 y si 1 , 2 son las races de la ecuacin caracterstica r 2 = rd1 + d2 , entonces tenemos:
an = 1 1n + 2 2n + 3 3n ,
bn = 1 1n + 2 2n ,
xn = 1 1 (1 1 )n + 1 2 (1 2 )n + 2 1 (2 1 )n + 2 2 (2 2 )n + 3 1 (3 1 )n + 3 2 (3 2 )n
(Los casos en que existen races mltiples se tratan de la misma manera.) Como consecuencia
el polinomio caracterstico de {xn }n es:
(r 1 1 )(r 1 2 )(r 2 1 )(r 2 2 )(r 3 1 )(r 3 2 ),
la cual se puede expresar en funcin de los polinomios caractersticos originales ya que sus
coeficientes son polinomios simtricos en los i y polinomios simtricos en los j .

28 de enero de 2013

Curso 20122013. NOTAS DE TRABAJO, 30

S EC . 5. S UCESIONES RECURRENTES HOMOGNEAS

29

En nuestro caso tenemos la siguiente expresin:


(r 1 1 )(r 1 2 )(r 2 1 )(r 2 2 )(r 3 1 )(r 3 2 )
= 12 22 32 13 23

12 22 3 13 22 + 12 2 32 13 22 + 1 22 32 13 22 + 12 22 3 12 23

+ 12 2 32 12 23 + 1 22 32 12 23 r

+ 12 2 3 13 2 + 1 22 3 13 2 + 1 2 32 13 2 + 12 22 12 22
+ 212 2 3 12 22 + 21 22 3 12 22 + 12 32 12 22 + 21 2 32 12 22

+ 22 32 12 22 + 12 2 3 1 23 + 1 22 3 1 23 + 1 2 32 1 23 r 2

1 2 3 13 + 12 2 12 2 + 1 22 12 2 + 12 3 12 2
+ 31 2 3 12 2 + 22 3 12 2 + 1 32 12 2 + 2 32 12 2
+ 12 2 1 22 + 1 22 1 22 + 12 3 1 22 + 31 2 3 1 22

+ 22 3 1 22 + 1 32 1 22 + 2 32 1 22 + 1 2 3 23 r 3

+ 1 2 12 + 1 3 12 + 2 3 12 + 12 1 2 + 21 2 1 2
+ 22 1 2 + 21 3 1 2 + 22 3 1 2 + 32 1 2 + 1 2 22

+ 1 3 22 + 2 3 22 r 4
(1 1 + 2 1 + 3 1 + 1 2 + 2 2 + 3 2 ) r 5 + r 6

= c32 d23 + c2 c3 d1 d22 r (c1 c3 (d12 d2 + 2d22 ) c22 d22 )r 2


(c2 c1 (d1 d2 ) + c3 (d13 + 3d1 d2 ))r 3 (c2 (d12 + 2d2 ) + c12 d2 )r 4
c1 d1 r 5 + r 6 (II.11)
As pues el polinomio caracterstico es:
c32 d23
+c2 c3 d1 d22 r

c1 c3 (d12 d2 + 2d22 ) c22 d22 r 2 
c2 c1 (d1 d2 ) + c3 (d13 + 3d1 d2 ) r 3
c2 (d12 + 2d2 ) + c12 d2 r 4
c1 d1 r 5
+r 6
ARITMTICA Y COMBINATORIA
(I. Sucesiones recurrentes)

P. Jara

C AP. II. S UCESIONES RECURRENTES

30

Ejercicio. 5.10.
Se considera las sucesiones recurrentes definidas por
a0 = 6, a1 = 5, a2 = 11 y la frmula an+3 = 3an+1 + 2an para n 0 y
b0 = 1, b1 = 0 y la frmula bn+2 = 5bn+1 6bn para n 0.
Se define la nueva sucesin recurrente con xn = an bn . Vamos a averiguar el trmino general
de xn y su ecuacin caracterstica.
Primero el polinomio caracterstico es el dado en la frmula (II.11). Como c1 = 0, c2 = 3,
c3 = 2, d1 = 5, d2 = 6, tenemos el valor:
r 6 39r 4 70r 3 + 324r 2 + 1080r + 864.
Cuyas races son 1 = 3, 2 = 3, 3
son:
x0 = a0 b0
x1 = a1 b1
x2 = a2 b2
x3 = a3 b3
x4 = a4 b4
x5 = a5 b5

= 2, 4 = 2, 5 = 4 y 6 = 6. Los valores iniciales


= 6 1 = 6,
= 5 0 = 0,
= 11 (6) = 66,
= 27 (30) = 810
= 43 (114) = 4902
= 103 (390) = 40170.

Para calcular el trmino general, xn = (1 + n2 )(3)n + (3 + n4 )(2)n + 5 4n + 6 6n , basta


resolver el sistema de ecuaciones lineales:

(1 + 02 )(3)0 + (3 + 04 )(2)0 + 5 40 + 6 60 = 6,

(1 + 12 )(3)1 + (3 + 14 )(2)1 + 5 41 + 6 61 = 0,

2
2
2
2
(1 + 22 )(3) + (3 + 24 )(2) + 5 4 + 6 6 = 66,
(1 + 32 )(3)3 + (3 + 34 )(2)3 + 5 43 + 6 63 = 810,

(1 + 42 )(3)4 + (3 + 44 )(2)4 + 5 44 + 6 64 = 4902,

5
5
5
5
(1 + 52 )(3) + (3 + 54 )(2) + 5 4 + 6 6 = 40170,
que tiene la solucin 1 = 6, 2 = 4, 3 = 9, 4 = 6, 5 = 9 y 6 = 6. As pues tenemos el
trmino general
xn = (6 + 4n)(3)n + (9 6n)(2)n + 9 4n 6 6n .
La comprobacin es fcil, ya que el trmino general de {an }n es
an = (3 2n)(1)n + 3 2n ,
y el de {bn }n es

28 de enero de 2013

bn = 3 2n 2 3n .

Curso 20122013. NOTAS DE TRABAJO, 30

S EC . 6. S UCESIONES RECURRENTES NO HOMOGNEAS

6.

31

Sucesiones recurrentes no homogneas

Las sucesiones recurrentes no homogneas son las soluciones a ecuaciones del tipo
an = an1 c1 + + ant ct + f (n),

(n t),

(II.12)

en donde f (n) es una funcin de n.


Para distinguir esta sucesin recurrente de las sucesiones antes introducidas, las vamos a
llamar sucesiones recurrentes no homogneas.
Las sucesiones recurrentes no homogneas no constituyen un espacio vectorial, como fcilmente se puede observar. Pero s podemos considerar el espacio vectorial de las sucesiones
recurrentes (homogneas) asociado; esto es, el definido por la ecuacin (homognea)
an = an1 c1 + + ant ct ,

(n t),

(II.13)

de forma que cada sucesin recurrente que verifica la ecuacin (II.12) se va a obtener como
una combinacin lineal de una solucin particular de (II.12) y una combinacin lineal de
sucesiones en el espacio vectorial definido por la ecuacin (II.13).

Una construccin de sucesiones recurrentes. La suma


Supongamos que tenemos dos sucesiones recurrentes, {an }n y {bn }n dadas por las ecuaciones de recurrencia
an = an1 c1 + + ant ct
bm = bm1 d1 + + bms ds
podemos suponer que tenemos trminos generales dados por expresiones del tipo siguiente:
P
an = hi=1 pi in ,
P
bn = kj=1 qj jn
en donde pi y qj son polinomios en n, cuyo grado depende de la multiplicidad de la raz i
j , respectivamente en el polinomio (ecuacin) caracterstica.
Definimos una nueva sucesin mediante:
xn = an + bn , para cada ndice n.
El trmino general de la nueva sucesin {xn }n es la suma de los trminos de las sucesiones.
X
X
xn =
pi in +
qj jn ,
i

entonces el problema es el clculo de la ecuacin caracterstica de {xn }n ; sta depende de los


polinomios pi y qj que aparezcan y de las races i y j .
ARITMTICA Y COMBINATORIA
(I. Sucesiones recurrentes)

P. Jara

C AP. II. S UCESIONES RECURRENTES

32

Cuando las races i y j son distintas, resulta que el polinomio caracterstico de la sucesin
{xn }n , es exactamente el producto de los polinomios caractersticos de {an }n y {bn }b .
Cuando existe una raz comn, entonces el polinomio caracterstico se obtiene a partir del
producto de los polinomios caractersticos, pero considerando las races comunes con la mxima multiplicidad.
En cualquier caso podemos siempre suponer que las races son distintas y tomar como polinomio caracterstico el producto de los dos polinomios caractersticos; la razn es que en
cualquier caso la sucesin {xn }n pertenece al espacio vectorial determinado por el producto
de los dos polinomios.
Ejercicio. 6.1.
Consideramos las sucesiones
{an }n definida por a0 = 6, a1 = 5, a2 = 11 y an = 3an2 + 2an3 , n 3 y
{bn }n definida por b0 = 1, b1 = 0 y bn = 5bn1 6bn2 , n 2.
Se define la nueva sucesin recurrente con xn = an + bn . Vamos a averiguar el trmino
general de xn y su ecuacin caracterstica.
Primero calculamos el trmino general de {an }n , que es: an = (3 2n)(1)n + 3 2n ; su
polinomio caracterstico es: r 3 3r 2 = (r + 1)2 (r 2).
Ahora calculamos el trmino general de {bn }n , que es: bn = 3 2n 3 3n ; su polinomio
caracterstico es: r 2 5r + 6 = (r 2)(r 3).
El trmino general de {xn }n es:
xn = (3 2n)(1)n + 3 2n + 3 2n 3 3n = (3 2n)(1)n + 6 2n 3 3n .
Su polinomio caracterstico es (r + 1)2 (r 2)(r 3), pero es tambin una sucesin asociada
a (r + 1)2 (r 2)2 (r 3), y por tanto el trmino general de {xn }n podemos obtenerlo a partir
de cualquiera de estos dos polinomios caractersticos.

Clculo del trmino general


Veamos una aplicacin de la suma de sucesiones recurrentes. Se considera una sucesin recurrente no homognea
an = an1 c1 + + ant ct + f (n),

(n t),

(II.14)

si f (n) es el trmino general de una sucesin recurrente con polinomio caracterstico Q(r),
entonces podemos proceder como sigue: consideramos la frmula de recurrencia homognea
an = an1 c1 + + ant ct ,
(n 0),
(II.15)
el polinomio caracterstico que define, sea P(r) y tomamos una sucesin recurrente del espacio vectorial asociado, sea {hn }n . Entonces an se puede escribir
an = hn + f (n),
28 de enero de 2013

Curso 20122013. NOTAS DE TRABAJO, 30

S EC . 6. S UCESIONES RECURRENTES NO HOMOGNEAS

33

esto es, {an }n es la suma de dos sucesiones: {hn }n y {f (n)}n ; por lo tanto, cuando {f (n)}n
es una sucesin recurrente, conocemos que la sucesin {an }n es una sucesin recurrente
asociada al polinomio P(r)Q(r).
Este simple argumento nos permite calcular fcilmente una solucin particular de la recurrencia no lineal (II.14), pues sabemos calcular soluciones de recurrencias homogneas. En
este clculo, por comodidad, los sumandos que corresponden a P(r) podemos eliminarlos
al obtener la solucin particular, ya que estos aparecern de nuevo al considerar la solucin
general.
Veamos un ejemplo.
Ejemplo. 6.2.
Se considera la relacin de recurrencia no homognea
an = 3an2 + 2an3 + 2n + 5n4 ,
sujeta a las condiciones iniciales a0 = 1, a1 = 2 y a2 = 3.
Vamos a calcular una solucin particular. Consideramos la ecuacin caracterstica que define
una sucesin con 2n + 5n4 como trmino general; en este caso (r 1)5 .
Tenemos que {an }n es la suma de dos sucesiones, una definida por la relacin hn = 3hn2 +
2hn3 , con polinomio caracterstico r 3 3r 2 = (r + 1)2 (r 2), y otra por los trminos
2n + 5n4 . Entonces un polinomio caracterstico para {an }n es:
(r 1)5 (r + 1)2 (r 2)
Una solucin genrica es:
(1 + 2 n + 3 n2 + 4 n3 + 5 n4 ) + (6 + 7 n)(1)n + 8 2n
Como antes hemos sealado, la parte (6 + 7 n)(1)n + 8 2n podemos eliminarla, ya que la
encontraremos de nuevo al obtener la solucin general; as pues trabajamos con 1 + 2 n +
3 n2 + 4 n3 + 5 n4 .
Al imponer que verifica la ecuacin de definicin se tiene:
1 + 2 n + 3 n2 + 4 n3 + 5 n4
= 3(1 + 2 (n 2) + 3 (n 2)2 + 4 (n 2)3 + 5 (2)n4 )
+2(1 + 2 (n 3) + 3 (n 3)2 + 4 (n 3)3 + 5 (n 3)4 )
+2n + 5n4 .
Al desarrollar se obtiene
41 + 122 303 + 784 2105
+(2 42 + 243 904 + 3125 )n
+(43 + 364 1805 )n2
+(44 + 485 )n3
+(5 45 )n4 = 0
ARITMTICA Y COMBINATORIA
(I. Sucesiones recurrentes)

P. Jara

C AP. II. S UCESIONES RECURRENTES

34
De aqu se deducen las relaciones:

41 +122 303 +784 2105 = 0

2
42 +243 904 +3125 = 0

43 +364 1805 = 0

44 +485 = 0

5
45 = 0
que tiene como solucin:
1 =

1341
315
5
, 2 = 233, 3 =
, 4 = 15, 5 = .
4
4
4

Entonces una solucin particular es:

1341
315 2
5
233n
n 15n3 n4
4
4
4

La solucin general es la que se obtiene al sumar a la solucin particular una solucin general
de hn = 3hn2 + 2hn3 , cuya ecuacin caracterstica, como ya sabemos, es: r 3 3r 2 =
(r + 1)2 (r 2). Por tanto la solucin general de la homognea es:
(1 + 2 n)(1)n + 3 2n
y una solucin general de la recurrencia es:

315 2
5
1341
233n
n 15n3 n4 + (6 + 7 n)(1)n + 8 2n
4
4
4

Para calcular 6 , 7 y 8 tenemos considerar las condiciones iniciales:

1341
+
+
=
1

6
8
4

2653
4 6 7 +28 = 2

5025
4 +6 +27 +48 = 3
157
2926
La solucin es: 6 = 401
36 , 7 = 6 , 8 = 9 .
La solucin general de la recurrencia no homognea es:


1341
315 2
5 4
401 157
2926 n
3

233n
n 15n n +

n (1)n +
2
4
4
4
36
6
9

28 de enero de 2013

Curso 20122013. NOTAS DE TRABAJO, 30

S EC . 7. F UNCIONES GENERATRICES

7.

35

Funciones generatrices

Sea {an }n una sucesin de nmeros reales o complejos, podemos definir entonces una funcin formal mediante
f (x) = a0 + a1 x + a2 x2 + a3 x3 +
Recordar que esta funcin formal no define una funcin de R a R de C a C, ya que en general
no est definida la suma infinita. Casos particulares se presentan cuando la sucesin {an }n
es nula a partir de un ndice; en este caso f (x) est definido por un polinomio. Otro caso es
cuando la serie que define f (x) es convergente, entonces f (x) estar definida en una parte de
R C, segn el caso.
Nosotros estamos interesados en los coeficientes, o los trminos de la sucesin que definen
a f (x), por lo que llamamos a f (x) la funcin generatriz de la sucesin {an }n .
Veamos un ejemplo.
Ejemplo. 7.1.
Si consideramos la sucesin constante igual a 1 la funcin generatriz es: f (x) = 1 + x + x2 +
1
.
x3 + , que corresponde al desarrollo en serie de la funcin x 7 1x
Casos ms sencillos se plantean cuando consideramos otras funciones.
Ejemplo. 7.2.
La funcin
   


 
n
n +
n
n n
n1
x 7 (1 + x) =
+
x +
x
+
x
0
1
n1
n
n

es la funcin generatriz cuya sucesin es:


   

  
n
n
n
n
,
,...,
,
, 0, 0, . . .
0
1
n1
n

ARITMTICA Y COMBINATORIA
(I. Sucesiones recurrentes)

P. Jara

C AP. II. S UCESIONES RECURRENTES

36

Ejemplo. 7.3.
La funcin
   


 
n
n
n
n1
n1
n n
x 7 (1 x) =

x + + (1)
x
+ (1)
xn
0
1
n1
n
n

es la funcin generatriz cuya sucesin es:


 


   
n
n
n
n n
n1
, 0, 0, . . .
, (1)
,
, . . . , (1)
n
n1
0
1
Como se verifica 1 xn = (1 x)(1 + x + + xn1 ), entonces la funcin
1 xn
x 7
= 1 + x + + xn1 ,
1x
resulta que es la funcin generatriz de la sucesin 1, 1, . . . , 1, 0, 0 ,. . . , en donde aparecen
exactamente n unos.
1
Llevado el proceso al lmite se obtiene que la funcin x 7 1x
es la funcin generatriz de
la sucesin infinita en la que los coeficientes de todos los trminos son iguales a uno, como
hemos sealado anteriormente.

Realizando modificaciones a estas funciones obtenemos nuevas funciones generatrices.


Ejemplo. 7.4.
1
Si derivamos 1x
con respecto a x se obtiene:
d
1
1
=
= 1 + 2x + 3x2 + 4x3 +
dx 1 x
(1 x)2
La sucesin asociada es: 1, 2, 3, 4, 5, . . . Al multiplicar por x se tiene
generatriz de la sucesin: 0, 1, 2, 3, . . .

x
,
(1x)2

que es la funcin

Ejemplo. 7.5.
Si derivamos esta nueva funcin, obtenemos:
d
x
1+x
=
= 1 + 22 x + 32 x2 + 42 x3 +
2
dx (1 x)
(1 x)3
La sucesin asociada es: 1, 22 , 32 , 42 , 52 , . . .
28 de enero de 2013

Curso 20122013. NOTAS DE TRABAJO, 30

S EC . 7. F UNCIONES GENERATRICES

37

Un problema domstico
Ejemplo. 7.6.
Mara tiene un montn de 12 monedas que quiere repartir entre tres huchas, sean A, B y C.
De cuntas formas lo puede hacer si en la hucha A tiene que introducir al menos 4 monedas, en las huchas B y C tiene que introducir al menos 2 monedas y en la hucha C no puede
introducir ms de 5 monedas?
S OLUCIN . Podramos hacer una tabla y considerar todos los casos de forma exhaustiva, a
modo de ejemplo tenemos:
A B C A B C A B C A B C A B C
4 3 5 5 2 5 6 2 4 7 2 3 8 2 2
4 4 4 5 3 4 6 3 3 7 3 2
4 5 3 5 4 3 6 4 2
4 6 2 5 5 2
Existe otra forma alternativa, que consiste en considerar:
para la hucha A el polinomio x4 + x5 + x6 + x7 + x8 ; sus coeficientes son todos iguales
a uno, que corresponde a la condicin de que en la hucha A tiene que haber al menos 4
monedas.
para la hucha B el polinomio x2 + x3 + x4 + x5 + x6 ; sus coeficientes son todos iguales a
uno. Como entre las huchas A y C tiene que haber al menos 6 monedas, quedan, como
mximo, 6 monedas para la hucha B, y como al menos tiene que haber 2, de ah el
polinomio que se considera.
para la hucha C el polinomio x2 + x3 + x4 + x5 ; sus coeficientes son todos iguales a uno
que corresponde a la condicin de que en la hucha C tiene que haber al menos 2 y como
mximo 5 monedas.
Al realizar el producto
(x4 + x5 + x6 + x7 + x8 )(x2 + x3 + x4 + x5 + x6 )(x2 + x3 + x4 + x5 )
= x19 + 3x18 + 6x17 + 10x16 + 14x15 + 16x14 + 16x13 + 14x12 + 10x11 + 6x10 + 3x9 + x8 .
El coeficiente de x12 es justamente el nmero que andamos buscado, en este caso 14. La
ventaja es que es ms sencillo realizar el producto de los tres polinomios para determinar
este coeficiente que hacer un tratamiento combinatorio de este problema.

Ejemplo. 7.7.
De cuntas formas es posible repartir 10 monedas entre tres personas si cada una debe
recibir al menos 2 monedas?
ARITMTICA Y COMBINATORIA
(I. Sucesiones recurrentes)

P. Jara

C AP. II. S UCESIONES RECURRENTES

38

S OLUCIN . Siguiendo el anterior mtodo de los polinomios, basta determinar el coeficiente


de x10 en el desarrollo de (x2 + x3 + x4 + x5 + x6 )3 . Como el desarrollo es:
x6 + 3x7 + 6x8 + 10x9 + 15x10 + 18x11 + 19x12 + 18x13 + 15x14 + 10x15 + 6x16 + 3x17 + x18
el valor pedido es 15.

Series de Maclaurin
Se trata de calcular el desarrollo de 1/(1 + x)n , o equivalentemente
de (1 + x)n , con n N.
n
Para esto introducimos los nmeros combinatorios r , para n, r N.
 (n)(n1)(nr+1)

Se define n
=
.
= (1)r n+r1
r
r!
r
n
Entonces la serie de Maclaurin de (1 + x) es:
(1 + x)n = 1 + (n)x +
=1+

r=1

(n)(n1) 2
x
2!

(n)(n1)(n2) 3
x
3!

(n)(n1)(nr+1) r
x
r!


r n+r1 xr
r=0 (1)
r

n r
r=0 r x ,

que es la expresin anloga al desarrollo del binomio (1 + x)n .


Por extensin se define la expansin en serie de Maclaurin de (1 + x)r para cualquier nmero
real r.

Series de Fibonacci
Consideramos la sucesin de Fibonacci:
a0 = 0, a1 = 1, an+2 = an+1 + an , si n 0.
La funcin generatriz definida por esta sucesin es:
f (x) = a0 + a1 x + a2 x2 + = 0 + x + x2 + 2x3 + 3x4 +
La definicin de la sucesin de Fibonacci nos proporciona la siguiente relacin entre las funciones generatrices:
f (x) xf (x) x2 f (x) = a0 + (a1 a0 )x + (a2 a1 a0 )x2 + + (an+2 an+1 an )xn+2 + = x.
En consecuencia se tiene la serie de Fibonacci:
f (x) =
28 de enero de 2013

x
.
1 x x2
Curso 20122013. NOTAS DE TRABAJO, 30

S EC . 7. F UNCIONES GENERATRICES

39

Los coeficientes de f (x) se puede determinar ahora atendiendo a esta fraccin.


Mtodo 1. Descomponiendo en fracciones simples:
x
B
A( x) + B( x)
A
+
=
.
=
2
1xx
x x
( x)( x)

1+ 5
2

Se tiene x2 + x 1 = (x )(x ), donde =


= 1,

+ = 1

y =

1 5
,
2

con relaciones:

5.

Adems se tiene
x = A( x) + B( x) = (A + B) (A + B)x,

de donde A + B = 1 y A( x) + B( x) = 0, entonces A =
yB=
se tiene:

= ()(x)
+ ()(x)
1xx2
1
1
= ()(1
x
()(1 x )
)

1
5(1 x )

Basta entonces desarrollar en serie la fraccin

1
(1 x )

de Fibonacci.
1
1
x
1xx2

1
5(1 x )

1
5

1
5

x
=1+ +

1 x
5(1 )
 2
x
x
+

1 x .
5(1 )

En consecuencia

para obtener los trminos de la sucesin

 2
x
+ ,


1+


1
1
x+

1
5



2
+ 1 1 + x + x +
5
 

2

1 2
1

x2 +

Finalmente observar que = y = , ver pgina 23. Por lo tanto se tiene


y obtenemos la frmula de Binet, ver pgina 23.
1
an =
5
Mtodo 2. Escribimos

1
1x

= ,

5 n
=
( n ) .
5

= 1 + x + x2 + , y aplicando a este caso se tiene:

= x(1 + x(1 + x) + x2 (1 + x)2 + x3 (1 + x)3 + )

ARITMTICA Y COMBINATORIA
(I. Sucesiones recurrentes)

1
1
n
n

=y

x
x
=
.
2
1xx
1 x(1 + x)

Consideramos el desarrollo
x
1x(1+x)

P. Jara

C AP. II. S UCESIONES RECURRENTES

40
=x

x2 + x3
x3 + 2x4 + x5
x4 + 3x5 + 3x6 + x7
x5 + 4x6 + 6x7 + 4x8 + x9

2
3
4
5
= x +x + 2x + 3x + 5x +

28 de enero de 2013

Curso 20122013. NOTAS DE TRABAJO, 30

S EC . 8. E JERCICIOS CON SOLUCIN

8.

41

Ejercicios con solucin

Sucesiones recurrentes
Ejercicio. 8.1.
Se considera la sucesin recurrente definida por
a0 = 3, a1 = 3, an+2 = 6an+1 9an , para n 0.
Calcular el trmino general.
SOLUCIN
S OLUCIN . Ejercicio (8.1.)
La ecuacin caracterstica es r 2 6r + 9 = 0, que tiene una raz = 3 demultiplicidad dos. El
trmino de una solucin general es:
an = 1 3n + 2 n3n ,
Las condiciones iniciales son: a0 = 3 y a1 = 3.
Al resolver el sistema

3 = 1 ,
,
3 = 1 3 + 2 3
llegamos a las soluciones 1 = 3, 2 = 2, y por tanto la sucesin que es solucin es de este
problema es:
an = 3 3n 2 n3n = (3 2n)3n .

Ejercicio. 8.2.
Se considera una sucesin de Fibonacci, esto es, an+2 = an+1 + an , siendo a0 y a1 arbitrarios.
Si todos los an son no nulos, demuestra que se verifica:
1

X
1
1

=
,
an1 an am am+1
aj1 aj+1

n m.

j=n

SOLUCIN
S OLUCIN . Ejercicio (8.2.)Llamamos t = m n, y hacemos induccin sobre t. Si t = 0, se
tiene:
1
an1 an

1
an+1 an1
an + an1 an1
an
1
=
=
=
=
an an+1
an1 an an+1
an1 an an+1
an1 an an+1
an1 an+1

ARITMTICA Y COMBINATORIA
(I. Sucesiones recurrentes)

P. Jara

C AP. II. S UCESIONES RECURRENTES

42

Supongamos que el resultado es cierto para t y vamos a ver qu ocurre para m n = t + 1.


1
an1 an

1
am am+1

1
1
= an1
an an+t+1 an+t+2
1
1
1
= an1
an an+t an+t+1 + an+t an+t+1
Pn+t
= j=n a 1a + an+t a1n+t+2
Pn+t+1j1 j+1
= j=n a 1a
j1 j+1
P
1
= m
.
j=n a a

1
an+t+1 an+t+2

j1 j+1

Tomar el ejemplo: a0 = 1, a1 = 1, entonces


n
an
an1 an
an1 an+1

0
1

1
1
1
2

2
2
2
3

3
3
6
10

4
5
15
24

5
8
40
65

6
13
104
168

7
21
273
442

8
34
714
1155

9
55
1870
3026

10
89
4895
7920

11
144
12816

Tomando n = 5 y m = 9 se obtiene
9

X
1
1
1
.
=
+
a4 a5
a9 a10
aj1 aj+1
j=5

1
1
1
1
1
1
1
=
+
+
+
+
+
a4 a5
a9 a10 a4 a6 a5 a7 a6 a8 a7 a9 a8 a10
1
1
1
1
1
1
1
=
+
+
+
+
+
40
4895 65 168 442 1155 3026

Ejercicio. 8.3.
REDACTAR.
SOLUCIN
S OLUCIN . Ejercicio (8.3.)
HACER.

Ejercicio. 8.4. (Espiral de Fibonacci)


Se considera la siguiente construccin de cuadrados:
28 de enero de 2013

Curso 20122013. NOTAS DE TRABAJO, 30

S EC . 8. E JERCICIOS CON SOLUCIN

43

Observa que el lado de cada uno de los cuadrados vara segn la sucesin de Fibonacci: 1, 1,
2, 3, 5, 8, 13, 21, 34, 55, . . .
La curva obtenida se conoce como la espiral de Durero y recuerda al desarrollo del Nautilus.

SOLUCIN
S OLUCIN . Ejercicio (8.4.)
HACER.
ARITMTICA Y COMBINATORIA
(I. Sucesiones recurrentes)


P. Jara

C AP. II. S UCESIONES RECURRENTES

44
Ejercicio. 8.5. (Nmeros de Perrin)
La sucesin an definida por

a0 = 3, a1 = 0, a2 = 2, an+3 = an+1 + an

fue introducida por Edouard Lucas (Amer. J. Math., vol. 1 (1978)). Aos despus fue utilizada
por R. Perrin. En 1982 Dan Shanks y Bill Adams en Math. of Computation, vol. 39 (1982) hacen
un estudio de esta sucesin a sus trminos le dan el nombre de nmeros de Perrin.
Determina la funcin generatriz de la sucesin de los nmeros de Perrin.
De forma anloga a la espiral de cuadrados de Fibonacci se tiene una espiral de tringulos
equilteros; vamos a ver cmo asociada a la misma tenemos una sucesin de nmeros de
Perrin.
SOLUCIN

S OLUCIN . Ejercicio (8.5.)


Si f (x) es la funcin generatriz, se verifica:

f (x)
2
x f (x)
x3 f (x)

a0 + a1 x+ a2 x2 + a3 x3 + a4 x4 +
a0 x2 + a1 x3 + a2 x4 +
a0 x 3 + a1 x 4 +

sumando se tiene


1x x

f (x) = a0 + a1 x + (a2 a0 )x2 = 3 x2

En consecuencia
f (x) =

1 x2
1 x2 x3

es la funcin generatriz.
Observa la siguiente construccin con tringulos equilteros:
28 de enero de 2013

Curso 20122013. NOTAS DE TRABAJO, 30

S EC . 8. E JERCICIOS CON SOLUCIN

45

Se tiene h0 + h1 = h3 , h1 + h2 = h4 , etc. Por lo tanto las alturas de los tringulos estn en la


misma progresin que los nmeros de Perrin. Esto nos debera permitir calcular la relacin
entre hn y hn+1 . Primero observa que la ecuacin caracterstica de la sucesin de Perrin es:
X 3 = X + 1.
Calcula el rea de la regin marcada con la letra A.
Determina la expresin general del trmino nsimo de la sucesin de nmeros de Perrin.


ARITMTICA Y COMBINATORIA
(I. Sucesiones recurrentes)

P. Jara

46

28 de enero de 2013

C AP. II. S UCESIONES RECURRENTES

Curso 20122013. NOTAS DE TRABAJO, 30

Captulo III
Ejercicios de repaso
9.

Ejercicios de repaso

Ejercicios de repaso
Problema. 9.1.
Dados enteros positivos a0 , a1 , . . . , a100 verificando:

a
>
a
,

1
0

a2 = 3a1 2a0 ,
...

a100 = 3a99 2a98 .


Probar que a100 > 299 .
[The problems of the All-Soviet-Union mathematical competitions 1961-1986. Problem 015]
S OLUCIN . Supongamos que a1 = a0 + h para algn entero positivo h. Entonces
a2 = 3a1 2a0 = 3(a0 + h) 2a0 = a0 + 3h.
En general se tiene entonces an = a0 + fn h, para algn entero positivo fn . La relacin de
definicin de la sucesin fn es la misma que para an , siendo f1 = 1 y f2 = 3, fn+2 = 3fn1 2fn .
La sucesin fn tiene por ecuacin caracterstica r 2 = 3r 2. Las races son 1 = 1 y 2 = 2.
Entonces una base del espacio de las sucesiones recurrentes es: {1}n y {2n1 }n . El trmino
general fn se expresa fn = 1 1 + 2 2n1 . Tenemos entonces

1 = 1 + 2 ,
3 = 1 + 2 2
La solucin es 1 = 1 y 1 = 2, y tenemos fn = 1 + 2 2n1 = 2n 1.
Se tiene entonces an = a0 + fn h = a0 + (2100 1)h > 299 .

47

C AP. III. E JERCICIOS DE REPASO

48

Problema. 9.2.
Se considera la sucesin {an }n definida por:
a1 = 1, a2 = 2, an+2 = 2an+1 + an , para n 2.
Esta sucesin comienza:
1, 2, 5, 12, 29, 70, 169, 408, 985, 2378, 5741, 13860, . . .
Se observa fcilmente que los trminos a2h son los nicos que son mltiplos de 2, que los
trminos a4h son los nicos que son mltiplos de 4, y que los trminos a8h son los nicos
que son mltiplos de 8.
Se pide probar que para cada entero positivo t se tiene que an es mltiplo de 2t si y slo si n
es mltiplo de 2t .
[Crux Mathematicorum, 1988, 225. From Erds to Kiev. Problems of Olympiad Caliber. MAA,
1996, pag. 79]
S OLUCIN . La ecuacin de recurrencia es an+2 = 2an+1 +an , luego laecuacin caracterstica

es: r 2 = 2r + 1. Las races de la ecuacin r 2 r 1 = 0 son: 1 = 1 + 2 y 2 = 1 2.


El trmino general de la sucesin an es: an = 1 1n1 + 2 2n1 . De donde resulta:
1 = 1 + 2 ,
2 = 1 1 + 2 2

Los valores son: 1 =


expresa:

2(1+ 2)
4

an =

2 n
4 1

2
4 1

y 2 =

2(1 2)
4

2
4 2 .

Y el trmino general se

2 n
4 2

n
2
(1
+
2)
4

2
4

2
4 (1

n
2)

o
(1 + 2)n (1 2)n

(n 1).

Cuando n es par, por ejemplo n = 2h tenemos:


n
2h
2h o
2
a2h = 4 (1 + 2) (1 2)
=
La expresin (1 +

2
4

n
h
ho n
h
ho
(1 + 2) + (1 2)
(1 + 2) (1 2)

2) + (1 2)h es mltiplo de 2 y no de 4.
 
h
h
Ph
h i
(1 + 2) + (1 2) = 2 i=0;par
2
  i
h
= 2(1 +
2 + )
2

28 de enero de 2013

Curso 20122013. NOTAS DE TRABAJO, 30

S EC . 9. E JERCICIOS DE REPASO

49

Como consecuencia la expresin de a2h es:


a2h = 2 X ah ,

siendo X un nmero impar

Como an es impar si y solo si n es impar, entonces tenemos el resultado haciendo induccin


sobre n.

Problema. 9.3.
Se considera una bandera, la cual se puede pintar, por franjas horizontales, de cuatro colores, sean A, B, C y D. Queremos averiguar cul es el nmero total de banderas que podemos
pintar con n franjas atendiendo a las siguientes condiciones:
(1) Cada franja est pintada de un color.
(2) Cada franja est pintada de un color y dos franjas contiguas lo estn de colores distintos.
(3) Cada franja est pintada de un color, dos franjas contiguas estn pintadas de colores
distintos y la franja superior y la franja inferior estn pintadas de distinto color.
S OLUCIN .
(1). Es claro que cada franja puede ser pintada de cualquiera de los cuatro colores, luego el
nmero total de banderas de n franjas es: 4n .
(2). La primera franja se puede pintar con cualquiera de los cuatro colores, en cambio la
segunda, tercera, etc. slo pueden pintarse de tres de los cuatro colores, luego el nmero
total de banderas es: 4 3n1 .
(3). Vamos a averiguar el nmero de banderas an que podemos pintar con n franjas, suponiendo que conocemos las banderas que se pueden pintar con menos franjas.
De cualquier bandera de n 1 franjas podemos obtener otra de n franjas sin ms que pintar
una n-sima franja de un color distinto a la primera franja y a la ltima franja; el nmero total
de banderas as obtenido es 2an1 .
De cualquier bandera de n 2 franjas podemos obtener otra de n franjas sin ms que pintar
la franja n 1 del mismo color que la primera y la franja n-sima de un color diferente; el
nmero total de banderas as obtenido es 3an2 .
Es claro que cada bandera de n franjas verificando las condiciones del enunciado est entre
las consideradas, ya que si la franja primera y penltima son distintas, se obtiene a partir de
la primera construccin, y si son iguales a partir de la segunda.
Tenemos entonces an = 2an1 + 3an2 para n 4. Los valores iniciales son: a2 = 12; las
banderas son: AB, AC, AD, BC, BD, CD y las que se obtienen cambiando el orden, en total 12.
El valor de a3 es 24, ya que tenemos las banderas ABC, ABD, ACD y BCD y todas las que se
obtienen permutando los colores, en total 46 = 24. Los valores de a0 y a1 son iguales a cero;
podemos no considerarlos en nuestra sucesin.
La ecuacin caracterstica es: r 2 = 2r + 3, sus races son:
1 = 1 y 2 = 3.
El trmino general es:
an = 1 (1)n + 2 (3)n ,
ARITMTICA Y COMBINATORIA
(I. Sucesiones recurrentes)

P. Jara

C AP. III. E JERCICIOS DE REPASO

50
que da las ecuaciones:


12 = 1 (1)2 + 2 (3)2
24 = 1 (1)3 + 2 (3)3

1 = 3
2 = 1

an = 3(1)n + 3n , para n 2.

Problema. 9.4.
Andrs tiene n euros para gastar. Puede gastarlos:
- en caramelos; una bolsa de caramelos le cuesta un euro, o
- en pasteles; hay dos tipos de pasteles y cada uno cuesta dos euros.
De cuntas formas distintas puede Andrs gastar los n euros? (Nota. Importa el orden en
que Andrs hace el gasto.)
S OLUCIN . Llamamos an al nmero de formas en que Andrs puede gastar n monedas. Veamos qu relaciones verifica an .
(1) Si el ltimo gasto de Andrs fue de un euro, slo lo puede gastar de una forma, luego tenemos an1 formas de gastar n euros.
(2) Si el ltimo gasto de Andrs fue de dos euros, puede gastarlo en uno de los dos tipos de
pasteles, luego tenemos 2an2 formas de gastar n euros.
La suma de estas posibilidades ser el nmero que andamos buscando:
an = an1 + 2an2
Tenemos adems los siguientes casos: a0 = 1, a1 = 1 y a2 = 3. As pues se trata de averiguar
el trmino general de una sucesin recurrente de orden 2.
La ecuacin caracterstica es: r 2 = r + 2, y sus races son: 1 = 1, 2 = 2.
La sucesin recurrente general es: an = 1 (1)n + 2 2n . Que verifica:
1 = 1 + 2 ,
1 = 1 (1) + 2 2,
an =

1 =
2 =

1
3
2
3

1
2
1
1
(1)n + 2n = (1)n + 2n+1 .
3
3
3
3


Problema. 9.5.
Calcular el nmero de listas de longitud n, formadas por los elementos 0, 1, y 2, en las que
no aparezcan dos ceros seguidos.
S OLUCIN . Llamamos an el nmero de listas de longitud n verificando las propiedades del
enunciado.
(1) Si a una lista, en las condiciones del enunciado, de longitud n 1 le aadimos 1 2,
entonces tenemos una lista vlida de longitud n; el nmero total es 2an1 ,
28 de enero de 2013

Curso 20122013. NOTAS DE TRABAJO, 30

S EC . 9. E JERCICIOS DE REPASO

51

(2) Si a una lista, en las condiciones del enunciado, de longitud n 2 le aadimos 10 20,
entonces tenemos una lista vlida de longitud n.
Como cualquier lista vlida de longitud n se obtiene de una de estas formas, resulta la igualdad:
an = 2an1 + 2an2 ,
Se tienen adems las siguiente condiciones iniciales: a0 = 1, a1 = 3, a2 = 9 1 = 8, ya que
las listas 11, 12, 22, 10, 01, 20, 02, 21 son vlidas, todas menos la 00.

La ecuacin caracterstica es: r 2 = 2r + 2, y las races son: 1 = 1 + 3, 2 = 1 3.

La sucesin recurrente general es: an = 1 (1 + 3)n1 + 2 (1 3)n1 . Que verifica adems:


1 = 1 + 2 ,
3 = 1 (1 +

3) + 2 (1 3),

1 =

3+2 3
6

2 =

32 3
6

La expresin del trmino general es:

n 32 3

3+2 3
(1 + 3) +
(1 3)n .
an =
6
6

Ejercicio. 9.6.
Se considera el conjunto de todas las listas de longitud n en el alfabeto {0, 1, 2}.
(1) Cuntas listas verifican que cada elemento es menor o igual que anterior?
(2) Cuntas son capicas?
(3) Cuantas contienen dos elementos consecutivos iguales?
(4) Cuntas hay que que no tienen ni dos unos ni dos doses consecutivos?
S OLUCIN .
(1) Llamamos an al nmero de listas en las que cada elemento es menor o igual que el anterior. Observa que de cada lista de este tipo de n elementos podemos formar una con
n + 1, basta aadir un elemento 0 al final. Observa que solo hay una lista con 2 al final, y
que sta est formada solo por elementos iguales a 2. Veamos que hay exactamente n + 1
listas de este tipo con n + 1 elementos que acaben en l; hacemos la demostracin por
induccin sobre n. Para n = 0 el resultado es cierto, ya que solo hay una lista con un elemento que acabe en 1. Supongamos que hay t listas de longitud t de este tipo que acaben
en uno; de cada una de ellas formamos una de t + 1 elementos simplemente aadiendo
un 1 al final; por otro lado como hay una sola lista que acaba en 2, si le agregamos un 1 al
final tenemos una lista de longitud t + 1 que acaba en 1. Es decir en total tenemos t + 1
listas de longitud t + 1 de este tipo que acaban en 1.
ARITMTICA Y COMBINATORIA
(I. Sucesiones recurrentes)

P. Jara

C AP. III. E JERCICIOS DE REPASO

52
As pues an+1 = an + n + 2.

Buscamos el polinomio caracterstica de una sucesin con trmino general del tipo n + 2;
ste es: (r 1)2 . Por otro lado la recurrencia an+1 = an tiene el polinomio caracterstico
r 1. La sucesin general determinada por el polinomio caracterstico (r 1)3 es 1 +
2 n + 3 n2 . Si esta sucesin verifica la relacin an+1 = an + n + 2 tenemos:
1 + 2 (n + 1) + 3 (n + 1)2 = 1 + 2 n + 3 n2 + n + 2
1 + 2 + 3 + (2 + 23 )n + 3 n2 = (1 + 2) + (2 + 1)n + 3 n2

1 + 2 + 3 = 1 + 2
2 + 23 = 2 + 1

3 = 3
3
1
,
2 = .
2
2
Al imponer que verifique las condiciones iniciales resulta:
3 =

a1 = 3 = 1 +

3 1
+
2 2

Esto es, 1 = 1. As pues el trmino general de la solucin es: an = 1 + 32 n + 12 n2 .


(2) Recordar que una lista es capica si leda en el orden habitual o en el orden inverso el
resultado es el mismo. Observar que una lista capica de 2n + 1 elementos se construye
a partir de una lista capica de 2n elementos agregando uno en la posicin central, y que
toda lista de 2n + 2 elementos tambin se obtiene de una de 2n elementos agregando, en
este caso, dos iguales en las posiciones centrales. Por lo tanto si llamamos an al nmero de
listas capicas de n elementos, se tiene a2n+1 = a2n+2 = 3a2n . Basta pues considerar los
elementos a2 , a4 , a6 , . . .. renombramos estos nmeros en la forma bn = a2n . La relacin
para los ai escrita para los bj es: bn+1 = 3bn para n 1, ya que no estamos consideramos
listas vacas, con la condicin inicial b1 = 3. Tenemos pues ua sucesin geomtrica de
trmino general bn = 3n , con n 1. Por lo tanto el valor que andamos buscando es:
a2n1 = a2n = bn = 3n ,

n 1.

(3) Llamamos an al nmero de listas con dos elementos consecutivos iguales; tiene sentido
esta definicin para n 2. Se verifica a2 = 3. Si llamamos bn al nmero de listas en las
que no hay elementos consecutivos iguales, entonces b2 = 6 y se verifica a2 +b2 = 3+6 =
9 = 32 , que es el nmero total de listas de dos elementos. Tal vez sea ms sencillo calcular
bn que an . Se tiene b3 = 2b2, ya que cada lista de dos elementos distintos proporciona
dos listas de tres elementos en las que no hay dos elementos consecutivos iguales. Este
argumento es vlido para cualquier longitud mayor que dos, y por tanto se tiene bn+1 =
2bn , si n 2. Como b2 = 6 = 2 3, resulta bn = 3 2n1 .
Como consecuencia an = 3n bn = 3n 3 2n1 = 3(3n1 2n1 ).
28 de enero de 2013

Curso 20122013. NOTAS DE TRABAJO, 30

S EC . 9. E JERCICIOS DE REPASO

53

(4) Llamamos an al nmero de listas de longitud n en las que no hay dos unos o dos doses
consecutivos. Es claro que a1 = 3, a2 = 7 y a3 = 17. Vamos a ver una recurrencia entre
estos nmeros. Dada una lista de n elementos verificando las condiciones, si agregamos
un 0 al final tendremos una lista de n + 1 elementos verificando las condiciones. Adems,
a cada lista de n elementos podemos agregar al final 1 o 2 segn si finaliza en 2 o en 1, de
esta forma tenemos una lista de n + 1 elementos que verifica las condiciones; nos quedan
las listas de n elementos que acaben un 0; en este caso agregamos al final 1. Observar que
de esta forma no tenemos todas las listas de n+1 elementos que verifican las condiciones;
nos faltan justamente las que finalizan en ..,02; pero de stas hay exactamente an1 , que
son los posibles inicios de n 1 elementos. Si contamos ahora todas las listas obtenidas,
resulta la igualdad
an+1 = 2an + an1 .
sta es la ecuacin de recurrencia que
verifica
la sucesin {an }n . Su polinomio caracte2 2r 1, y sus races son 1+ 2 y 1 2. La sucesin general tiene la expresin
rstico es:
r

1 (1 + 2)n + 2 (1 2)n . Al imponer las condiciones iniciales tenemos:


a1 = 3 = 1 (1 + 2) + 2 (1 2)
a2 = 7 = 1 (1 + 2)2 + 2 (1 2)2
La solucin es: 1 =

1+ 2
2

y 2 =

1 2
2 , de forma que el trmino general de la sucesin es:

n 1 2


1+ 2
1
an =
(1 + 2) +
(1 2)n =
(1 + 2)n + (1 2)n .
2
2
2

Problema. 9.7.
Dado un entero positivo n un desorden de n es una ordenacin del conjunto {1, 2, 3, . . . , n}
en la que ningn elemento ocupa su lugar original.
Calcular el nmero de desrdenes posibles de un conjunto de n elementos.
S OLUCIN . Llamamos an al nmero de desrdenes del conjunto de {1, 2, 3, . . . , n} Veamos
como podemos construir un estado de desorden de estos n elementos a partir un conjunto
menor de elementos.
(1) Dado un estado de desorden de los n 1 elementos {1, 2, 3, . . . , n 1}, conseguimos otro
estado de desorden intercambiando n con cada uno de los otros elementos. En total tenemos
(n 1)an1 estados de desorden;
(2) Otra forma de obtener un estado de desorden de n elementos es considerar una permutacin de los n 1 primeros elementos en la que un solo elemento est en su posicin original,
sea i. Ahora en el lugar de i ponemos n y ponemos i al final (derecha); se obtiene as un estado
de desorden. En total tenemos (n 1)an2 estados de desorden.
El total de estados de desorden es an = (n 1)(an1 + an2 ). Se verifica adems: a1 = 0 y
a2 = 1; como consecuencia a3 = 2(1 + 0) = 2; estos son los dos ciclos 231 y 312.
ARITMTICA Y COMBINATORIA
(I. Sucesiones recurrentes)

P. Jara

C AP. III. E JERCICIOS DE REPASO

54

Vamos a calcular el trmino general de esta sucesin. Para ello introducimos dos sucesiones
auxiliares:
bn = n 1, si n 1,
cn = cn1 + cn2 , si n 3, c1 = 1, c2 = 1
Es claro
que la sucesin
{cn }n es la sucesin de Fibonacci, cuyo trmino general es: cn =
 n
 n
1+ 5
5
55 12 5 . Tenemos entonces que el trmino general de {an }n es:
5
2

(n 1) 5
an =
5

!n
1+ 5

!n !
1 5
2


Problema. 9.8.
De cuntas formas se puede cubrir un tablero rectangular de dimensin 2 n con piezas
de dimensiones 1 2 y 2 2.
S OLUCIN . Llamamos an al nmero de formas en que podemos cubrir un tal tablero 2 n,
y observamos cmo es la ltima o ltimas piezas que ponemos.
(1) La ltima pieza es una 1 2, en esta caso slo se puede poner de una forma, supuesto
que hemos completado en tablero de dimensin 2 (n 1), luego el nmero de formas que
aporta es an1 .
(2) La ltima pieza es una pieza 2 2, podemos hacerlo de dos formas: bien una pieza 2 2, o
bien dos piezas 1 2 horizontales, de ambas formas podramos acabar el tablero, el nmero
total de formas de completarlo que aporta es 2an2 .
Como consecuencia el nmero an est determinado por las relaciones:
an = an1 + 2an2 .
Los valores iniciales son: a0 = 1, a1 = 1, a2 = 3, esto es, dos piezas 1 2 verticales, dos
piezas 1 2 horizontales y una pieza 2 2. Veamos el caso de a3 , el valor por la frmula es:
a3 = 3 + 2 1 = 5, las posibles distribuciones son:

Para calcular el trmino general primero determinamos la ecuacin caracterstica: r 2 = r + 2;


las races son: 1 = 1 y 2 = 2.
El trmino general de la sucesin es: An = 1 1n + 2 2n = 1 (1)n + 2 2n . Al imponer las
condiciones iniciales se tiene

a0 = 1 = 1 + 2
a1 = 1 = 1 + 22
La solucin es: 1 =
2 n
32 .
28 de enero de 2013

2
3

y 2 = 13 . El trmino general de la sucesin buscada es: an = 31 (1)n +

Curso 20122013. NOTAS DE TRABAJO, 30

S EC . 9. E JERCICIOS DE REPASO

55

Nota: Semejante al problema de la pgina 50.

Problema. 9.9.
Bernardo sube la escalera de su casa saltando en cada paso uno o dos escalones. Si la escalera de la casa de Bernardo tiene treinta escalones, de cuntas formas diferentes puede
subir Bernardo la escalera.
S OLUCIN . Llamamos an al nmero de formas distintas en que puede Bernardo subir una
escalera de n escalones.
(1) Si el ltimo paso de Bernardo es de un escaln, entonces Bernardo sube la escalera de
an1 formas distintas,
(2) Si el ltimo paso de Bernardo es de dos escalones, entonces Bernardo cube la escalera de
an2 formas distintas.
El nmero total de formas de subir una escalera de n escalones es: an = an1 + an2 . Los
casos iniciales son: a0 = 0, a1 = 1 y a2 = 2; esto es, estamos en presencia de la sucesin de
Fibonacci.

Problema. 9.10.
Determinar el nmero de cadenas binarias (con ceros y unos) de longitud n que contienen
dos ceros consecutivos.
S OLUCIN . Llamamos an al nmero de cadenas vlidas de longitud n. Dada una cadena
vlida de longitud n, estudiamos las siguientes posibilidades:
(1) La cadena acaba en 1; en este caso la cadena proviene de una cadena vlida de longitud
n 1; tenemos en total an1 ,
(2) La cadena acaba en 0; en este caso la cadena puede haber sido obtenida a partir de una
cadena vlida de longitud n 2 agregando bien 00 bien 10, tenemos en total 2an2 de stas;
o bien a partir de una cadena no vlida de longitud n2 a la que hemos agregado 00, tenemos
2n2 an2 de stas.
El nmero total de cadenas vlidas de longitud n es: an = an1 + an2 + 2n2 . Los valores
iniciales son: a1 = 0, a2 = 1

La ecuacin caracterstica es: r 2 = r + 1, cuyas races son: 1 = 1+2 5 , 2 = 12 5 , que fue


estudiada con la sucesin de Fibonacci
Falta ahora encontrar una solucin particular de la ecuacin de recurrencia no homognea
an = an1 + an2 + 2n2 . Supongamos que sta sea 2n2 , entonces se verifica:
2n2 = 2n3 + 2n4 + 2n2 ,
= 4.
Una solucin particular es 2n . La solucin general es
an = 2n + 1
ARITMTICA Y COMBINATORIA
(I. Sucesiones recurrentes)

!n1
1+ 5
+ 2
2

!n1
1 5
.
2
P. Jara

C AP. III. E JERCICIOS DE REPASO

56
Al introducir las condiciones iniciales tenemos:

1 = 2

0 = 2 + 1 + 2 ,
1 = 22 + 1
La solucin es:

 

1+ 5
1 5
+

,
2
2
2

2 5+5
an = 2
5
n

2 =

1+ 5
2 55
+
2
5

5+5
5 ,

2 55
5

!
1 5
.
2


Problema. 9.11.
Calcular el trmino general de la sucesin definida por
an = an1 + n2 + 3n 5, si n 1, y a1 = 2
S OLUCIN . Tenemos que al calcular las diferencias, stas son:
a
a
2 a
3 a

1 6 19 42
77
1 5 13 23 35
6 8 10 12
2 2
2

Entonces la sucesin es una progresin aritmtica de orden 3. La frmula para calcular el


trmino general es:
n  
X
n
an =
i a1 .
i
i=0

Ver la frmula (I.2) en la pgina 9.


 
 
 
 
n
n
n
n
2
an =
a1 +
a1 +
a1 +
3 a1
0
1
2
3
 
 
 
 
n
n
n
n
=
2+
(1) +
6+
2
0
1
2
3
3
2
= n +6n 310n+6 .

Problema. 9.12.
De cuntas formas se puede cubrir un tablero de tamao 32n con piezas de tamao 12?
S OLUCIN . Llamamos an al nmero de formas de cubrir el tablero 3 2n. En funcin de
tableros completos este tablero se puede terminar de llenar:
(1) con tres piezas horizontales,
28 de enero de 2013

Curso 20122013. NOTAS DE TRABAJO, 30

S EC . 9. E JERCICIOS DE REPASO

57

(2) con una pieza horizontal y dos verticales


(2.1) con las dos piezas verticales en la parte superior
(2.2) con las dos piezas verticales en la parte inferior
En resumen un tablero de tamao 3 2(n 1)4 se puede rellenar de tres formas diferentes.
Si el tablero tiene tamao 3 2(n 2), entonces se puede rellenar de dos formas, segn el
siguiente esquema

El nmero total de formas en que podemos completar el tablero es: an = 3an1 + 2an2 .

Problema. 9.13.
Se considera la sucesin recurrente definida por:
a0 = 2 = a1 y an = 2an1 2an2 .
Hallar el trmino general.
S OLUCIN . Es claro que la ecuacin caracterstica es: r 2 = 2r 2, por lo que las races son:
1 = 1 + i

2 = 1 i.

Entonces el trmino general es:


an = 1 (1 + i)n + 2 (1 i)n ,
sujeto a las condiciones iniciales a0 = 2 = a1 . Tenemos que resolver el siguiente sistema de
ecuaciones lineales:

2 = 1 + 2 ,
1 = 1
2 = 1 (1 + i) + 2 (1 i),
2 = 1
El trmino general es:
an = (1 + i)n+ (1 i)n 
i sen n
= cos n
2 +
2 + cos

n
= 2 cos 2

n
2

i sen

n
2


Problema. 9.14.
Calcular la suma de las potencias cuartas y quintas de los nmeros enteros positivos
1, 2, . . . , n.
ARITMTICA Y COMBINATORIA
(I. Sucesiones recurrentes)

P. Jara

C AP. III. E JERCICIOS DE REPASO

58

S OLUCIN . Potencias cuartas. De la frmula de la pgina 11,



k2  
X
k
k
k1
k
Cn = (n + 1) (n + 1)
Cni
k1
i

i=1

se obtiene para k = 5 las siguientes expresiones:


 
P
5
3
Cni
5Cn4 = (n + 1)5 (n + 1) i=1
i
= n5 + 5n4 + 10n3 + 10n2 + 5n + 1 n 1 (5Cn1 + 10Cn2 + 10Cn3 )
2

= n5 + 5n4 + 10n3 + 10n2 + 4n (5 n 2+n + 10 2n


4
3
= n5 + 5n2 + 5n3 n6
5
4
3
= 6n +15n 6+10n n .

+3n2 +n
6

+ 10 (n(n+1))
)
4

Potencias quintas. Utilizando la misma frmula, para k = 6 se obtiene:


 
P4
6
6
5
Cni
6Cn = (n + 1) (n + 1) i=1
i
= n6 + 6n5 + 15n4 + 20n3 + 15n2 + 6n + 1 n 1
(6Cn1 + 15Cn2 + 20C 3 n + 15Cn4 )
= 21 n2 (n + 1)2 (2n2 + 2n 1)

Problema. 9.15.
Se considera la sucesin de Fibonacci: a0 = 0, a1 = 1 y an = an1 + an2 para n 2.
(1) Probar que an+k = an ak+1 + an1 ak .
(2) Probar que ank mltiplo de ak .
(3) Se puede extender la definicin de la sucesin de Fibonacci para incluir a ndices negativos definiendo ak = ak+2 ak + 1 cuando k es un entero negativo. Probar que se verifica:
an = (1)n+1 an para cada entero positivo n.
(4) m. c. d.{an , ak } = am.c.d.{n,k} .
S OLUCIN . (1). Hacemos induccin sobre k.
Para k = 0 tenemos: an = an a1 + an1 a0 = an .
Para k = 1 tenemos: an+1 = an a2 + an1 a1 = an + an1 = an+1 .
Suponemos que el resultado es cierto para todo m k, entonces se verifica:
an+k+1 = an+k + an+k1
= (an ak+1 + an1 ak ) + (an ak + an1 ak1 )
= an (ak+1 + ak ) + an1 (ak + ak1 )
= an ak+2 + an1 ak+1 .
(2). Hacemos induccin sobre k.
28 de enero de 2013

Curso 20122013. NOTAS DE TRABAJO, 30

S EC . 9. E JERCICIOS DE REPASO

59

Para k = 0 tenemos a0 = 0 y el resultado es cierto. Suponemos que el resultado es cierto para


todo m k, entonces se verifica:
an(k+1) = ank+n = ank ak+1 + ank1 ak
Por la hiptesis ak divide a ank y a ak , luego divide a an(k+1) .
(3). Hacemos induccin sobre n. Comprobamos que si n = 1 el resultado es correcto:
a1 = a1+2 a1+1 = a1 a0 = 1 = (1)2 a1
Suponemos que el resultado es cierto para todo m n, entonces se verifica:
a(n+1) = an+1 an
= a(n1) an
= (1)n an1 (1)n+1 an
= (1)n+1 (an1 + an )
= (1)(n+1)+1 an+1
(4). Sea d = m. c. d.{n, k} y sea X = m. c. d.{an , ak }. Como n = dn0 y k = dk0 para ciertos
enteros positivos o nulos n0 y k0 , resulta
an = adn0 es un mltiplo de ad .
El mismo argumento vale para ak , luego ad es un divisor de an y de ak , por tanto es un divisor
de X .
Por otro lado, la identidad de Bezout expresa d en funcin de n y k, en la forma d = n + k,
entonces resulta
ad = an+k = an ak+1 + an1 ak .
Como an divide a an , entonces X divide a an , y como ak divide a ak , entonces X divide a
ak . En consecuencia X divide a ad , y tenemos el resultado.

Problema. 9.16.
P
Se considera la sucesin de Fibonacci {an }n , y se define una nueva sucesin Fn = ni=0 ai .
Encontrar una expresin recurrente para esta nueva sucesin y determinar su trmino general.
S OLUCIN . Se tiene Fn = Fn1 + an , entonces
Fn Fn1 = an = an1 + an2 = (Fn1 Fn2 ) + (Fn2 Fn3 ,
de donde resulta que Fn = 2Fn1 Fn3 . Los valores iniciales son: F0 = 0, F1 = 1 y F2 = 2, y
la ecuacin caracterstica es:
r 3 = 2r 2 1
Sus races son: 1 = 1, 2 =

1+ 5
2

ARITMTICA Y COMBINATORIA
(I. Sucesiones recurrentes)

y 3 =

1 5
2 .

P. Jara

C AP. III. E JERCICIOS DE REPASO

60
El trmino general de Fn es:
n

Fn = 1 1 + 2

!n
1+ 5
+ 3
2

!n
1 5
2

que el imponer las condiciones iniciales nos da el sistema de ecuaciones lineales:

0 = 1 + 2 +
3

1+ 5
1 5
1 = 1 + 2 2 + 3 2
 2
 2

2 = 1 + 2 1+2 5 + 3 12 5
cuya solucin es: 1 = 1, 2 =

1+5
5 5

y 3 =

1+ 5

Fn = 1 +
5 5

15
.
5+ 5

De forma que el trmino general es:

!n

1+ 5
1 5

+
2
5+ 5

!n
1 5
2

que toma los valores: 0, 1, 2, 4, 7, 12, 20, 33, 54, 88, 143, 232, 376, 609, 986, 1596, 2583, . . .

Problema. 9.17.
Se considera la sucesin de Fibonacci: a0 = 0, a1 = 1 y an = an1 + an2 para n 2. Si
definimos Pn = a2n y definimos In = a2n+1 , encontrar una relacin de recurrencia lineal
para Pn e In .
S OLUCIN . Tenemos:
Pn = a2n = a2n1 + a2n2 = a2n1 + Pn1 .
De aqu se obtiene s2n1 = Pn Pn1 , y por tanto podemos proceder como sigue:
Pn = a2n1 + Pn1
= a2n2 + a2n3 + Pn1
= Pn1 + a2n3 + Pn1
= Pn1 + a2(n1)1 + Pn1
= Pn1 + (Pn1 Pn2 ) + Pn1
= 3Pn1 Pn2
Las condiciones iniciales son: P0 = 0, P1 = 1.
Para el clculo del trmino general de
P primero calculamos
la ecuacin caracterstica: r 2 =

n
3r 1, cuyas races son: : 1 = 3+2 5 y 2 = 32 5 , entonces el trmino general es: Pn =
 n
 n
1 3+2 5 + 2 32 5 ; al imponer las condiciones iniciales se tiene el sistema de ecuaciones lineales:
)
0 = 1 
+ 2 
 
1 = 1 3+2 5 + 2 32 5
28 de enero de 2013

Curso 20122013. NOTAS DE TRABAJO, 30

S EC . 9. E JERCICIOS DE REPASO
que tiene solucin 1 =

1
5

61

y 2 = 1 . El trmino general es:


5

1
Pn =
5

!n
3+ 5
1

2
5

!n
3 5
.
2

Para In procedemos en la misma forma y obtenemos In = 3In1 In2 con las condiciones
iniciales I0 = 1, I1 = 2.
Para el clculo del trmino general de In procedemos en la misma forma. En este caso tenemos que resolver el sistema de ecuaciones lineales:
1 = 1 
+ 2 
 
2 = 1 3+2 5 + 2 32 5
que tiene solucin 1 =

5+ 5
10

y 2 =

5+ 5
In =
10

5 5
10 .

El trmino general es:

!n

3+ 5
5 5
+
2
10

!n
3 5
.
2


Problema. 9.18. (Los nmeros de Lucas)


Consideramos la sucesin recurrente definida por: l0 = 2. l1 = 1 y la frmula ln = ln1 + ln2
para n 2. Los nmeros ln se llaman los nmeros de Lucas. Determinar el trmino general
de la sucesin de los nmeros de Lucas, y probar que si {fn }n es la sucesin de Fibonacci,
entonces se verifica:
ln = fn1 + fn+1 ;
ln = fn+2 fn2 ;
fh ln = fn+h + (1)h+1 fnh .
Probar la siguiente relacin entre nmeros de Fibonacci y nmeros de Lucas f2n = fn ln .

S OLUCIN . La ecuacin caracterstica es r 2 = r + 1, y sus races son: 1 = 1+2 5 y 2 = 12 5 .


 n
 n
El trmino general es: ln = 1 1+2 5 + 2 12 5 . Al imponer las condiciones iniciales se
obtiene el siguiente sistema de ecuaciones lineales:
2 = 1 
+ 2 
 
1 = 1 1+2 5 + 2 12 5

La solucin es: 1 = 1 y 2 = 1. De forma que el trmino general es: ln =


ARITMTICA Y COMBINATORIA
(I. Sucesiones recurrentes)

n
1+ 5
2

n
1 5
2

P. Jara

C AP. III. E JERCICIOS DE REPASO

62

Podemos hacer uso dela expresin


del

 trmino general de la sucesin de Fibonacci: f0 = 0,

f1 = 1 y fn =

5
2

n
1+ 5
2

fn1 + fn+1 =
=
=

5
2

n
1 5
,
2

entonces se tiene:

n1
 n1
 n+1
 n+1
1+ 5
5 1 5
5 1+ 5
5 1 5

2
5
2
5
2
5
2
 n1 
 2   n1 
 2 
1+ 5
1 + 1+2 5
55 12 5
1 + 12 5
2
 n1    n1  
5 5+ 5
5 5 5
1+ 5
12 5
2
5
2
 2 n 5 
n
1+ 5
1 5
+
2
2

5
5

5
5

=
= ln

La misma relacin para fn+2 se puede probar ahora como sigue:


ln = fn+1 + fn1
= fn+1 + fn fn + fn1
= fn+2 fn2 ,
ya que de la relacin fn = fn1 + fn2 se obtiene fn2 = fn fn1 .
Podemos extender estas relaciones a nmeros de Fibonacci y nmeros de Lucas con ndices
no necesariamente positivos. Al estudiar diferentes casos, observamos la relacin:
fh ln = fn+h + (1)h+1 fnh .
Cuya demostracin es sencilla si hacemos induccin sobre h:
fh+1 ln = fh ln + fh1 ln
= fn+h + (1)h+1 fnh + fn+h1 + (1)h fnh+1
= (fn+h + fn+h1 ) + (1)h+2 (fnh + fnh+1 )
= fn+h+1 + (1)h+2 fnh1
La relacin final del enunciado es evidente, basta tomar h = n.

Problema. 9.19.
Consideramos {fn }n la sucesin de Fibonacci, probar que se verifican las siguientes identidades:
(1) Identidad de Cassini. fn2 fn+1 fn1 = (1)n1 f12 .
(2) Identidad de Catalan. fn2 fn+r fnr = (1)nr fr2 , r 1.
(3) Identidad de dOcagne. fm fn+1 fn fm+1 = (1)n fmn .
(4) Identidad de GelinCesro. fn4 fn2 fn1 fn+1 fn+2 = 1.
28 de enero de 2013

Curso 20122013. NOTAS DE TRABAJO, 30

S EC . 9. E JERCICIOS DE REPASO

63

S OLUCIN .

Problema. 9.20.
Considerar las sumas parciales de la sucesin de los nmeros de Lucas.
S OLUCIN . .

Problema. 9.21.
Determinar un polinomio p(x) de grado cuatro, sin trmino independiente, que verifique
p(x) p(x 1) = x3 . Demuestra que la suma de los cubos de los n primeros nmeros
enteros positivos es p(n).
XXVIII Olimpada Matemtica Dist. Univ. Valencia. Problema nm. 5
S OLUCIN . Tenemos p(0) = 0, y de p(x) p(x 1) = x3 se deduce que p(0) p(1) = 0,
luego p(1) = 0. Entonces se tiene la factorizacin p(x) = x(x + 1)(ax2 + bx + c).
Vamos a dar nuevos valores:
n = 1 p(1) p(0) = 1 p(1) = 1 = 1 2(a + b + c)
n = 2 p(2) p(1) = 23 p(2) = 23 1 = 7 = (2 3(4a + 2b + c)
n = 3 p(3) p(2) = 33 p(3) = 33 7 = 20 = 3 4(9a + 3b + c)
Tenemos que resolver el sistema de ecuaciones lineales:
a + b + c = 21
4a + 2b + c =
9a + 3b + c =
La solucin es: a =

1
4

7
6
20
12

5
3

= b, c = 0. El polinomio es: p(x) = x(x + 1)( 14 x2 + 14 x) =

x(x+1)
2

2

Por induccin sobre n se prueba que la suma de los n primeros enteros positivos es p(n).

Problema. 9.22.
Se considera la sucesin de los nmeros triangulares, esto es, los nmeros que representan los puntos que formas los tringulos siguientes:
i
i
i

i
i

i
i

Esta sucesin tiene los siguientes elementos: 1, 3, 6, 10, 15, . . .


Se trata de encontrar la frmula del trmino general de esta sucesin.
ARITMTICA Y COMBINATORIA
(I. Sucesiones recurrentes)

P. Jara

C AP. III. E JERCICIOS DE REPASO

64

S OLUCIN . Al hacer las diferencias sucesivas de los elementos de esta sucesin tenemos:
an
an
2 an

1
2
1

3
3
1

6
4
1

10
5

15

Para construir el trmino general procedemos al igual que en el problema (4.2.).


an1 = a0 + Sn2 2 an1
=2+n11
=n+1
an1 = a0 + Sn1 an1
= 1 + (2+(n+1))n
(n + 1)
2
n(n+1)
= 2

Problema. 9.23.
Se considera la sucesin de los nmeros tetradricos, esto es, los nmeros que representan los puntos que formas los tetraedros siguientes:
i
@ i

Ai
 A

i

Esta sucesin tiene los siguientes elementos: 1, 4, 10, 20, 35, . . .


Se trata de encontrar la frmula del trmino general de esta sucesin.
S OLUCIN . Al hacer las diferencias sucesivas de los elementos de esta sucesin tenemos:
an
an
2 an
3 an

1
3
3
1

4
6
4
1

10
10
5

20
15

35

Para construir el trmino general procedemos al igual que en el problema (4.2.).


2 an1 = 2 a0 + Sn2 3 an1
=3+n11
=n+2
an1 = a0 + Sn2 2 an1
(n + 2)
= 3 + (3+(n+2))n
2
2
= n +3n+2
2
28 de enero de 2013

Curso 20122013. NOTAS DE TRABAJO, 30

S EC . 9. E JERCICIOS DE REPASO

65

an1 = a0 + Sn1 an1


2
= 1 + 12 Cn2 + 32 Cn1 + Cn0 n +3n+2
2
3
2
+n
3 n(n+1)
n2 +3n+2
= 12 2n +3n
+
+
n

6
2
2
2
3
2
+4n
= 2n +6n
12
n(n+1)(n+2)
=
6
Nota. Los nmeros tetradricos son la cuarta fila oblicua del tringulo de Tartaglia:
1
1
1
1
1
1

3
4

1
2

1
3

6
10

1
4

10

1
5

1 6 15 20 15 6 1
1 7 21 35 35 21 7 1
La
 tercera
  son los nmeros triangulares, por lo tanto las frmulas para el trmino general son
n
n
y
respectivamente.

3
2
Problema. 9.24.
Determinar el trmino general de la sucesin {an }n verificando la ecuacin de recurrencia
an = 2an1 an2 + k,
verificando las condiciones iniciales a0 = 0 y a1 = h.
S OLUCIN . Calculamos una solucin particular. Sea 1 + 2 n + 3 n2 , entonces se tiene:
1 + 2 n + 3 n2 2(1 + 2 (n 1) + 3 (n 1)2 ) + (1 + 2 (n 2) + 3 (n 2)2 ) k
= 23 k,
entonces si tomamos 3 = k2 , 1 = 0 = 2 , tenemos la solucin particular k2 n2 .
La parte homognea tiene por ecuacin caracterstica r 2 = 2r 1; la solucin es 1 = 1 = 2 ,
entonces la solucin general de la parte homognea es: 4 + 5 n, y la solucin general de la
ecuacin de recurrencia no homognea es: k2 n2 + 4 + 5 n.
Al hacer intervenir ahora a las condiciones iniciales, se obtiene el sistema de ecuaciones lineales

4 = 0
k
2 + 4 + 5 = h
cuya solucin es: 4 = 0 y 5 = h k2 . La solucin general es:
an =

k 2
k
k
n + (h )n = (n2 n) + hn.
2
2
2

ARITMTICA Y COMBINATORIA
(I. Sucesiones recurrentes)

P. Jara

C AP. III. E JERCICIOS DE REPASO

66


Problema. 9.25.
Sean a y b enteros positivos. Hallar el trmino general de la ecuacin recurrente
an = an1 + a +

b2 + 4aan1 , si n 1,

a0 = 0.

[Generalizacin de un problema propuesto en IMO 1981]


S OLUCIN . Vamos a tratar de eliminar los radicales.
Tenemos
q
an an1 = a + b2 + 4aan1 ,

(III.1)

y se verifica
p
2 + 4a b2 + 4aa
b2 + 4aan = (b2 + 4aa
)
+
4a
n1
n1
p
2
2
= (2a + b + 4aan1 ) .
p
p
Luego b2 + 4aan = 2a + b2 + 4aan1 , lo que da lugar a la siguiente relacin:
p
an1 = an2 + a + b2 +
p4aan2
= an2 + a 2a + b2 + 4aan1 .
De aqu se obtiene:
an1 an2

q
= a + b2 + 4aan1 .

(III.2)

Al realizar (III.1)(III.2) se tiene:


an 2an1 + an2 = 2a.
Adems:
a1 = a0 + a +

b2 + 4aa0 = a + b2 = a + b,

luego tenemos una sucesin verificando:


an = 2an1 an2 + 2a, si n 2,

a0 = 0, a1 = a + b,

entonces aplicando el problema (9.24.) el trmino general es:


2a 2
2a
an =
n + (a + b
)n = an2 + bn = (an + b)n,
a
a

n 2.


Problema. 9.26.
Demostrar que en la sucesin definida por los siguientes datos:
a0 = 0 y
p
an+1 = (an + 1)k + (k + 1)an + 2 k(k + 1)an (an + 1), n 0, k un entero positivo.
todos sus trminos an , n 1, son enteros positivos.
[No elegido en la IMO 1983]
28 de enero de 2013

Curso 20122013. NOTAS DE TRABAJO, 30

S EC . 9. E JERCICIOS DE REPASO

67

S OLUCIN . Como la sucesin es estrictamente creciente, y como a0 = 0, entonces todos los


trminos an , n 1 son positivos. Vamos a ver que son enteros.
Al tratar de eliminar los radicales se tiene:
p
an+1 (an + 1)k (k + 1)an = 2 k(k + 1)an (an + 1)
Elevando al cuadrado y agrupando en el primer miembro se obtiene:
an2 2an (an+1 + k + 2an+1 k) + (an+1 k)2 = 0.
Resolvemos esta ecuacin en an y se obtiene:

2(an+1 +k+2an+1 k) 4(an+1 +k+2an+1 k)2 4(an+1 k)2


an =
2
p
= an+1 (2k + 1) + k 2 an+1 (an+1 + 1)(k + 1)k
Ahora, como la sucesin es estrictamente creciente, en el radical debemos tomar el signo
menos":
p
an = an+1 (2k + 1) + k 2 an+1 (an+1 + 1)(k + 1)k
entonces, utilizando la definicin para an+2 se tiene:
p
an = an+1 (2k + 1) + k 2 an+1 (an+1 + 1)(k + 1)k
= an+1 (2k + 1) + k (an+2 (an+1 + 1)k (k + 1)an+1 )
= an+1 (3k + 2) + k + (an+1 + 1)k an+2
= an+1 (4k + 2) + 2k an+2 .
an+2 = an+1 (4k + 2) an + 2k,
Tenemos entonces que todos los valores son enteros, ya que
p
a1 = (a0 + 1)k + (k + 1)a0 + 2 k(k + 1)a0 (a0 + 1) = k
es tambin entero.

Problema. 9.27.

Sean m, n y r enteros positivos verificando 1 + m + n 3 = (2 + 3)2r1 . Demostrar que


entonces m es un cuadrado perfecto.
[II Olimpada Iberoamericana Uruguay 1987 (Felipe Fritz Braga, mencin especial del jurado)]

S OLUCIN . Al conjugar por 3 tenemos las relaciones:

1 + m + n3 = (2 + 3)2r1
1 + m n 3 = (2 3)2r1 ,

que al sumar dan: 2 + 2m = (2 + 3)2r1 + (2 3)2r1 .


ARITMTICA Y COMBINATORIA
(I. Sucesiones recurrentes)

P. Jara

C AP. III. E JERCICIOS DE REPASO

68

Si consideramos

=
2
+
3
y

=
2

3,
stas
son
races
del
polinomio
(x

(2
+
3))(x
1
2

(2 3)) = x2 4x+1. Podemos considerar la sucesin recurrente con ecuacin caracterstica


x2 4x + 1, cuyo trmino general es:

ar = 1 (2 + 3)r + 2 (2 3)r .
Si tomamos las condiciones iniciales a0 = 1 y a1 = 1, entonces tenemos que resolver el
sistema de ecuaciones lineales

1 = 1 + 2

1 = 1 (2 + 3) + 2 (2 3)
La solucin es: 1 =

31
2

y 2 =

31
2 .

El trmino general es:

ar =

31
(2 + 3)r
2

2

3+1
(2 3)r
2

Se verifica ahora:
(ar )2 =
=

31
2

(2 +

2r
2 3
2 (2 + 3)

= 12 (2 +

3)2r +
+

3+1
2

2

(2

2r
2+ 3
2 (2 3)

   
2r
31
3+1
3) 2
2
2

2r1 1

3)
+ 2 (2 3)2r1 1

=m

Problema. 9.28.
Sea m un entero positivo y definamos la sucesin siguiente:
a1 = 2(m + 1),

1
an+1 = 4m
(man 1 + 1 + 2man ), si n 1.
Obtener una expresin explcita del trmino an en funcin de n y m.
[Gazeta Matematica. 1988. Rumania]
S OLUCIN . Observamos el valor de a2 .
a2 =
=
=
=
28 de enero de 2013

1
4m (man 1 + 1 + 2ma
p n)
1
1 + 2m2(m + 1))
4m (m2(m + 1) 1 +
1
2
4m (2m 2m 1 + 2m + 1)
m+2
2 .
Curso 20122013. NOTAS DE TRABAJO, 30

S EC . 9. E JERCICIOS DE REPASO

69

Tenemos que a1 > a2 , ya que si a1 < a2 , entonces


2(m + 1) < m+2
2 ;
4m + 4 < m + 2;
3m + 2 < 0;
m < 32 ,
lo que es una contradiccin. Adems a1 , a2 > 0.
Veamos ahora la diferencia an+1 an .
an+1 an =
=

1
+ 2man ) an
4m (man 1 + 1
1
4m (3man 1 + 1 + 2man ).

Si 3man 1 + 1 + 2man > 0, entonces 1 + 2man > 3man + 1. Vamos a suponer que
an > 0, entonces podemos tomar cuadrados manteniendo la desigualdad:
1 + 2man > 9m2 an2 + 1 + 6man
0 > 9m2 an2 + 4man ,
lo cual es imposible. Tenemos pues que si an > 0, entonces an > an+1 .
Vamos a ver que tambin an+1 > 0 en este caso. Si an+1 0, entonces

ma

1
+
1 + 2man 0
n

1 +2man 1 man
0 1 + 2man 1 man
( man 1)
2
2
1 + 2man 1 + m an 2man
4man m2 an2
4m m2 an
4 man
Llegamos as a una contradiccin: man 1 < 4 man . Como consecuencia tenemos una
sucesin decreciente de nmeros positivos (no necesariamente enteros).
Vamos ahora a eliminar radicales.

1
an+1 = 4m
(man 1 + 1 + 2man )
4man+1 = man 1 + 1 + 2man
4man+1 man + 1 = 1 + 2man
Al tomar cuadrados tenemos:
2
16m2 an+1
+ m2 an2 + 1 + 8man+1 8m2 an an+1 2man = 1 + 2man
2
man 4(2man+1 + 1)an + 8an+1 (2man+1 + 1) = 0
Calculando las races se tiene:
an =

4(2man+1 +1)

42 (2man+1 +1)2 4m8an+1 (2man+1 +1)


2m

4(2man+1 +1)4
2m

2(2man+1 +1)2
m

ARITMTICA Y COMBINATORIA
(I. Sucesiones recurrentes)

2man+1 +1

2man+1 +1

.
P. Jara

C AP. III. E JERCICIOS DE REPASO

70

p
man = 2(2man+1 + 1) 2 2man+1 + 1.

Para averiguar el signo, supongamos que man = 2(2man+1 + 1) + 2 2man+1 + 1, entonces al


introducir este valor en la relacin original se tiene

4man+1 = 2(2man+1 + 1) + 2 2man+1 + 1 1 + 2man + 1

0 = 1 + 2 2man+1 + 1 + 2man + 1,
lo que es imposible, pues todos los trminos del miembro de la derecha son positivos.

Se tiene entonces man = 2(2man+1 + 1) 2 2man+1 + 1.


Podemos considerar entonces las dos relaciones siguientes:

4man+2 = man+1 1 + 1 + 2man+1

man = 2(2man+1 + 1) 2 2man+1 + 1


al sumar el doble de la primera y la segunda se obtiene:
8man+2 + man = 2man+1 2 + 4man+1 + 2,
de donde resulta:
8an+2 = 6an+1 an .
Resolvemos esta recurrencia con los valores iniciales a1 = 2(m + 1) y a2 = m+2
2 . La ecuacin
1
1
2
caracterstica 8r = 6r 1 tiene races 1 = 2 y 2 = 4 , entonces tenemos que resolver el
sistema de ecuaciones lineales
)
1
1

=
2(m
+
1)
2 1
4 2 
2
1 2
1 + 41 2 = m+2
2
2
Se obtiene 1 = 4 y 2 = 8m, luego el trmino general es:
n
n
an = 4 12 + 8m 41
n
2n
= 4 21 + 8m 21
para n 1.

Problema. 9.29.
Sea k un entero positivo, se define una sucesin mediante las reglas:
x1 = k,
p
xn+1 = kxn + (k2 1)(xn2 1), si n 1.
Demostrar que todos los xn son enteros positivos.
[Competicin Krschak. Hungra. 1988]
28 de enero de 2013

Curso 20122013. NOTAS DE TRABAJO, 30

S EC . 9. E JERCICIOS DE REPASO

71

S OLUCIN . Vamos a eliminar radicales; se tiene:


p
xn1 kxn = (k2 1)(xn2 1)
2
xn+1
+ k2 xn2 2kxn xn+1 = (k2 1)(xn2 1)
2
+ k2 1) = 0
xn2 xn (2kxn+1 ) + (xn+1

Al resolver esta ecuacin se tiene:


xn =

q
2
2
2kxn+1 4k2 xn+1
4(xn+1
+k2 1)
2

q
2
= kxn+1 (k2 1)(xn+1
1)
q
2
Supongamos que xn = kxn+1 + (k2 1)(xn+1
1), como
xn+1 = kxn +

p
(k2 1)(xn2 1)

r
q
q
2
2
2
2
= k(kxn+1 + (k 1)(xn+1 1)) + (k 1)((kxn+1 + (k2 1)(xn+1
1))2 1),
entonces
(1 k2 )xn+1

r
q
q
2
2
2
2
1))2 1)
= k (k 1)(xn+1 1) + (k 1)((kxn+1 + (k2 1)(xn+1

lo que implica que (1 k2 )xn+1 0, pero (1 k2 ) 0, luego xn+1 < 0, pero como x1 > 0 la la
sucesin es creciente, llegamos a una contradiccin.
q
2
Tenemos entonces xn = kxn+1 (k2 1)(xn+1
1). De aqu obtenemos:

xn+2

2
(k2 1)(xn+1
1)
q
2
= kxn+1 + (k2 1)(xn+1
1.

xn = kxn+1

Al sumar se tiene
xn+2 + xn = 2kxn+1 ,
2
de dondexn+2 = 2kxn+1 x
n . La ecuacin caracterstica es: r = 2kr 1, cuyas races son:
1 = k + k2 1 y 2 = k k2 1. El trmino general es:
p
p
xn = 1 (k + k2 1)n1 + 2 (k k2 1)n1 ;

al imponer las condiciones iniciales x1 = k y


q
q
x2 = kx1 + (k2 1)(x12 1) = k2 + (k2 1)2 = 2k2 1,
ARITMTICA Y COMBINATORIA
(I. Sucesiones recurrentes)

P. Jara

C AP. III. E JERCICIOS DE REPASO

72
tenemos que resolver el sistema de ecuaciones lineales

1 +
2 = k

(k + k2 1)1 + (k k2 1)2 = 2k2 1

k+ k2 1
k k2 1
y

=
. Tenemos que el trmino general
2
2
2

2
2 1)n1 + k k2 1 (k k2 1)n1
xn = k+ 2k 1
(k
+
k
2

1
n
n
2
= 21 (k + k2
 1) + 2 (k k 1)

P
i
n
= 21 i,par
2kni k2 1
i

La solucin es: 1 =

es:

 
n ni 2
(k 1)i/2 ,
i,par i k

que es un entero positivo.

Problema. 9.30.
Sean m un entero positivo y p un entero arbitrario. Se define la sucesin {xn }n mediante
x1 = 0,
p
xn+1 = mxn + (m2 1)xn2 + p2 , si n 1.
Demostrar que todos los trminos de la sucesin son enteros.
[Gazeta Matematica. Rumania. 1990]
S OLUCIN . Vamos a eliminar radicales:
p
xn+1 mxn = (m2 1)xn2 + p2
2
xn+1
+ m2 xn2 2mxn xn+1 = (m2 1)xn2 + p2
2
p2 = 0
xn2 2mxn+1 xn + xn+1

Al resolver esta ecuacin se tiene:


xn =
=

q
2
2
2mxn+1 4m2 xn+1
4xn+1
+4p2
2mxn+1 2

q 2
2
xn+1
(m2 1)+p2

q2
2 (m2 1) + p2 .
= mxn+1 xn+1
La sucesin {xn }n es creciente, ya que x1 = 0 y el resto se deduce de la ecuacin de recurrencia.
Vamos a determinar qu signo es el correcto. Si suponemos que xn = mxn+1 +
q
2 (m2 1) + p2 , se verifica:
xn+1
p
xn+1 = mxn + (m2 1)xn2 + p2
r
q
q
2
2 (m2 1) + p2 )2 + p2
= m(mxn+1 + xn+1 (m2 1) + p2 ) + (m2 1)(mxn+1 + xn+1
28 de enero de 2013

Curso 20122013. NOTAS DE TRABAJO, 30

S EC . 9. E JERCICIOS DE REPASO

73

De aqu se tiene
2

(1 m )xn+1

r
q
q
2
2 (m2 1) + p2 )2 + p2
= m( xn+1 (m2 1) + p2 ) + (m2 1)(mxn+1 + xn+1

Tenemos que (1 m2 )xn+1 > 0, como (1 m2 ) 0, se tiene xn+1 < 0, lo que es una contradiccin.
q
2 (m2 1) + p2 . Con sta y la relacin inicial se tiene:
Obtenemos pues xn = mxn+1 xn+1
q
2
xn+2 = mxn+1 + (m2 1)xn+1
+ p2
q
2
+ p2 ,
xn = mxn+1 (m2 1)xn+1
sumando se tiene:
xn+2 + xn = 2mxn+1 .
2
Tenemos la
ecuacin caracterstica r = 2mr 1, cuyas races son: 1 = m +
2 = m m2 1. Las condiciones iniciales son: x1 = 0 y
q
x2 = mx1 + (m2 1)x12 + p2 = p.

m2 1 y

Tenemos entonces el sistema de ecuaciones lineales


1 +
2 =0

(m + m2 1)1 + (m m2 1)2 = p

y 2 = p2 . Entonces el trmino general es:


2 m 1

xn = (m + m2 1)n1 1 + (m m2 1)n1 2

Las races son: 1 =

p
2 m2 1

= (m +
=

m2 1)n1

p
2 m2 1

=p

p
2 m2 1

+ (m

m2 1)n1

p
2 m2 1




n1
ni1 ( m2 1)i
2
m
i,impar
i




n1
ni1 ( m2 1) i1
2
m
i,impar
i

que es un entero.

Problema. 9.31.
Sea p(x) un polinomio de grado n que verifica
p(j) = 2j1 , para j = 1, 2, 3, . . . , n, n + 1.
Determinar p(n + 2).
[Olimpada de Israel 1988].
ARITMTICA Y COMBINATORIA
(I. Sucesiones recurrentes)

P. Jara

C AP. III. E JERCICIOS DE REPASO

74

S OLUCIN . Consideramos la sucesin p(1), p(2), . . . , p(n + 1); como p(x) es un polinomio de
grado n, entonces esta sucesin es una progresin aritmtica de orden n, a la cual podemos
calcular el trmino general. Veamos cuales son las diferencias sucesivas. Tenemos p(i) =
p(i + 1) p(i) = 2i 2i1 = 2i1 = p(i). En general se tiene t p(i) = p(i), siendo siempre
i < n + 1 t. Cuando t = n slo tenemos una diferencia, y sta es n p(1) = 1.
Podemos calcular el trmino general aplicando la teora que hemos desarrollado o bien la
teora ms general de interpolacin de Lagrange.
Otra forma directa, para este caso, es la siguiente:
p(n + 2) = p(n + 1) + p(n + 1) = p(n + 1) + p(n + 1)
= p(n + 1) + p(n) + p(n) = p(n + 1) + p(n) + p(n)
=
= p(n + 1) + p(n) + p(n 1) + + p(2) + p(2)
= p(n + 1) + p(n) + p(n 1) + + p(2) + p(1) + p(1)
= 2n + 2n1 + 2n2 + + 2 + 1 + 1
= 2n+1 1 + 1 = 2n+1

Problema. 9.32.
Sean {xn }n y {yn }n sucesiones de enteros positivos definidas por:
x0 = 1 = x1 , xn+1 = xn + 2xn1 , si n 1.
y0 = 1, y1 = 7, yn+1 = 2yn + 3yn1 , si n 1.
Demostrar que x0 = x1 = y0 = 1 es el nico trmino que tienen en comn ambas sucesiones.
[Olimpada EEUU. 1973]
S OLUCIN . Calculamos los trminos generales de las sucesiones {xn }n y {yn }n :
xn = 23 2n + 13 (1)n ,
yn = 2 3n (1)n .
Supongamos que existen n y m tales que xn = ym , entonces se tiene:
2 n
32

+ 13 (1)n = 2 3m (1)m

2n+1 + (1)n = 2 3m+1 3(1)m


2n+1 2 3m+1 = ((1)n + 3(1)m)
2n 3m+1 = 21 ((1)n + 3(1)m )
28 de enero de 2013

Curso 20122013. NOTAS DE TRABAJO, 30

S EC . 9. E JERCICIOS DE REPASO

75

Los valores del miembro de la derecha son:


n par
n par
n impar
n impar

m par
m impar
m par
m impar

12 (1 + 3) = 2
12 (1 3) = 1
12 (1 + 3) = 1
12 (1 + 3) = 2

Si 2n 3m+1 = 2 y n 6= 0, entonces 2 | 3m+1 , lo que es una contradiccin; si n = 0, entonces


1 3m+1 = 2, y resulta 3m+1 = 3, esto es, m + 1 = 1, o equivalentemente m = 0. (ste da
el caso x0 = 1 = y0 )
Si 2n 3m+1 = 2, como n 6= 0, ya que n es impar, resulta 2 | 3m+1 , lo que es una contradiccin.
Si 2n 3m+1 = 1, tenemos que n es par; sea n = 2t, y m es impar, sea m + 1 = 4h; tenemos:
22t = 3m+1 + 1 = 32h + 1 = 9h + 1.
Tomando clases de resto mdulo 4 tenemos 0 1h + 1 (m
od 4), lo que es una contradiccin.
n
m+1
Si 2 3
= 1, tenemos que n es impar, sea n = 2t + 1 y m es par, sea m = 2h, entonces:
22t+1 = 3m+1 1 = 32h+1 . 1
Si t = 0, entonces 2 = 32h+1 1, tomando h = 0 tenemos una solucin. Si t 6= 0, entonces
8 | 22t+1 y tomando clases de resto mdulo 8 resulta 0 32h+1 1 9 3 1 3 1 2
(m
od 8), lo que es una contradiccin. (ste da el caso x1 = 1 = y0 )

Problema. 9.33.
Tenemos un plano con dos puntos pintados de rojo y n puntos pintados de negro. Podemos
pintar segmentos solo entre puntos de distinto color.
(1) Calcular el nmero mnimo de segmentos que son necesarios para que todos los puntos
estn conectados.
(2) De cuntas formas diferentes se puede dibujar este nmero mnimo de segmentos para
que todos los puntos estn conectados.
S OLUCIN . (1). Para que todos los puntos estn conectados, de cada punto negro debe salir
un segmento, y como el otro extremos ha de ir a un punto rojo, tenemos al menos n segmentos. Pero en esta situacin los dos puntos rojos no estn conectados, luego necesitamos un
segmento ms, de forma que una punto negro est unido a los dos puntos rojos. De forma
que el nmero mnimo de segmentos es n + 1.
(2). La forma es tener el nmero mnimo de segmentos para el caso de n puntos negros se
puede construir a partir de una situacin con n 1 puntos. Por ello si an es el nmero de
formas diferentes en que se puede dibujar el nmero mnimo para n puntos negros, y como
a partir de una situacin con n 1 puntos se puede pasar a una situacin de n puntos de dos
formas distintas (uniendo el n-simo punto negro con cada uno de los puntos rojos), se tiene
la relacin an = 2an 1. As pues la relacin de recurrencia es: an = 2an1 , obteniendo pues
ARITMTICA Y COMBINATORIA
(I. Sucesiones recurrentes)

P. Jara

C AP. III. E JERCICIOS DE REPASO

76

una progresin geomtrica de razn 2. El trmino a0 no tiene significado, y el trmino a1 es


igual a 2. Tenemos entonces la frmula an = 2n .
e
M
M

eU
e
MU

M

MU


M U
M U

M U 
M U 
M U 
M U 
M u Uu

M
M


M
Mu

e
e

MU
U

M]
U

M]

M U]

M U]
M U]
M U ]
M U ]
M U ]
M u Uu ]u

Problema. 9.34.
Determinar una frmula explcita para el trmino general de la sucesin definida:
a0 = 1,
a
, si n 1.
an = 1+an1
n1
S OLUCIN . Tenemos
a1 =
a2 =

1
1
= ,
1+1
2

1/2
1
= .
1 + 1/2
3

Hacemos induccin sobre n; supongamos que se tiene an = n1 , entonces se verifica:


an+1 =

1/n
1/n
1
an
=
=
=
.
1 + an
1 + 1/n
(n + 1)/n
n+1


Problema. 9.35.
Calcular la suma
1 2 3 + 2 3 4 + + n(n + 1)(n + 2)
S OLUCIN . Llamamos an = 1 2 3 + 2 3 4 + + n(n + 1)(n + 2) y a0 = 0, entonces la
sucesin {an }n es una progresin aritmtica de orden cuatro.
b[4]
b[3]
b[2]
b[1]
b[0]

a0

a1
123

a2
234

323

a3
345

334
323

456
345

324
32

a4
567
356
325

32

a5

325
32


28 de enero de 2013

Curso 20122013. NOTAS DE TRABAJO, 30

S EC . 9. E JERCICIOS DE REPASO

77

Problema. 9.36.
Se considera para cada entero positivo o nulo n el polgono regular de n + 3 lados, y llamamos an al nmero de diagonales de dicho polgono.
Dar una expresin explcita de an en funcin de n y comprobar que la sucesin {an }n es una
progresin aritmtica de orden 2.
S OLUCIN . Si numeramos los vrtices de un polgono de n + 3 lados a partir del 1 siguiendo
el sentido de las agujas del reloj, vamos a ver cuantas diagonales podemos construir.
Con base el vrtice 1 tenemos exactamente n diagonales, una por cada vrtice distinto de los
vrtices 1, 2 y n + 3.
Con base el vrtice 2 tenemos exactamente n diagonales, una por cada vrtice distinto de los
vrtices 2, 3 y 1.
Con base el vrtice 3 tenemos exactamente n 1 diagonales, una por cada vrtice distinto de
los vrtices 3, 4 y 2, a los que hay que aadir el vrtice 1, pues la diagonal (1, 3) ya la hemos
contado.
Es claro que con base el vrtice 4 tenemos n2 diagonales, y para el vrtice j tenemos nj+2.
Por tanto el nmero total de vrtices es:
n + n + (n 1) + (n 2) + + 2 + 1 = n +
Tenemos pues una progresin aritmtica de orden 2.

n(n + 3)
(n + 1)n
=
.
2
2


Problema. 9.37.
Se considera para cada entero positivo o nulo n el prisma recto de bases los polgonos regulares de n + 3 lados, y llamamos dn al nmero de todas las diagonales de dicho prisma.
Dar una expresin explcita de dn en funcin de n y comprobar que la sucesin {dn }n es una
progresin aritmtica de orden 2.
S OLUCIN . Consideramos un prisma recto de base un polgono de n + 3 lados. Numeramos
los vrtices del polgono de n+3 lados de la base superior a partir del 1 siguiendo el sentido de
las agujas del reloj, y hacemos la misma numeracin en la base de forma que los vrtices con
el mismo nmero estn unidos por una arista vertical. Veamos cuantas diagonales podemos
construir.
Tenemos todas las diagonales de las bases, en total 2an siguiendo la notacin del problema (9.36.).
Para cada cara lateral, que es un rectngulo tenemos dos diagonales, en total 2(n + 3) ya que
tenemos n + 3 caras.
Slo nos queda por contar las diagonales interiores al prisma. Del vrtice 1, de la base superior, podemos construir las diagonales a los vrtices de la base inferior, una por cada vrtice
distinto del vrtice 1, tenemos entonces n diagonales, una para cada uno de los vrtices distintos de 1, 2 y n + 3. Como esta construccin la podemos hacer para cada vrtice de la base
superior, tenemos por tanto n(n + 3).
ARITMTICA Y COMBINATORIA
(I. Sucesiones recurrentes)

P. Jara

C AP. III. E JERCICIOS DE REPASO

78

Al sumar todos estos nmeros tenemos que el nmero total de diagonales es:
2

n(n + 3)
+ 2(n + 3) + n(n + 3) = 2(n + 1)(n + 3).
2

Si llamamos dn a este nmero, resulta que la sucesin {dn }n es una progresin aritmtica de
orden 2.

Problema. 9.38.
Sea f (1) = 1 y f (1) + f (2) + + f (n) = n2 f (n). Calcular razonadamente f(2001).
[Gacetilla Matemtica. Problema 107]
S OLUCIN . De entre las muchas formas de resolver este problema vamos a mostrar una en
la que se utilizan progresiones aritmticas.
Tenemos
f (1) + f (2) = 22 f (2),
1 = (22 1)f (2),
f (2) = 31
f (1) + f (2) + f (3) = 32 f (3),
1 + 31 = (32 1)f (3),
f (3) = 16
1
1
En la misma forma se tiene f (4) = 10
, f (5) = 15
. Podemos conjeturar que se tiene f (n) =
1 !
2
= (n+1)n ; esto es, las denominadores son los nmeros triangulares: 1, 3, 6, 10, 15, . . . ,
n+1
2


n+1
con trmino general
comenzando en a1 .
2
Vamos a hacer induccin para probar este resultado. Supongamos que el resultado es cierto
para n y vamos a probarlo para n + 1.

f (n + 1) =
=

f (1)+f (2)++f (n)+f (n+1)


(n+1)2 1
2
n f (n)
(n+1)2 1
n2 1

n + 1

=
=
=
=
=
28 de enero de 2013

(n+1)2 1
2n2
(n+1)n
(n+1)2 1

2n2
(n+1)n((n+1)2 1)
2n2
(n+1)n(n+2)n
2
(n+2)(n+1) .

Curso 20122013. NOTAS DE TRABAJO, 30

S EC . 9. E JERCICIOS DE REPASO

79

Para el valor que se pide, tenemos:


f (2001) =

2
1
1
=
=
.
(2001 + 1)2001
1001 2001
2003001


Problema. 9.39. (Los nmeros de Pell)


Se define al sucesin de Pell mediante las condiciones iniciales a0 = 0 y a1 = 1 y la ecuacin
de recurrencia
an = 2an1 + an , si n 2.
Determinar el trmino general de la sucesin de Pell.
S OLUCIN .

Problema. 9.40. (Nmeros de Padovan)
Se considera la sucesin recurrente definida por:
a0 = a1 = a2 = 1,
an = an2 + an3 , si n 3.
Determinar el trmino general de esta sucesin. Los nmeros de la sucesin {an }n se llaman
nmeros de Padovan.
Los nmeros de Padovan
aparecen en el lugar (2, 2) (coeficiente central) de la matriz M n+2 ,

010

siendo M la matriz 0 0 1.
110
S OLUCIN .

Problema. 9.41. (Nmeros de Jacobsthal)
Se considera la sucesin {an }n definida por:
a0 = 0, a1 = 1,
an = an1 + 2an2 , si n 2.
Dar una descripcin explcita del trmino general an .Demostrar que se verifican las siguientes condiciones:
(1) P
an +1an1 an2 = (1)n an1 .
(2) ni=2 ai = 21 (an+2 3).
S OLUCIN . .
ARITMTICA Y COMBINATORIA
(I. Sucesiones recurrentes)


P. Jara

C AP. III. E JERCICIOS DE REPASO

80

Problema. 9.42.
Se realiza un experimento que consiste en lanzar un dado hasta que aparezcan dos nmeros pares. Determinar una relacin de recurrencia para el nmero de experimentos que
finalizan en el n-simo lanzamiento o antes.
S OLUCIN . Llamamos an1 al nmero de experimentos que finalizan antes del n-simo lanzamiento. Si un experimento finaliza en el n-simo lanzamiento, puede ser debido a una de
las siguientes posibilidades:
(a). uno de los n 1 lanzamientos anteriores era par; tenemos n 1 posibilidades,
(b). si uno de los lanzamientos era para tenemos para cada uno tres posibles valores; tenemos 3 posibilidades,
(c). tenemos exactamente n 2 nmeros impares, y en total el nmero de posibilidades para
estos es: 3n2 .
(d). el ltimo lanzamiento es para, tenemos tres posibilidades.
En total se tiene entonces
an = an1 + (n 1) 3 3n2 3 = an1 + (n 1) 3n .
con valores iniciales a0 = 0, a1 = 0
Se trata ahora de dar un valor explcito para an . Como es una recurrencia lineal no homognea, tenemos que determinar una solucin particular y una solucin general de la recurrencia homognea. Comencemos por sta ltima.
La ecuacin caracterstica es: r = 1, luego una solucin general es: an = 1 .
Como la parte no homognea es: (n 1)3n , y sta proviene de la ecuacin caracterstica (r
3)(r 3), entonces una solucin particular se obtiene al considerar la ecuacin caracterstica
(r 1)(r 3)2 , y eliminar de la solucin la parte correspondiente a (r 1); se tiene entonces
(2 + 3 n)3n ; al introducir este valor en la recurrencia se tiene:
(2 + 3 n)3n (2 + 3 (n 1))3n1 (n 1) 3n = 0
(2 + 3 n)3 (2 + 3 (n 1)) (n 1) 3 = 0
32 + 33 n 2 3 n + 3 3n + 3 = 0
32 2 + 3 + 3 + (33 3 3)n = 0

De aqu se obtiene

22 + 3 + 3 = 0
23 3 = 0

cuya solucin es: 2 = 49 y 3 = 32 .

La solucin general de la recurrencia no homognea es: ( 94 + 23 n)3n + 1 . Al imponer las


condiciones iniciales se tiene:
( 94 + 32 0)30 + 1 = 0,
( 49 + 32 1)31 + 1 = 0,
28 de enero de 2013

Curso 20122013. NOTAS DE TRABAJO, 30

S EC . 9. E JERCICIOS DE REPASO

81

luego 1 = 94 , y el trmino general es: an = ( 49 + 32 n)3n + 49 .

Problema. 9.43.
Encontrar el trmino general de la sucesin {an }n definida por la recurrencia
an = an2 , si n 2, a0 = 1, a1 = 0.
S OLUCIN . La ecuacin caracterstica es: r 2 = 1, cuya races son 1 = i y 2 = i, entonces
una solucin general es: an = 1 in + 2 (i)n , al imponer las condiciones iniciales se tiene:

1 + 2 = 1
1 i + 2 (i) = 0
de donde resulta: 1 = y 2 =. El trmino general es: an = ( 21 in + 12 (i)n ).

Problema. 9.44.
Se considera la sucesin {an }n verificando la recurrencia
a1 = 1,
n
an1 + n3 , si n 2.
an = n1
Determinar el trmino general.
S OLUCIN . Definimos sucesiones auxiliares {bn }n y {cn }n mediante:
bn =

n!
(n1 !

= n, si n 1,

c1 = 1, cn = cn1 +

n3
bn

= cn1 + n2 , si n 2.

Si suponemos que ar = br cr para r < n, por induccin tenemos:


n
n
bn cn = n(cn1 + n2 ) = ncn1 + n3 =
(n 1)cn1 + n3 =
an1 + n3 = an .
n1
n1
El trmino general de {cn }n es fcil, ya que es la suma de los n primeros cuadrados: cn =
2n3 +3n2 +n
, y por tanto
6
an = bn cn = n

2n4 + 3n3 + n2
n2 (n + 1)(2n + 1)
2n3 + 3n2 + n
=
=
.
6
6
6


Problema. 9.45.
Se considera la sucesin {an }n verificando la recurrencia
a0 = 1,
an = nan1 + (1)n , si n 1.
Determinar el trmino general.
ARITMTICA Y COMBINATORIA
(I. Sucesiones recurrentes)

P. Jara

C AP. III. E JERCICIOS DE REPASO

82

S OLUCIN . Definimos sucesiones auxiliares {bn }n y {cn }n mediante:


bn = n!,
c0 = 1, cn = cn1 +

(1)n
bn

= cn1 +

(1)n
n! ,

si n 1.

Se verifica an = bn cn . Como el trmino general de {cn }n es


1
1
(1)n
+ +
,
2! 3!
n!
resulta que el trmino general de an es:
n!(

1
1
(1)n
+ +
).
2! 3!
n!


28 de enero de 2013

Curso 20122013. NOTAS DE TRABAJO, 30

Captulo IV
Ejercicios avanzados
10.

Ejercicios avanzados

Ejercicios avanzados
Problema. 10.1.
Se considera la sucesin {Sn }n definida para cada entero positivo o nulo n mediante:
Sn = 1 + 3 + 5 + + (2n 1)
Determinar el trmino general de esta sucesin.
S OLUCIN . Es claro que si n = 0, entonces S0 = 1 + (1) = 0 y S1 = 1, S2 = 1 + 3 = 4, etc.
Adems se verifica la siguiente relacin de recurrencia
Sn = Sn1 + (2n 1).
Como consecuencia la ecuacin caracterstica es: r = 1, pero como no es una recurrencia
homognea, el trmino no homogneo es: 2n 1. Una solucin particular ser de la forma
an2 + bn + c; vamos a ver cuales son los coeficientes:
an + bn + c = a(n 1)2 + b(n 1) + c + (2n 1),
2an a + b 2n + 1 = 0
2(a 1)n + (a + b + 1) = 0,



a1=0
luego
, esto es, a = 1, b = 0 y c es indeterminado. Una solucin particular
ab1=0
es: n2 . La solucin de la recurrencia homognea Sn = Sn1 es 1, luego la solucin general es:
1 + n2 ,
83

C AP. IV. E JERCICIOS AVANZADOS

84

sujeta a las condiciones iniciales S0 = 0, S1 = 1, luego tenemos:


1 + 02 = 0,
de donde se tiene 1 = 0 y el trmino general es Sn = n2 .
Nota. Podramos haber considerado tambin la solucin de 1 +12 = 1, obteniendo el mismo
resultado.
Nota. Tambin podramos haber supuesto al inicio que el trmino general es Sn = n2 , algo
que se deduce de una simple observacin, y probar el resultado por induccin sobre n.

Problema. 10.2.
Determinar el trmino general de la sucesin {an }n definida por la recurrencia
an = 3an1 + 5 7n , si n 1 y a0 = 2.
S OLUCIN . Tenemos an = 41 (35 7n 27 3n ).

Problema. 10.3.
Determinar el trmino general de la sucesin {an }n definida por la recurrencia
an = 3an1 + 5 3n , si n 1 y a0 = 2.
S OLUCIN . Tenemos an = (2 + 5n)3n .

Problema. 10.4.
Determinar el trmino general de la sucesin {an }n definida por la recurrencia
an = 4an1 4an2 + n2 , si n 2 y a0 = 0, a1 = 2.
55
S OLUCIN . Tenemos an = 54
n(2)n +

1
2
27 (3n

4n).

Problema. 10.5.
Determinar el trmino general de la sucesin {an }n definida por la recurrencia
an = 5an1 8an2 + 4an3 , si n 3 y a0 = 1, a1 = 2, a2 = 3.
S OLUCIN .

Problema. 10.6.
Determinar el trmino general de la sucesin {an }n definida por la recurrencia
an an1 = 4[(an1 an2 ) (an2 an 3)], si n 2 y a0 = 0, a1 = 1, a2 = 3.
S OLUCIN . an = 2n 1
28 de enero de 2013


Curso 20122013. NOTAS DE TRABAJO, 30

S EC . 10. E JERCICIOS AVANZADOS

85

Problema. 10.7.
Los dos primeros trminos de una sucesin son, respectivamente, 1 y 2. Si cada trmino es
la media aritmtica del anterior con la media aritmtica de los dos adyacentes (anterior y
posterior), determinar:
(1) Una forma explcita del trmino general de la sucesin.
(2) Probar mediante induccin la validez del resultado obtenido.
(3) Un procedimiento para calcular el trmino cuadragsimo realizando a lo ms 10 operaciones.
S OLUCIN .

Problema. 10.8.
Determinar el trmino general de la sucesin {an }n definida por la recurrencia
an = 6an1 9an2 , si n 2 y a0 = 1, a1 = 6.
S OLUCIN . an = (n + 1)3n

Problema. 10.9.
Determinar el trmino general de la sucesin {an }n definida por la recurrencia
an = 6an1 9an2 + 4n, si n 2 y a0 = 1, a1 = 6.
S OLUCIN . an = (8n 6)3n1 + n + 3.

Problema. 10.10.
Determinar a y b, siendo a el nmero de enteros positivos menores o iguales que 100 que no
son divisores ni por 3 ni por 7 ni por 11 y b es el nmero de enteros positivos, en el mismo
rango, que son divisibles por 2 y por 9.
Dar la forma explcita del trmino general de la sucesin definida por la recurrencia
an = aan1 (130b+)an2 , si n 2, a0 = 0, A1 = 10.
Siendo a y b los nmeros obtenidos en el apartado anterior. Probar que cada trmino de la
sucesin es un mltiplo de 10.
S OLUCIN . a = 52, b = 5 an = 31n 21n = (31 21)(31n1 + 31n2 21 + 31n3 212 + +
312 21n3 + 31 21n2 + 21n1 )

Problema. 10.11.
Nos regalan tres sellos y decidimos comenzar una coleccin. El ao siguiente compramos
8 nuevos sellos, de forma que tenemos ya 11 sellos en nuestra coleccin. Si cada ao compramos un nmero de nuevos sellos igual al doble de los que compramos el ao anterior,
al cabo de cuntos aos habremos superado los 100.000 sellos?
ARITMTICA Y COMBINATORIA
(I. Sucesiones recurrentes)

P. Jara

C AP. IV. E JERCICIOS AVANZADOS

86
S OLUCIN . an = 2n+2 5. El nmero de aos es 13.

Problema. 10.12.
Se trazan en el plano n rectas de forma que cada una de ellas corta a todas las dems y no
existen tres que se corten en un mismo punto.
Dar una frmula explcita para an , el nmero de regiones en que estas n rectas dividen al
plano?
Dar una frmula para bn el nmero de regiones no acotadas que aparecen en la configuracin anterior.
S OLUCIN .

Problema. 10.13.
Existe un pas de cuyo nombre no puedo ahora acordarme cuya moneda oficial
es el euro, y que tiene los siguientes valores faciales para sus monedas y billetes:
Monedas
9 y 19 euros
Billetes
9, 19, 125 y 232 euros
Puede cambiarse algn billete de ms de 100 euros en monedas?
En caso afirmativo, de cuntas formas puede hacerse?
Se ha propuesto por el consejo de ministros de este pas realizar una emisin de nuevos
billetes hasta completar una serie de billetes con cien valores diferentes. El ministro de hacienda ha observado que los actuales billetes cumplen la recurrencia siguiente:
an = 2an1 an2 + 329n 815, si n 3, a1 = 9, a2 = 19.
y ha decidido que los nuevos billetes tambin cumplan con esta relacin. Cul es el valor
del ltimo billete de la nueva serie?
S OLUCIN . an = 14 [(1)n (97n 290) + 329n 486] a100 = 14 [(1)100 (97 100 290) + 329
100 486] = 10456 euros.

Problema. 10.14.
Determinar el trmino general de la sucesin {an }n definida por la recurrencia
an = 4an1 5an2 + 2an3 2, si n 3 y a0 = 2, a1 = 5, a2 = 11.
S OLUCIN . an = 2n + n2 + n + 1

Problema. 10.15.
Ana ha abierto una cuenta en un banco y ha realizado una imposicin por valor de 1.000
euros. Resulta que el banco le da a Ana un inters anual del 6 %, y que le hace ingresos
mensuales del inters. Qu cantidad tiene Ana al cabo de un ao?
28 de enero de 2013

Curso 20122013. NOTAS DE TRABAJO, 30

S EC . 10. E JERCICIOS AVANZADOS

87

S OLUCIN . Si llamamos an a la cantidad de dinero de Ana en el mes n, resulta que a0 = 1,000


y an = an1 + 0, 005an1 = 1, 005an1 , ya que el inters del 6 % anual se traduce en un inters
del 0,5 % mensual. S tiene entonces an = 1, 005n a0 .


Problema. 10.16.
Este es el cuento de la lechera para hacerse tremendamente rico. Primero ahorramos todo
lo que podamos hasta que juntemos un capital respetable, y del que podamos prescindir en
caso de necesidad, por ejemplo supongamos que en un ao podemos ahorrar 1.000 euros.
El segundo ao seguimos ahorrando y comenzamos a hacer negocios, compraventa de
productos y acciones, etc., que menos que juntar al menos 4.000 euros. Ya con esta respetable cantidad podemos dedicarnos a ms actividades econmicas, de forma que podemos
cada ao comprar bienes por cuatro veces nuestro capital del ao anterior y venderlo por
tres veces el capital de ese ao, parece una situacin posible verdad?
Determinar una frmula que nos d el capital al final de n aos.
Al cabo de cuanto tiempo tendremos al menos un milln de euros de capital?
Qu cantidad tendremos al cabo de 15 aos?
S OLUCIN .


Problema. 10.17.
Se tiene la siguiente recurrencia:
a0 = 0, a1 = 9, a2 = 1, a3 = 21,
an = 5an1 6an2 4an3 + 8an4 , si n 4.
Dar una descripcin explcita del trmino general.
S OLUCIN . an = (n2 n 3)2n + 3(1)n .

Problema. 10.18.
Se tiene la siguiente recurrencia:
a0 = 0, a1 = 1,
an = an1 + 6an2 + n, si n 2.
Dar una descripcin explcita del trmino general.
S OLUCIN . an =

1
n
36 (5 6(n + 1) + 9 3

ARITMTICA Y COMBINATORIA
(I. Sucesiones recurrentes)

8 (2)n ).


P. Jara

C AP. IV. E JERCICIOS AVANZADOS

88

Problema. 10.19.
Se tiene la siguiente recurrencia:
a0 = 1,
an = 2an1 + 4n , si n 1.
Dar una descripcin explcita del trmino general.
S OLUCIN .

Problema. 10.20.
Se considera la sucesin definida por:
a0 = 0,

an+1 =

p
6 + an , si n 0.

(1) Prueba que {an }n es una sucesin montona creciente.


(2) Prueba que {an }n est acotada superiormente por 3.
(3) Calcula el lmite de {an }n .
S OLUCIN .
(1) Tenemos a1 =
6 se tiene:

6 > a0 , Supongamos que para todos t n se verifica at1 < at , sumando


6 + at1 < 6 + at ,

calculando la raz cuadrada se tiene:


p
p
at = 6 + at1 < 6 + at = at+1 ,
por lo tanto la sucesin es creciente.
(2) Supongamos que at < 3 para cada t n, entonces 6 + at < 6 + 3 = 9, y tomando raz
cuadrada resulta
p

at+1 = 6 + at < 9 = 3.

(3) Como an+1 = 6 + an , tomando lmite, si ste existe, se tiene l = 6 + l, luego l2 = 6 + l,


de donde resulta que l es raz de l2 l 6. luego l = 2 3. Por lo tanto el lmite es 3.

Problema. 10.21.
Determina el nmero an de permutaciones del conjunto {1, 2, . . . , n} que verifican |(i)
i| 1 para i = 1, 2, . . . , n.
28 de enero de 2013

Curso 20122013. NOTAS DE TRABAJO, 30

S EC . 10. E JERCICIOS AVANZADOS

89

S OLUCIN . Consideramos que n permanece fijo, entonces el nmero de las permutaciones


pedidas es el nmero de permutaciones de este tipo del conjunto {1, 2, . . . , n1}, por lo tanto
son an1
Ahora consideramos que n no es fijo, en este caso se tiene (n) = n 1, y tambin se verifica
(n 1) = n. Resulta que el nmero de permutaciones pedidas es en ese caso el del conjunto
{1, 2, . . . , n 2}, por lo tanto son an2 .
Los valores iniciales son: a0 = 1, a1 = 1 y a2 = 2, por lo tanto la sucesin
es la de Fibonacci

5 n+1
{fn }n pero corrida una posicin, esto es, an = fn+1 . Tenemos pues an = 5
n+1 ).

Problema. 10.22.
Determina el nmero bn de permutaciones del conjunto circular {1, 2, . . . , n} que verifican |(i) i| 1. (En este caso suponemos que 1 = n + 1 cuando sea necesario.)
S OLUCIN . Analizamos los distinto casos segn n.
1. n queda fijo; tenemos exactamente an1 permutaciones, siendo {an }n la sucesin del problema (10.21.).
2. (n) = n 1. Puede ocurrir que (n 1) = n, en este caso tenemos exactamente an1
permutaciones. Por el contrario, si (n 1) 6= n tiene que ser (1) = n, lo que obliga a que
(2) = 1, (3) = 2, . . . , (n 1) = n 2, que es una permutaciones cclica. Por lo tanto el
nmero total de permutaciones en este caso es: an2 + 1.
3. Si (n) = 1. Puede ocurrir que (1) = n, y tenemos an2 permutaciones o por el contrario
que (1) 6= n, en este caso tenemos la permutacin circular: (1) = 2, (2) = 3, . . . , pi(n1) =
n. El nmero total de permutaciones es: an2 + 2.
Tenemos entonces bn = an1 + 2an2 + 2. Vamos a calcular estevalor. Sabemos que an =
an1 + an2 , luego se tiene: bn = an + an2 + 2. Por otro lado an = 55 (n+1 n+1 ), entonces:

bn =
=
=

5 n+1
n+1 + n1 n1 ) + 2 = 5 (n1 (2 + 1) n1 ( 2
(

5
5

5(2 +1)
5( 2 +1)
n1
n1

+ 2 = n1 n1 () + 2
5
5
n + n + 2.

+ 1)) + 2

ARITMTICA Y COMBINATORIA
(I. Sucesiones recurrentes)

P. Jara

90

28 de enero de 2013

C AP. IV. E JERCICIOS AVANZADOS

Curso 20122013. NOTAS DE TRABAJO, 30

Captulo V
Ejemplos
11.

Ejemplos

Ejemplos
Ejemplo. 11.1.
Se considera n rectas en el plano de forma que se corten dos a dos y no hay tres coincidentes.
Llamamos an al nmero de regiones en que se divide el plano cuando tenemos n rectas, y
llamamos bn al nmero de regiones no acotadas. Dar frmulas para an y bn .
S OLUCIN . Si tenemos una sola recta, entonces a1 = 2 = b1 . Para el caso n = 2 resulta
a2 = 4 = b2 . Para n = 3 ya tenemos regiones no acotadas. En efecto, la nueva recta corta a
cada una de las existentes y hace aparece nuevas regiones, exactamente tres, una ms que
las rectas previamente existentes. Para ver esto basta con considerar los puntos de corte de
la nueva recta con las rectas existentes, en este caso dos, que dividen a la nueva recta en tres
segmentos, una acotado y dos no acotados; cada uno de estos segmentos no acotados es
una frontera de regiones no acotadas, y el segmento acotado determina una regin acotada
y otra no acotada. As pues en este caso resulta a3 = 4 + 3 = 7, y b3 = 4 + 2 = 6. Para
n > 3 tenemos an = an1 + n, ya que la nueva recta queda dividida en n segmentos, por
los puntos de corte con las rectas existentes; y cada segmento divide regiones previas en dos,
luego tenemos n nuevas regiones. Para contar las regiones no acotadas, basta ver que los
segmentos no acotados dividen regiones no acotadas en regiones no acotadas, mientras que
un segmento acotado divide una regin acotada en dos regiones acotadas, y una regin no
acotada en dos, una acotada y la otra no, luego tenemos la recurrencia bn = bn1 + 2.
Caso de la sucesin an . La frmula de recurrencia es an = an1 + n, luego la recurrencia
homognea es an = an1 ; la solucin homognea es la sucesin contante igual a 1. La solucin de la parte no homognea es 2 n2 + 1 n + 0 . Al comprobar la frmula de recurrencia
91

C AP. V. E JEMPLOS

92
tenemos:

2 n2 + 1 n + 0 = 2 (n 1)2 + 1 (n 1) + 0 + n
0 = 22 n + 2 1 + n
1
2 = 1
es: + 12 (n2 + n);

2 =

La solucin general

al imponer las condiciones iniciales tenemos:

+1=2

+ 21 (4 + 2) = 4

+ 12 (9 + 3) = 7

La solucin es: = 1, luego el trmino general an es: an = 1 + 12 n(n + 1).


Caso de la sucesin bn . La frmula de recurrencia es bn = bn1 + 2, luego la recurrencia homognea es bn = bn1 ; la solucin homognea es la sucesin contante igual a 1. La solucin
de la parte no homognea es 1 n + 0 . Al comprobar la frmula de recurrencia tenemos:
1 n + 0 = 1 (n 1) + 0 + 2
0 = 1 + 2
1 = 2
La solucin general es: + 2n; al imponer las condiciones iniciales tenemos:

+2=2
+ 2(2) = 4

+ 2(3) = 6
La solucin es: = 0, luego el trmino general bn es: bn = 2n.

Ejemplo. 11.2.
Se considera un conjunto de 2n , n 1, nmeros reales. Cuntas comparaciones son necesarias hacer, entre pares de estos nmeros, para determinar el mximo y el mnimo de este
conjunto?
S OLUCIN . Llamamos an el nmero de comparaciones necesarias en el caso de n. Tenemos a1 = 1. Para n 2, para calcular an consideramos el conjunto S = {x1 , x2 , . . . , x2n }. Si
descomponemos S = {x1 , x2 , . . . , x2n1 } {x2n1 +1 , x2n1 +2 , . . . , x2n } y aplicamos el paso previo, necesitamos entonces 2an1 comparaciones. Finalmente comparamos los mximos y los
mnimos de cada conjunto, por lo que necesitamos hacer dos comparaciones ms. Entonces
an = 2an1 + 2 = 2(an1 + 1), si n 1.
La solucin general de la resolucin homognea es 2n , y la solucin de la pate no homognea es la sucesin contante, luego una solucin genrica ser . Comprobamos la frmula de
28 de enero de 2013

Curso 20122013. NOTAS DE TRABAJO, 30

S EC . 11. E JEMPLOS

93

recurrencia para esta solucin genrica:


= 2 + 2
= 2
La solucin general es: 2n 2, y al imponer las condiciones iniciales tenemos:
2 2 = 1
Entonces = 32 . La solucin es: an = 23 2n 2 = 3 2n1 2, si n 1.

Ejemplo. 11.3.
Pablo pide un anticipo de A euros que debe pagar en t plazos. Si llamamos i al inters en
cada uno de los plazos, qu cantidad, contante, P debe pagar Pablo al final de cada plazo?
S OLUCIN . Llamamos an a la cantidad de dinero que debe al final del plazo n-simo. Tenemos la siguiente relacin de recurrencia: an+1 = an + ian P, con las condiciones a0 = A,
at = 0 y 0 n t 1.
Consideramos la relacin de recurrencia homognea: an+1 = (1 + i)an ; una solucin general
es (1 + i)n .
La parte no homognea es P, una solucin genrica es ; el imponer la frmula de recurrencia tenemos:
= (1 + i) P
= Pi
La solucin general es: (1 + i)n + Pi ; si imponemos las condiciones iniciales resulta:


(1 + i)0 + Pi = A
= A Pi
(1 + i)t + Pi = 0
(1 + i)t + Pi = 0


P
P
A
(1 + i)t + = 0
i
i

P
1 (1 + i)t = A(1 + i)t
i
A(1 + i)t i
A(1 + i)t i
Ai
=
=
P=
1
1 (1 + i)t
(1 + i)t 1
1 (1+i)
t

Ejemplo. 11.4.
Resolver la relacin de recurrencia an = 3an1 + 5 3n con la condicin inicial a0 = 2.
S OLUCIN . Tenemos an = 3an1 + f (n), y consideramos la relacin de recurrencia homognea an = 3an1 ; una solucin general es: 3n .
ARITMTICA Y COMBINATORIA
(I. Sucesiones recurrentes)

P. Jara

C AP. V. E JEMPLOS

94

Consideramos tambin la solucin del trmino no homogneo, en este caso n3n . Al sustituir
en la frmula de recurrencia se tiene
n3n = 3(n 1)3n1 + 5 3n ,
simplificando se obtiene = 5. Una solucin particular es: 5n3n .
La solucin general es: 3n + 5n3n ; al imponer las condiciones iniciales resulta:
= 2.
La solucin es: an = 2 3n + 5n3n = (2 + 5n) 3n .

Ejemplo. 11.5.
Resolver la relacin de recurrencia an = 3an1 + 5 7n con la condicin inicial a0 = 2.
S OLUCIN . Tenemos an = 3an1 + f (n), y consideramos la relacin de recurrencia homognea an = 3an1 ; una solucin general es: 3n .
Consideramos tambin la solucin del trmino no homogneo, en este caso 7n . Al sustituir
en la frmula de recurrencia se tiene
7n = 37n1 + 5 7n ,
35 n
5 n+1
.
simplificando se obtiene = 35
4 . Una solucin particular es: 4 7 = 4 7
5 n+1
n
; al imponer las condiciones iniciales resulta:
La solucin general es: 3 + 4 7

=
n
La solucin es: an = 27
4 3 +

5
4

7n+1 =

5
4

27
.
4

7n+1

1
4

3n+3 .

Ejemplo. 11.6.
Resolver la relacin de recurrencia an = an1 + 3n2 con la condicin inicial a0 = 7.
S OLUCIN . Tenemos an = an1 + f (n), y consideramos las siguientes relaciones:
a1 = a0 + f (1)
a2 = a1 + f (2) = a0 + f (1) + f (2)
..
.
an = a0 + f (1) + + f (n)
Se tiene entonces an = a0 +

Pn

i=1 3i

= a0 + 3

Pn

i=1 i

.
= 7 + 3 n(n+1)(2n+1)
2

Ejemplo. 11.7.
Sea an el nmero de sucesiones de los dgitos 0, 1 y 2 en las que no hay dos ceros consecutivos. Determinar una frmula para an .
28 de enero de 2013

Curso 20122013. NOTAS DE TRABAJO, 30

S EC . 11. E JEMPLOS

95

S OLUCIN . Para n = 1 tenemos a1 = 3. Para n = 2 tenemos a2 = 8, ya que la posibles


sucesiones son: 0,1; 0,2; 1,0; 1,1; 1,2; 2,0; 2,1; 2,2.
La forma de construir una sucesin de longitud n a partir de una de longitud n 1 es agregando un 1 y 2; de esta forma tenemos todas las posibles sucesiones que acaban en 1 2.
Si una sucesin de longitud n acaba en 0, entonces los n 2 primeros trminos pueden ser
arbitrarios, pero el trmino n 1 es o bien 1 o bien 0; esto significa que el valor es an es:
an = 2an1 + 2an2
2
La ecuacin
caracterstica es: r = 2r + 2; las soluciones de esta ecuacin son: = 1 +
= 1 3.
La solucin general es:

1 (1 + 3)n + 2 (1 3)n .

3y

Al imponer las condiciones iniciales resulta:


1 (1 + 3) + 2 (1 3) = 3
1 (1 + 3)2 + 2 (1 3)2 = 8


1 (1 +
3) + 2 (1 3)
= 3
1 (4 + 2 3) + 2 (4 2 3) = 8
Al multiplicar la primera ecuacin por 2 y restarla a la segunda se obtiene: 2(1
+ 2 ) = 2,
esto es, 1 + 2 = 1. Al sustituir este valor en la primera ecuacin
resulta: 1 (1
+ 3) + (1

3+2 3
32 3
1 )(1 3) = 3, esto es, (21 1) 3 + 1 = 3; y de aqu 1 = 3 y 2 = 6 .
El resultado final es:

n 32 3

3+2 3
an =
(1 + 3) +
(1 3)n .
3
6

Ejemplo. 11.8.
Sea an el nmero de sucesiones de los dgitos 0, 1 y 2 en las que no hay dos uno dos ceros
consecutivos. Dar una frmula para an .
Ver el problema (9.5.).
S OLUCIN . Para n = 1 tenemos a1 = 3. Para n = 2 tenemos a2 = 7, ya que las sucesiones
que podemos construir son: 0,1; 0,2;1,0; 1,2;2,0; 2,1; 2,2. Para n = 3 tenemos a3 = 17, ya que
las sucesiones que podemos construir son: 0,1,0; 0,1,2; 0,2,0; 0,2,1; 0,2,2;
1,0,1; 1,0,2; 1,2,0; 1,2,1; 1,2,2;
2,0,1; 2,0,2; 2,1,0; 2,1,2; 2,2,0; 2,2,1; 2,2,2.
A partir de una sucesin se longitud n 1 podemos construir otra de longitud n agregando
un 2; si el lugar n 1 es distinto de 2, entonces podemos construir una secesin de longitud n
agregando 0 1, segn corresponda; si el lugar n 1 es igual a 2, entonces podemos construir
ARITMTICA Y COMBINATORIA
(I. Sucesiones recurrentes)

P. Jara

C AP. V. E JEMPLOS

96

dos de longitud n, agregando 0 y 1; el nmero de sucesiones vlidas de longitud n 1 con un


dos en el lugar n 1 es exactamente an2 , entonces tenemos la siguiente frmula para an .
an = an1 + an1 + an2 = 2an1 + an2 .
Vamos a comprobarlo: a3 = 2a2 + a1 = 2 7 + 3 = 17. Otra comprobacin se puede hacer al
calcular a4 ; ser a4 = 2a3 + a2 = 2 17 + 7 = 41; en efecto, tenemos
0,1,0,2; 0,1,2,2; 0,2,0,2; 0,2,1,2; 0,2,2,2;
1,0,1,2; 1,0,2,2; 1,2,0,2; 1,2,1,2; 1,2,2,2;
2,0,1,2; 2,0,2,2; 2,1,0,2; 2,1,2,2; 2,2,0,2; 2,2,1,2; 2,2,2,2;
0,1,0,1; 0,1,2,0; 0,2,0,1; 0,2,1,0; 0,2,2,0;
1,0,1,0; 1,0,2,0; 1,2,0,1; 1,2,1,0; 1,2,2,0;
2,0,1,0; 2,0,2,1; 2,1,0,1; 2,1,2,0; 2,2,0,1; 2,2,1,0; 2,2,2,0;
0,1,2,1; 0,2,2,1;
1,0,2,1; 1,2,2,1;
2,0,2,1; 2,1,2,1; 2,2,2,1.
Tenemos entonces que la ecuacin caracterstica es:
r 2 = 2r + 1,

cuyas races son: = 1 + 2 y = 1 2. Una solucin general es:

1 (1 + 2)n + 2 (1 2)n ;
al imponer las condiciones iniciales resulta:


1 (1 + 2) + 2 (1 2) = 3
1 (1 + 2)2 + 2 (1 2)2 = 7


1 (1 +
2) + 2 (1 2)
=
3

1 (3 + 2 2) + 2 (3 2 2)2 = 7
Multiplicando la primera ecuacin por 2 y restando a la segunda se tiene 1 + 2 = 1. Sus

1+ 2
tituyendo este valor
en
la
primera
ecuacin
resulta:
(2

1)
2
+
1
=
3,
luego

=
y
1
1
2

entonces 2 = 12 2 .
Vamos a comprobar que realmente esta es la solucin. En efecto,

a1 = 1+ 22+1 (1 + 2) + 12 2 (1 2) = 3+22 2 + 32 2 = 3.

2 12
2 (1+2)3 (12)3
1+ 2
a2 = 2 (1 + 2) + 2 (1 2) =
+
= 7.
2
2

Ejemplo. 11.9.
Sea an el nmero de sucesiones de 1 y 2 de forma que su suma sea n. Dar una frmula para
an .
28 de enero de 2013

Curso 20122013. NOTAS DE TRABAJO, 30

S EC . 11. E JEMPLOS

97

S OLUCIN . Para n = 1 tenemos a1 = 1. Para n = 2 tenemos a2 = 2, ya que existen dos


sucesiones: 1,1; 2. Para n = 3 tenemos a3 = 3, ya que existen tres sucesiones: 1,1,1; 1,2; 2,1.
De cada sucesin que suma n 1 podemos construir una sucesin que suma n agregando un
1, y de cada sucesin que suma n 2 podemos construir dos sucesiones que suman n, agregando un 2 agregando 1, 1; este ltimo caso ya ha sido considerado, por lo que la frmula
para an es la siguiente:
an = an1 + an2 .

La ecuacin caracterstica es: r 2 = r + 1. Las races son: = 1+2


general es:

1+ 5 n
1 5 n
1 (
) + 2 (
) .
2
2

y =

1 5
2 ,

y una solucin

Al imponer las condiciones iniciales resulta

1 ( 1+25 ) + 2 ( 12 5 ) = 1
1 ( 1+2 5 )2 + 2 ( 12 5 )2 =

Las soluciones son: 1 =

1+ 5
2 5

)
2

y 2 = 1 5 , por lo que el resultado final es:

1
an =
5

2 5

!n+1
1
1+ 5

2
5

Nota. Ver la sucesin de Fibonacci.

!n+1
1 5
.
2


Ejemplo. 11.10.
Tenemos motocicletas y coches. Determinar el nmero de formas de acomodar motocicletas y coches en una hilera de n huecos si cada motocicletas necesita un hueco y cada coche
necesita dos. (Las motocicletas y los coches son indistinguibles entre s.)
S OLUCIN . Llamamos an al nmero de formas posibles para una hilera de n huecos. Si tenemos una ordenacin de n huecos y al final tenemos una motocicleta, los n 1 huecos
restantes se pueden ordenar de an1 formas, y si tenemos al final un coche, los n 2 huecos
restantes se pueden ordenar de an2 formas, por lo tanto el valor de an es:
an = an1 + an2 .
Los valores iniciales son a1 = 1 y a2 = 2, por lo tanto la resolucin del problema es similar al
Ejercicio (11.9.).

ARITMTICA Y COMBINATORIA
(I. Sucesiones recurrentes)

P. Jara

C AP. V. E JEMPLOS

98

Ejemplo. 11.11.
Si un caso de una enfermedad infecciosa se descubre en un colegio, se pn la probabilidad
de que al menos un caso se descubra en la semana nsima. Se tiene la siguiente evidencia:
1
pn = pn1 pn2 , si n 2.
4
Con las condiciones iniciales p0 = 0 y p1 = 1. Cundo se tendr una probabilidad inferior
1
a 100
?
S OLUCIN . La ecuacin caracterstica es: r 2 = r 14 , y sus races son: = 12 con multiplicidad
dos.
n
n
Las soluciones bsicas son an = 12 y bn = n 12 . La solucin general es:
 n
 n
1
1
+ 2 n
.
1
2
2
Al imponer las condiciones iniciales tenemos:

1
1



1 0
1 0
+

0
2
2
2 =
1
1
+

2 2 =1
2

1 = 0
1 21 + 2



1
2

=1

Luego 1 = 0 y 1 = 2. La solucin de la sucesin recurrente es:


 n
1
pn = 2n
.
2
n
n1
1
1
Para que este valor sea menor que 100
tiene que ser 2n 12 < 100
, luego 100n 12
< 1, esto
n1
n3
n3
8
4
4
es, 100n < 2
; 25n < 2
. Cuando n 11 tenemos 25n > 2
(275 > 2 = 2 2 = 256),
9
y para n = 12 resulta 300 < 2 = 512.
La solucin es para n 12 ya que la funcin 2n3 25n es una funcin estrictamente creciente.

28 de enero de 2013

Curso 20122013. NOTAS DE TRABAJO, 30

S EC . 11. E JEMPLOS

99

Ejemplo. 11.12.
Se considera b R+ y el determinante dn de la matriz

b b 0 0 0
b b b 0 0

0 b b b 0

0 0 b b b

An = .. .. .. .. .. . .
. . . . .
.

b
0

b
b
b

b
b

la diagonal, la diagonal superior y la diagonal inferior tienen todas sus entradas iguales a b.
Calcular el valor de dn .



b b 0
b b


S OLUCIN . Tenemos d1 = b, d2 = = 0 y d3 = b b b = b3 .
bb
0 b b

0
0

.
.
An1 ..
An ..
En general An+1 =
, luego dn+1 =| An | b | Bn | b, siendo Bn =
,

b
b
0 b b
0 0 b
entonces | Bn |=| An1 | b.
Resulta entonces dn+1 = |An |b |An1 |b2 = dn b dn1 b2 .
 
 
La ecuacin caracterstica es: r 2 = rb b2 . Las races son: = b 1+ 2 3 y = b 1 2 3 .
La solucin general es:
1 b

1+

n

+ 2 b

n

Al imponer las condiciones iniciales tenemos:


 
 

1 b 1+ 2 3 + 2 1 2 3 = b
 2
 2
1 b2 1+ 2 3 + 2 b2 1 2 3 = 0
 

1+ 3
1
+ 2 1 2 3 = b
2
 2
 2
1+ 3
1
+ 2 1 2 3 =
2


Cuya solucin es: 1 =

1+ 3
2 3

3 .
y 2 = 1
2 3

ARITMTICA Y COMBINATORIA
(I. Sucesiones recurrentes)

P. Jara

C AP. V. E JEMPLOS

100
El trmino general es:
dn =
=
=

 n
 n

1 3 n 1 3
1+ 3 n 1+ 3
b

b
2
2 3
2 3
 2
 n+1 
n+1
n
1+ 3
b
1 2 3
2
3
 
 n+1 
n+1
bn
(i)
1+i 3
1i2 3
2
3

1+i 3
3
1

o
=
cos

+
sin

con
cos

=
y
sin

=
2
2
2 , luego = 3 (= 60 ).
 n+1
 n+1
(n+1)
1+i 3
Entonces 1+i2 3
= (cos 3 +i sin 3 )n+1 = cos (n+1)
+i
sin
y
i
3
3
2
(n+1)
sin 3 .
 n+1
De la misma forma resulta i 1i2 3
+ sin (n+1)
.
= i cos (n+1)
3
3

Tenemos

Como consecuencia dn =

= i cos (n+1)

bn

2 sin (n+1)
.
3
3

Ejemplo. 11.13.
Resolver la sucesin recurrente
an = 2(an1 an2 ),

n2

con las condiciones iniciales a0 = 1, a1 = 2.


S OLUCIN . La ecuacin caracterstica es r 2 = 2(r 1). Las races son = 1 + i, = 1 i.
Las soluciones bsicas son: bn = (1 + i)n y cn = (1 i)n .
La solucin general es: 1 (1 + i)n + 2 (1 i)n . Al imponer las condiciones iniciales tenemos:
1 + 2 = 1
1 (1 + i) + 2 (1 i) = 2

1+i
1i
2 y 1 = 2 .
La solucin es: an = 21 ((1 i)(1 + i)n + (1 + i)((1 i)n )) = (1 + i)n1

n
Tenemos que 1 + i = 2(cos 4 + sin 4 ), luego (1 + i)n = 2 (cos n
4

La solucin es: 2 =

cuencia resulta

+ (1 i)n1 .
+ sin n
4 ). Como conse-

an = (1 + i)n1 + (1 i)n1
n1
n1
n
= 2
(cos (n1)
+
i
sin
)
+
2
(cos (n1)
i sin n
4
4
4
4 )
n+1
(n1)
= 2
cos 4 .

28 de enero de 2013

Curso 20122013. NOTAS DE TRABAJO, 30

S EC . 11. E JEMPLOS

101

Ejemplo. 11.14.
Resolver la sucesin recurrente
2an+3 = an+2 + 2an+1 an ,

n 0,

con las condiciones iniciales a0 = 0, a1 = 1, a2 =.

S OLUCIN . La ecuacin caracterstica es: 2r 3 = r 2 + 2r 1. Las races son = 21 , = 1,


= 1.
n
Las soluciones bsicas son: bn = 1, cn = (1)n y dn = 12 . La solucin general es: 1 +
n
2 (1)n + 3 12 .
Al imponer las condiciones iniciales tenemos:

1 + 2 + 3 = 0
1 2 + 12 3 = 1

1 + 2 + 14 3 = 2

De aqu se obtiene 1 = 52 , 2 61 , 3 = 83 .
La solucin es: an =

5
2

(1)n
6

8
3


1 n
2 .

Ejemplo. 11.15.
Encontrar una relacin de recurrencia para el nmero de sucesiones binarias de longitud n
que no tienen dos ceros consecutivos.

S OLUCIN . Llamamos an al nmero en cuestin. Es claro que a0 = 1, a1 = 2, a2 = 3 y a3 = 5,


pues las sucesiones de longitud tres son: 0,1,0; 0,1,1; 1,0,1; 1,1,0; 1,1,1.
En general dada una sucesin de n trminos, si acaba en 1, entonces los n 1 primeros trminos son arbitrarios, tenemos entonces an1 sucesiones. Si acaba en 0, entonces el trmino
que ocupa el lugar n1 es un 1, y los primeros n2 trminos son arbitrarios, tenemos entonces an2 . La frmula de recurrencia es: an = an1 + an2 , con la condiciones iniciales a0 = 1
y a1 = 2.
Nota. Ver la sucesin de Fibonacci.
ARITMTICA Y COMBINATORIA
(I. Sucesiones recurrentes)


P. Jara

C AP. V. E JEMPLOS

102

Ejemplo. 11.16.
Se considera un reactor al que en el momento cero se le inyecta un neutrn de alta energa.
En cada periodo de tiempo, por ejemplo un microsegundo (106 segundos) se produce el
siguiente proceso:
(1). Un neutrn se alta energa interacta con un ncleo (de material de fusin, por ejemplo
uranio) y en este proceso para el siguiente periodo se producen dos nuevos neutrones de
alta energa y uno de baja energa.
(2). Un neutrn de baja energa interacta con un ncleo, y en este proceso se produce un
electrn de alta energa y uno de baja energa.
Si suponemos una situacin ideal en la que cada neutrn libre interacta con un ncleo en
cada periodo, y si llamamos an al nmero de neutrones de alta energa y bn al nmero de
neutrones de baja energa en el periodo nsimo. Determinar una frmula general para an
y bn .
S OLUCIN . Los valores iniciales son a0 = 1, b0 = 0 y la regla de formacin es:

an+1 = 2an + bn
bn+1 = an + bn
P
P
Si llamamos f (x) = an xn y g(x) = bn xn , entonces tenemos:

an+1 xn+1 = 2an xn+1 + bn xn+1
bn+1 xn+1 = an xn+1 + bn xn+1

P
P
P
n+1 = 2x
n+x
n
a
x
a
x
b
x
n
n
n+1
P
P
P
bn+1 xn+1 = x an xn + x bn xn

f (x) a0 = 2xf (x) + xg(x)
g(x) b0 = xf (x) + xg(x)

f (x)(1 2x) = xg(x) + a0
xf (x) = g(x)(1 x) b0

f (x)(1 2x)x = x2 g(x) + xa0
f (x)(1 2x)x = g(x)(1 x)(1 2x) b0 (1 2x)
0 = g(x)(x2 (1 x)(1 2x)) + xa0 + (1 2x)b0
)
x
g(x) = x2 3x+1
f (x) =

g(x)(1x)
x

1x
x2 3x+1

5
=
x2 3x + 1 = 0, entonces x =
=
2
=

3+ 5
2
3 5
2

Valor de g(x).
x2
28 de enero de 2013

x
A
B
( x)A + ( x)B
=
+
=
3x + 1
x x
( x)( x)
Curso 20122013. NOTAS DE TRABAJO, 30

S EC . 11. E JEMPLOS

103
x = ( x)A + ( x)B = x(A + B) + B + A

A + B = 1
B + A = 0
B + (1 B) = 0;
B=
A=

B( ) = ;

)
(3 5)/2

5+3 5
=
=

10
5

53 5
1 B = 10

5+3 5 1
5 + 3 5 1
g(x) =
+
.
10
x
10
x
Valor de f (x).
1x
A
B
( x)A + ( x)B
=
+
=
x2 3x + 1
x x
( x)( x)
1 x = ( x)A + ( x)B = x(A + B) + B + A

A+B=1
B + A = 1

)
1(3
5)/2
1

= 510 5
B = =
5
5+ 5
10

A=1B=

5+ 5 1
5 5 1
f (x) =
+
.
10 x
10
x
Tenemos entonces
an =
=

5+ 5 (n+1)
5 5 (n+1)
5+ 5 n+1

=
10 
10
10

n+1
 n+1
5+ 5 3 5
+ 510 5 3+2 5
10
2

5 5 n+1
10

5 (n+1)
5 (n+1)
5 n+1
+ 5+3

= 5+3
+
bn = 5+3
10
10
10





n+1

n+1
5 3 5
5 3+ 5
= 5+3
+ 5+3
10
2
10
2

Nota. Se

tiene: 1

=y

1
x

1
1
1 x

1+


x 2

5+3 5 n+1

10

Ejemplo. 11.17.
Dado un conjunto de n elementos distintos y r 0, llamamos a(n, r) al nmero de formas
en que se pueden elegir r elementos (incluso con repeticin) de este conjunto de n elementos. Determinar una frmula para a(n, r).
S OLUCIN . Tenemos la frmula a(m, r) = a(n 1, r) + a(n, r 1), ya que fijado un elemento b1 del conjunto {b1 , . . . , bn }, el valor de a(n, r) se calcular teniendo en cuenta que b1 no
ARITMTICA Y COMBINATORIA
(I. Sucesiones recurrentes)

P. Jara

C AP. V. E JEMPLOS

104

intervenga en cuyo caso es a(n 1, r), o bien b1 interviene, lego el valor es a(n, r 1), y en
conjunto tenemos
a(n, r) = a(n 1, r) + a(n 1, r).
Construimos las funciones generatrices
fn (x) =

a(n, r)xr ,

r=0

con a(n, 0) = 1 y a(o, r) = 0 para cada n, r > 0.


La relacin anterior significa, para n 1,
X
X
X
fn (x) =
a(n, r)xr =
a(n 1, r)xr +
a(n, r 1)xr = fn1 (x) + xfn (x)
fn (x)(1 x) = fn1 (x)
fn1 (x)
.
fn (x) =
1x
1
n , y resulta
Por induccin tenemos fn (x) = (1x)
n = (1 x)
X n
X n + r 1
r r
fn (x) =
(1) x =
xr ,
r
r
r
r

luego la solucin es: a(n, r) = n+r1
.
r

Ejemplo. 11.18.
Se considera la recurrencia no homognea
an = 3an1 + n,
con la condicin inicial a0 = 1. Determinar una frmula para an .
S OLUCIN . Escribimos todos los casos desde n = 1, multiplicamos por xn y hacemos la
suma:

X
X
X
i
i
ai x = 3
ai1 x +
ixi .
i=1

Definimos la funcin f (x) =

i=1 ai

i=1

xi .

i=1

Se verifica:

f (x) a0 = 3xf (x) +

ixi

i=1

La funcin generatriz
1 + 2x

+ 3x3

+ , luego

P
i
i
i=0 ix =
i=1 ix es la funcin
P
x
i
=
i=1 ix , y obtenemos:
(1x)2
f (x) a0 = 3xf (x) +

28 de enero de 2013

generatriz de

d
dx

1
1x

1
(1x)2

x
,
(1 x)2

Curso 20122013. NOTAS DE TRABAJO, 30

S EC . 11. E JEMPLOS

105

de donde

x/(1 x)2 + a0
x
1
=
+
1 3x
(1 x)2 (1 3x) 1 3x
Para descomponer en fracciones simples.
f (x) =

x
A
B
A(1 x)(1 3x) + B(1 3x) + C(1 x)2
C
=
+
=
+
(1 x)2 (1 3x)
1 x (1 x)2 1 3x
(1 x)2 (1 3x)
x = A(1 x)(1 3x) + B(1 3x) + C(1 x)2 = (3A + C)x2 (4A + 3B + 2C)x + (A + B + C)

3A + C = 0

4A + 3B + 2C = 1

A+B+C =0
1/4

de donde A = 41 , B = 21 y C = 43 . Tenemos entonces f (x) = 1x


1/4
1x

1/2
(1x)2

3/4
13x

1
13x

1/2
(1x)2

7/4
. El coeficiente de xn es:
+ 13x




1 2+n1
7 n
1 1 n+1
7
1 n 7
1
2n
(1) + 3 =
+ 3n = + 3n ,
1
4
2
n
4
4 2
n
4
4 2 4

1
ya que 13x
= 1 + 3x + (3x)2 + (3x)3 + .
Nota. Hacer este ejercicio si utilizar funciones generatrices!

Ejemplo. 11.19.
Determinar cuntos subconjuntos de cuatro elementos del conjunto {1, 2, 3, . . . , 14, 15}
existen que no contienen dos enteros consecutivos.
S OLUCIN . Dado un subconjunto vlido, sea {a, b, c, d}, con a < b < c < d, tenemos que
a 1, b a, c b, d c, 15 d
son enteros y su suma es 14, siendo todos menores que 14.
Entonces cada subconjunto admisible est determinado por un quinteto a1 , a2 , a3 , a4 y a5 ,
verificando:

a1 + a2 + a3 + a4 + a5 = 14,
0 a1 , a2 , a3 , a4 , a5 ,

2 a2 , a3 , a4
La funcin que define esta situacin es:
f (x) = (1 + x + x2 + x3 + )2 (x2 + x3 + x4 + )3 = x6 (1 + x + x2 + x3 + ) = x6

1
.
(1 x)5

Tenemos que calcular el coeficiente de x14 en f (x), o equivalentemente el coeficiente de x8


1
en (1x)
5 . Como




X
X
1
5
5+r1 r
5
r
= (1 x) =
(x) =
x,
(1 x)5
r
r
r=0

ARITMTICA Y COMBINATORIA
(I. Sucesiones recurrentes)

r=0

P. Jara

C AP. V. E JEMPLOS

106

entonces tenemos que calcular



    
5+81
12
12
12 11 10 9
11 10 9
=
=
=
=
= 99 5 = 495.
6
6
4
432
2

Ejemplo. 11.20.
Determinar el coeficiente de x8 en
S OLUCIN . Descomponemos

1
.
(x3)(x2)2

1
(x3)(x2)2

en fracciones simples:

B
C
1
A
A(x 2)2 + B(x 2)(x 3) + C(x 3)
+
+
=
=
(x 3)(x 2)2
(x 3) (x 2) (x 2)2
(x 3)(x 2)2
1 = (A + B)x2 + (4A 5B + C)x + (4A + 6B 3C),

A+B=0

4A 5B + C = 0

4A + 6B 3C = 1
La solucin es: A = 1, B = 1 y C = 1. Tenemos entonces:
1
(x3)(x2)2

1
(x3)

1
(x2)

1
(x2)2

= 13

1
1 x3

= 13


x r
3

1
2

1 1
2 28


1 2 1
4 8 28

1 1
2 1 x2

1
1
4 (1 x2 )2


x r
2

1
4

2 x 2
2
2 x 
2 + 2
2
0 + 1

El coeficiente de x8 es:
13 318 +

= 319 +

1
29


1 2+81 1
4
8
28

= 319 +

1
29


1 9
210 8

= 319 +

1
29

1
9
210

= 319 +

29
210

= 319

7
210


Ejemplo. 11.21.
Comprobar que

2n
n

28 de enero de 2013

Pn

i=0

n2
i .

Curso 20122013. NOTAS DE TRABAJO, 30

S EC . 11. E JEMPLOS

107
n
n
n 
n2
+
+

+
+
,
0
1
n1
n

S OLUCIN . Tenemos (1 + x)2n = ((1 + x)n )2 =

entonces

2n
n n
n n 
n  n
n n
n = 0 n + 1 n1 + + n1 1 + n 0

Pn

i=0

n n 
i ni

Pn

i=0

n n
i
i

Pn

i=0

n2
i .


Ejemplo. 11.22.
De cuntas formas puede un maestro repartir 24 libros entre cuatro alumnos si cada alumno
debe tener al menos tres y no ms de ocho.
S OLUCIN . Para cada alumno la distribucin de libros es x3 + x4 + x5 + x6 + x7 + x8 y como
tenemos cuatro alumnos, la distribucin es: f (x) = (x3 + x4 + x5 + x6 + x7 + x8 )4 . Vamos a
calcular el coeficiente de x24 en f (x).
3

8 4

12

5 4

f (x) = (x + x + x + x + x + x ) = x (1 + x + x + x + x + x ) = x
Se trata entonces de determinar el coeficiente de x12 en la expansin de


1x6
1x

4

1
= (1 x6 )4 (1x)
4
h


= 1 41 x6 + 42 x12

4 18
3 x

+ x24

ih

12

1x6
1x

1 x6
1x

4

4
.

4
4
4
2
0 + 1 (x) + 2 (x)

Este coeficiente es:


1

4
12
12 (1)

4 4
6
1
6 (1)

4 4
2
0

4+121
4 4+61
4 4+01

+
12
1
6
2
0

15
4 9
4 3
12 1 6 + 2 0

15
9
3

4
+
6
3
3
0

= 151413
4 987
32
32 + 6 3 = 560 336 + 6 = 230.

Ejemplo. 11.23.
Determinar el coeficiente de x15 en la expansin de f (x) = (x2 + x3 + x4 + )4 .
S OLUCIN . Tenemos x2 + x3 + x4 + = x2 (1 + x + x2 + ) =
ARITMTICA Y COMBINATORIA
(I. Sucesiones recurrentes)

x2
1x .

P. Jara

C AP. V. E JEMPLOS

108
Tenemos f (x) =

x8
,
(1x)4

y por tanto basta determinar el coeficiente de x7 en la expansin de

1
.
(1x)4





X
X
1
4
4+i1 r
4
i
= (1 x) =
(x) =
x.
(1 x)4
i
i
i=0
i=0


Luego el coeficiente es: 4+71
= 10
7
7 = 120.

Ejemplo. 11.24.
De cuntas formas se puede seleccionar r objetos de un conjunto de n objetos distintos
(est permitida la repeticin).
S OLUCIN . Si solo tenemos un objeto las diferentes formas de seleccionar 0, 1, 2, 3, . . . objetos
estn dadas por la serie geomtrica 1 + x + x2 + x3 +
Si en vez de uno tenemos n objetos, las diferentes formas estarn dadas por la funcin
f (x) = (1 + x + x2 + x3 + )n .
Se trata de determinar el coeficiente de xr en f (x).




X
X
1
n
n+r1 r
n
r
f (x) =
= (1 x) =
(x) =
x,
r
r
(1 x)n
i=0

luego la solucin es:

n+r1
.
r

i=0

Ejercicio. 11.25.
Se considera la sucesin {an }n definida por a0 = 6, a1 = 22, a2 = 38 y an+3 = an+2 +
8an+1 12an para n 0. Determina el trmino general de la sucesin.
S OLUCIN . an = 2(3)n+1 + n2n+1

Ejercicio. 11.26.
En un taller de reparaciones de coches un operario se dice que est en el nivel n si le faltan n
etapas para llegar a reparar el vehculo en el que est trabajando. Sabemos que desde cada
nivel n hay dos formas de llegar al nivel n 1 y tres de llegar al nivel n 2. Si llamamos an
al nmero de formas en que se puede llegar a reparar un vehculo estando en el nivel n y si
a0 = 1. Da una frmula para calcular an si a1 = 2.
S OLUCIN . Observa que se tiene la relacin an = 2an1 + 3an+2 . Por tanto {an }n es una
sucesin recurrente con polinomio caracterstico r 2 2r 3. Las races de este polinomio son
1 y 3, entonces una solucin general es: an = 1 (1)n + 2 3n . Al imponer las condiciones
iniciales se tiene:

a0 = 1 = 1 + 2 ,
a1 = 2 = 1 + 32
28 de enero de 2013

Curso 20122013. NOTAS DE TRABAJO, 30

S EC . 11. E JEMPLOS
La solucin es 1 =
o frac343n

(1)n
4

109
1
4 y 2
3n+1
4 .

= 34 . El trmino general de la sucesin pedida es: an = 14 (1)n +




Ejercicio. 11.27.
Se supone que la facturacin de una empresa es cada ao la media entre la del ao anterior
y la del ao siguiente. Si las ventas en el ao 2000 fueron a0 = a y el ao 2001 fueron a1 = b.
Calcula las ventas en el ao 2000 + n.
S OLUCIN . Si llamamos an a las ventas en el ao 2000 + n se verifica:
an =

an1 + an+1
,
2

an+1 = 2an an1 .


Se trata de una sucesin recurrente cuyo polinomio caracterstico es r 2 2r + 1 que tiene raz
1 con multiplicidad dos. La expresin de la solucin general es: an = 1 1n + 2 n1n = 1 + n2 .
Al imponer las condiciones iniciales se tiene:

a0 = a = 1 ,
a1 = b = 1 + 2
La solucin es: 1 = a, 2 = b a. El trmino general de la sucesin es:
an = a + n(b a) = a(1 n) + bn.

Ejercicio. 11.28.
Un sistema luminoso emite tres tipos de seales, una de las cuales dura un segundo y las
otras dos dos segundos cada una. Halla el nmero de seales diferentes que se pueden emitir en n segundos suponiendo que las seales se emiten de forma continua.
S OLUCIN . Llamamos an al nmero de seales que se pueden emitir en n segundos. Observar que a1 = 1 y a2 = 3. Adems se tiene an+2 = an+1 + 2an para n 0. Tenemos pues
una sucesin recurrente con polinomio caracterstica r 2 r 2, cuyas races son: 1 y 2. La
solucin general es: an = 1 (1)n + 2 2n , que al imponer las condiciones iniciales resulta:

a1 = 1 = 1 + 22 ,
a2 = 3 = 1 + 42
La solucin es: 1 = 13 y 2 = 23 . El trmino general de la sucesin es: an = 13 (1)n + 32 2n =
(1)n+1
3

2n+1
3 .

ARITMTICA Y COMBINATORIA
(I. Sucesiones recurrentes)

P. Jara

110

28 de enero de 2013

C AP. V. E JEMPLOS

Curso 20122013. NOTAS DE TRABAJO, 30

Captulo VI
Miscelnea
12.

Nmeros naturales. El principio de induccin

Se considera N el conjunto de los nmeros naturales, esto es,


N = {0, 1, 2, 3, 4, 5, . . .}
Una propiedad P de nmeros naturales determina un subconjunto XP de N, el formado por
todos los nmeros naturales que verifican la propiedad P. Veamos un ejemplo, sea P la propiedad ser mltiplo de 2, en este caso se tenemos
XP = {0, 2, 4, 6, . . .}.
Este conjunto se representa por 2N. Si P es la propiedad ser un nmero impar, entonces
XP = {1, 3, 5, 7, . . .}.
Este conjunto es el complemento de 2N, esto es, N \ 2N.
Existe una frmula para la suma de los primeros nmeros naturales hasta n :
0 + 1 + + (n 1) + n =

n(n + 1)
.
2

Esta frmula podemos comprobarla para algunos nmeros. Veamos algunos ejemplos:
n 0 + 1 + + (n 2) + (n 1)
0
0
1
1
2
3
3
6
4
10

(n1)n
2

0
1
3
6
10

Pero no basta esta comprobacin para afirmar que la frmula es correcta, por esto hacemos
el siguiente razonamiento. Llamamos X al subconjunto de N formado por todos los nmeros
111

C AP. VI. M ISCELNEA

112

naturales para los que la frmula es cierta. Ya hemos comprobado que 0, 1, 2, 3, 4 X . Como
queremos ver que la frmula es cierta para todos los nmeros naturales debemos comprobar
que X = N. Para probar nos apoyamos en que los nmeros naturales verifican la propiedad
siguiente: Si un subconjunto S de N contiene a 0 y al siguiente de cada elemento de S, entonces S = N. Es el llamado Principio de Induccin.
Nos vamos a basar en esta propiedad para ver que la frmula es cierta para todos los nmeros
naturales. En efecto, ya hemos comprobado que 0 X , y supongamos que x X , esto es, que
se verifica
x(x + 1)
.
0 + 1 + 2 + + x =
2
Vamos a comprobar que tambin x + 1 X y tendremos X = N. Este es inmediato:
x
 (x + 1)(x + 2)
x(x + 1)
+ (x + 1) = (x + 1)
+1 =
.
0 + 1 + 2 + + x + (x + 1) =
2
2
2
Obtenemos por tanto
0 + 1 + 2 + + n =

n(n + 1)
, para cada n N.
2

Este mtodo se conoce como el mtodo de induccin y es muy til a la hora de probar propiedades de nmeros naturales.
Existen muchas otras frmulas de este tipo. Por ejemplo:
0 + 2 + 4 + + 2n = n(n + 1), para cada n N.
En este caso no es necesario hacer induccin, basta sacar el 2 factor comn a todos los sumandos.
Otra frmula til es:
1 + 3 + 5 + + (2n 1) = n2 , para cada n N.
En este caso basta considerar el siguiente desarrollo:
1+3+5+ +(2n1) = (0+1+2+ +2n)(0+2+4+ +2n) =

2n(2n + 1)
n(n+1) = n2 .
2

Ejercicio. 12.1.
Dados enteros positivos a0 , a1 , . . . , a100 verificando:

a1 > a0 ,

a2 = 3a1 2a0 ,
...

a100 = 3a99 2a98 .


Probar que a100 > 299 .
[The problems of the All-Soviet-Union mathematical competitions 1961-1986. Problem 015]
28 de enero de 2013

Curso 20122013. NOTAS DE TRABAJO, 30

S EC . 12. N MEROS NATURALES . E L PRINCIPIO DE INDUCCIN

113

S OLUCIN . [Ver tambin el Ejercicio (9.1.)] De la relacin a1 > a0 sabemos que existe h
N \ {0} tal que a1 = a0 + h.
De la relacin a2 = 3a1 2a0 se obtiene:
a2 = 3(a0 + h) 2a0 = a0 + 3h.
De esta forma tambin obtenemos:
a3 = 3a2 2a1 = 3(a0 + 3h) 2(a0 + h) = a0 + 7h
Es decir, para cada ndice n se tiene una expresin:
an = a0 + fn h,
siendo f0 = 0, f1 = 1, f2 = 3, f3 = 7, . . . .
La sucesin {fn }n sigue una ley de formacin, por ejemplo fn = 2n 1. Esta ley que se comprueba para los valores 0, 1, 2, 3, queremos probar que es cierta para todo nmero natural.
Vamos a probar este hecho por induccin sobre n. Llamamos X al subconjunto de N formado por los nmeros naturales x tales que fx = 2x 1. Ya hemos comprobado que 0, 1 X ;
supongamos que 0, 1, 2, . . . , x X , siendo x 2, y vamos a probar que tambin x + 1 X . en
efecto, se tiene:
ax+1 = 3ax 2ax1 = 3(a0 + (2x 1)h) 2(a0 + (2x1 1)h)
= a0 + (3(2x 1) 2(2x1 1))h = a0 + (3 2x 2 2x1 1)h
= a0 + (2x+1 1)h.
Como consecuencia fx+1 = 2x+1 1 y tambin x + 1 X . Entonces X = N.
Al estudiar el caso de a100 tenemos: a100 = a0 + (2100 1)h, y como a0 0 y h 1, resulta
que a100 > 299 .


ARITMTICA Y COMBINATORIA
(I. Sucesiones recurrentes)

P. Jara

C AP. VI. M ISCELNEA

114

13.

Progresiones aritmticas

Una sucesin de nmeros reales {an }n = {a0 , a1 , a2 , . . .} se llama una progresin aritmtica
si cada trmino an se obtiene del anterior sumando un nmero fijo d. El nmero d se llama
la diferencia de la progresin. Esto es, si an+1 = an + d para cada n 0. Observar que la
progresin aritmtica est dada por el primer trmino a0 y la diferencia, ya que para cada
ndice n se tiene an = a0 + nd. (La demostracin de este hecho se debe hacer por induccin.)
La descripcin an = a0 + nd, para cada n N se conoce como el trmino general de la
progresin.
La suma de los primeros trminos de una progresin aritmtica se calcula de forma sencilla
aplicando lo ya conocido sobre la suma de los primeros nmeros naturales.
a0 + a1 + + an = a0 + (a0 + d) + + (a0 + nd) = na0 + (0 + 1 + + n)
= na0 +

n(n+1)
2

2na0 +n(n+1)
2

(a0 +an )(n+1)


.
2

Observar que si queremos sumar trminos consecutivos de la progresin {an }n se tiene:


ar + ar+1 + + ar+s =

(a0 + ar+s )(r + s + 1) (a0 + ar1 )r


(ar + ar+s )(s + 1)

=
.
2
2
2

Esto es la suma del primero ms el ltimo por el nmero de trminos divido por 2.
Vamos a intentar sumar los cuadrados de los primeros nmeros naturales:
0 + 1 + 22 + + n2
Si llamamos sn a este valor, tenemos una sucesin {sn }n . Como no es una progresin aritmtica, tal y como anteriormente hemos definido, no sabemos como hacer esta suma. Vamos a
hacer algunas operaciones con los nmeros sn . Los escribimos en una columna y a la derecha
de cada uno escribimos la diferencia con el siguiente; esto nos forma una segunda columna,
formada por los cuadrados de los nmeros naturales. Repetimos el proceso para obtener una
tercera columna que, ahora s, es una progresin aritmtica. Si a esta columna le hacemos lo
mismo obtenemos una cuarta columna en la que todos los trminos con constantes. Decimos que la sucesin {sn }n es una progresin aritmtica de tercer grado, mientras que la
sucesin {n2 }n es una progresin aritmtica de segundo grado.
n
0
1
2
3
4
5

sn
0
1
5
14
30
55

sn
1
4
9
16
25

2 sn
3
5
7
9

3 sn
2
2
2

En general una sucesin {an }n forma una progresin aritmtica de grado t si al someterla a
este proceso la columna t tiene todos sus trminos iguales.
28 de enero de 2013

Curso 20122013. NOTAS DE TRABAJO, 30

S EC . 13. P ROGRESIONES ARITMTICAS

115

Podemos probar que el trmino general, an , de una progresin aritmtica de grado t se obtiene como
n  
X
n
an =
i a0 .
i
i=0

En donde hemos utilizado la siguiente notacin:


0 a0 es el trmino a0 ,
1 a0 es el trmino que ocupa el primer lugar en la segunda columna, y en general i A0
es el elemento que ocupa el primer lugar en la columna i + 1.
En el caso de la suma de cuadrados de los primeros nmeros naturales se tiene:





P
sn = ni=0 ni i s0 = n0 s0 + n1 1 s0 + n2 2 s0 + n3 3 s0
=10+n1+
=

n3 +n2 +n
6

n(n1)
2

3+

n(n1)(n2)
6

2=n+

n(n1)
2

3n(n1)(n2)
3

n(n+1)(2n+1)
6

Ejercicio. 13.1.
Determina la frmula para la suma de los cubos de primeros nmeros naturales.
S OLUCIN . Llamamos {sn }n a la sucesin definida sn = 0 + 1 + 23 + + n3 . Hacemos la
construccin en este caso:
n sn sn 2 sn 3 sn 4 sn
0 0
1
7
12
6
1 1
8
19
18
6
2 9
27
37
24
6
3 36 64
61
30
4 100 125 91
5 225 216
6 441
El trmino general es:
P
sn = ni=0

n i
i s0

=10+n1+
=n+

7n(n1)
2

n
0 s0

n(n1)
2

n 1
1 s0

7+

n 2
2 s0

n(n1)(n2)
6

+ 2n(n 1)(n 2) +

12 +

n 3
3 s0

n 4
4 s0

n(n1)(n2)(n3)
24

n(n1)(n2)(n3)
4

n(n+1)
2

2

.


ARITMTICA Y COMBINATORIA
(I. Sucesiones recurrentes)

P. Jara

C AP. VI. M ISCELNEA

116

Ejercicio. 13.2.
Demuestra que se verifica
0 + 1 + 24 + + n4 =

28 de enero de 2013

n(n + 1)(2n + 1)(3n2 + 3n 1)


.
30

Curso 20122013. NOTAS DE TRABAJO, 30

S EC . 14. P ROGRESIONES GEOMTRICAS

14.

117

Progresiones geomtricas

Una progresin geomtrica es una sucesin {an }n verificando an+1 = an r para cada n
0. Observa que una progresin geomtrica est determinada por los valores de a0 y de r.
Llamamos a r la razn de la progresin. El caso en que r = 0 nos da una sucesin trivial, por
lo en general no lo vamos a considerar. Observar que se tiene an = a0 r n para cada n 0.
Para progresiones geomtricas es de inters calcular la suma de los primeros trminos.
a0 + a1 + + an = a0 + a0 r + + a0 r n
= a0 (1 + r + + r n )
n+1

= a0 x x11 .

ARITMTICA Y COMBINATORIA
(I. Sucesiones recurrentes)

P. Jara

C AP. VI. M ISCELNEA

118

15.

Sucesiones recurrentes

Otro tipo de sucesiones de inters son las sucesiones definidas por recurrencia. Una sucesin
{an }n est definida por recurrencia si a partir de un ndice cada trmino se puede calcular,
mediante una frmula, a partir de los trminos anteriores. Los ejemplos que vamos a estudiar
son aquellos que son semejantes a la sucesin de Fibonacci. Recordar que esta sucesin est
definida mediante
a0 = 0,

a1 = 1,

an+2 = an+1 + an , para todo n 0.

Consideremos por tanto el caso general, esto es, una sucesin definida por:
a0 ,

a1 ,

an+2 = c1 an+1 + c2 an , para todo n 0.

Siendo c1 , c2 R. Como ya hemos comentamos todas las sucesiones que verifican esta relacin forman un espacio vectorial, por lo tanto para estudiarlas todas vamos a dar una base del
mismo. No viene al caso, pero conviene sealar que la dimensin del espacio vectorial para
la sucesin de Fibonacci es igual a dos, y por tanto una base est formada por dos elementos
(dos sucesiones).
El mtodo de clculo consiste en ver qu progresiones geomtricas verifican la relacin. Consideremos una progresin geomtrica an = a0 r n (podemos suponer que a0 6= 0 y r 6= 0, ya
que en otro caso estaramos ante una sucesin trivial). Se tiene entonces las siguientes relaciones que se pueden simplificar en la forma obvia.
a0 r n+2 = c1 a0 r n+1 + c2 r n ,

r 2 = c1 r + c2 (ecuacin caracterstica).

Por tanto r es raz del polinomio X 2 c1 X c2 (polinomio caracterstico). Si es una raz de


este polinomio (puede ser un nmero complejo), entonces la sucesin {1, , 2 , . . .} verifica
la relacin an+2 = c1 an+1 + c2 an .
Si el polinomio tiene dos races distintas, y , entonces tenemos dos sucesiones distintas
{1, , 2 , . . .} y {1, , 2 , . . .} que satisfacen la relacin, y por tanto stas forman una base del
espacio vectorial.
Si es una raz doble del polinomio, como es tambin raz de la derivada del polinomio, se
tiene:
2 c1 = 0.
Multiplicando por se tiene:
22 c1 = 0.
En este caso la sucesin {0, , 22 , 33 , . . .} verifica la relacin an+2 = c1 an+1 +c2 an . En efecto,
si n 0 tenemos
c1 (n + 1)n+1 + c2 nn = nc1 n+1 + nc2 n + c1 n+1 = nn+2 + 22 n = (n + 2)n+2 .
La base est formada por {1, , 2 , . . .} y {0, , 22 , 33 , . . .}.
28 de enero de 2013

Curso 20122013. NOTAS DE TRABAJO, 30

S EC . 15. S UCESIONES RECURRENTES

119

En este caso cualquier sucesin genrica que verifica la relacin ser una combinacin de
estas dos sucesiones, esto es, es de la forma d1 n +d2 nn , siendo d1 , d2 R. Ahora los valores
iniciales a0 y a1 nos permiten calcular los coeficientes d1 y d2 .
Ejemplo. 15.1.
Determinar el trmino general de una sucesin {an }n que verifica las condiciones:
a0 = 1,

a1 = 6,

an+2 = 4an+1 4an , para todo n 0.

S OLUCIN . En este caso c1 = 4 y c2 = 4 y el polinomio caracterstico es: X 2 4X + 4. Este


polinomio tiene una nica raz, = 2, con multiplicidad dos. Por lo tanto las sucesiones que
forman la base son:
{1, , 2 , . . .} = {1, 2, 22 , . . .} y
{0, , 22 , 33 , . . .} = {0, 2, 8, 24, . . .}.
Una sucesin genrica tiene trmino general igual a:
d1 n + d2 nn .
Al verificar las condiciones iniciales resulta:
d1 0 + d2 0 0 = 1
d1 1 + d2 1 1 = 6

d1 = 1
2d1 + 2d2 = 6

La nica solucin es: d1 = 1, d2 = 2. la sucesin {an }n tiene trmino general igual a


an = n + 2nn = (1 + 2n)n = (1 + 2n)2n .
Comprobar que esta sucesin verifica las condiciones!

Vamos a hacer un ejemplo del caso en que las races del polinomio caracterstico son distintas.
Ejemplo. 15.2.
Determinar el trmino general de una sucesin {an }n que verifica las condiciones:
a0 = 1,

a1 = 5,

an+2 = an+1 + 2an , para todo n 0.

S OLUCIN . En este caso c1 = 1 y c2 = 2 y el polinomio caracterstico es: X 2 X 2. Este


polinomio tiene dos races distintas, = 1, = 2. Por lo tanto las sucesiones que forman la
base son:
{1, , 2 , . . .} = {1, 1, 1, 1, . . .} y
{1, , 2 , 3 , . . .} = {1, 2, 22 , 23 , . . .}.
ARITMTICA Y COMBINATORIA
(I. Sucesiones recurrentes)

P. Jara

C AP. VI. M ISCELNEA

120
Una sucesin genrica tiene trmino general igual a:
d1 n + d2 n .
Al verificar las condiciones iniciales resulta:
d1 0 + d2 0 = 1
d1 1 + d2 1 = 5

d1 + d2 = 1
(1)d1 + 2d2 = 5

La nica solucin es: d1 = 1, d2 = 2. la sucesin {an }n tiene trmino general igual a


an = n + 2 n = (1)n + 2 2n = (1)n+1 + 2n+1 .
Comprobar que esta sucesin verifica las condiciones!

28 de enero de 2013

Curso 20122013. NOTAS DE TRABAJO, 30

S EC . 16. E JERCICIOS RESUELTOS . S ELECCIN

16.

121

Ejercicios resueltos. Seleccin

Ejercicio (9.4.)
Ejercicio (9.5.)
La parte (1) del siguiente ejercicio no la hemos estudiado.
Ejercicio (9.6.)
Ejercicio (9.8.)
Ejercicio (9.13.)
Ejercicio (11.25.)
Ejercicio (11.26.)
Ejercicio (11.27.)
Ejercicio (11.28.)

ARITMTICA Y COMBINATORIA
(I. Sucesiones recurrentes)

P. Jara

C AP. VI. M ISCELNEA

122

17.

Recurrencia en combinatoria

El nmero de subconjuntos de r elementos de un conjunto de n elementos se representa por


n
r , y se calcula como
 
n
n(n 1) (n r + 1)
n!
=
=
.
r
r(r 1) 1
r!(n r)!
Consecuencia de la definicin son las siguientes propiedades:
Lema. 17.1.
Para n y r nmeros enteros se tiene:


(1) n0 = 1 = nn .

n 
(2) Si n > r, entonces nr = nr
.

(3) nr = 0 si r > n.
Ademsse tiene la siguiente propiedad que permitir dar un mtodo recurrente para el clculo de nr .
Proposicin. 17.2.
Para cada n, r N, se tiene


   

n+1
n
n
=
+
.
r
r
r1

D EMOSTRACIN . Dado un conjunto X = {x0 , x1 , . . . , xn } con n + 1 elementos y r N, si


r > n + 1, entones en la expresin del enunciado
todos los elementos son iguales a cero y se

tiene la relacin. Si r = n+1, entonces nr = 0 y se tiene la igualdad 1 = 1, que evidentemente
es cierta. Si r < n + 1 al formar un subconjunto de Xcon r elementos, si este subconjunto
no contiene a xn , entonces podemos formarlo de nr formas distintas, y si contiene a xn ,
entonces
el subconjunto formado por los r 1 elementos restantes puede construirse de
n 
r1 formas distintas. en consecuencia se tiene


para cualesquiera n, r N.
28 de enero de 2013

   

n+1
n
n
=
+
.
r
r
r1

Curso 20122013. NOTAS DE TRABAJO, 30

S EC . 17. R ECURRENCIA EN COMBINATORIA

123

Fruto de este resultado se tiene el llamado tringulo de Tartaglia:


0
0
1
1
0
1
2
2
2
0
1
2
3
3
3
3
0
1
2
3




4
4
4
4
4

0
1
2
3
4
5
5
5
5
5
5
0

1
1
1

1
3
1
1
4
6 4
1
1
5
10
10
5
1
1
6
15
20
15
6
1
1
7
21
35
35
21
7
1
1 8
28
56
70
56
28
8 1
1 9 36
84
126
126
84
36 9 1
1 10 45 120 210
252
210 120 45 10 1
1

La primera aplicacin del tringulo de Tartaglia es el desarrollo del binomio, ya que la fila
nsima tiene los coeficientes del desarrollo del binomio (a + b)n .
 
 
 


 
n n
n n1
n n2 2
n
n n
n
n1
(a + b) =
a +
a
b+
a
b + +
ab
+
b .
0
1
2
n1
n
Podemos considerar lo anterior como un desarrollo en serie de la siguiente forma:
(1 + x)n = 1 + nx +

n(n 1)x2 n(n 1)(n 2)x3


+
+
2!
3!

Si n = 1, 2, 3, . . ., tenemos un valor exacto de esta expansin. En cambio si tomamos n = 1/2


el valor que se obtiene:
1
1
1 1 x2 1 1 3 x3
(1 + x) 2 = 1 + x +
+
+ ,
2
2 2 2! 2 2 2 3!

1
que da una aproximacin al valor (1 + x) 2 . Para calcular 2 procedemos como sigue:
1

(1 + 1) 2 = 1 +

1 1
1
+
+
2 8 16
2

En este caso no tenemos problemas, y la razn es que la serie 1+nx+ n(n1)x


+ n(n1)(n2)x
+
2!
3!
converge cuando x 1. por esta razn no podemos aplicarla para aproximar por ejemplo
ARITMTICA Y COMBINATORIA
(I. Sucesiones recurrentes)

P. Jara

C AP. VI. M ISCELNEA

124

35, pues el desarrollo de la serie (1+34) 2 no converge. Sin embargo, si se toma un cuadrado
perfecto anterior a 35, en este caso 25, podemos escribir

35 = (25 + 10) 2 = 5(1 +

10 1
)2 =
25

Para el clculo de la raz cbica procedemos de la misma forma. En este caso tenemos que
calcular el desarrollo de
1
1
1 2 x2 1 2 5 x3
+
+
(1 + x) 3 = 1 + x +
3
3 3 2! 3 3 3 3!

Para el calculo de la raz cbica de 35, 3 35 procedemos como sigue:

1
8 1
3
35 = (27 + 8) 3 = 3(1 + ) 3 =
27
en donde ahora hemos considerado un cubo menor que 35.
Problema. 17.3.
Determinar el nmero de soluciones enteras positivas de la ecuacin
x1 + + xm = n,
en donde n, m N.
La solucin a este problema nos da tambin el nmero de combinaciones con repeticin de
m elementos tomados de n en n.
S OLUCIN . Llamamos Sn,m al nmero de soluciones distintas de la ecuacin anterior, en
donde n, m N. Observa que:
(1) S0,m = 0 para cada m N.
(2) Sn,0 = 0 para cada n N \ {0}.
(3) Sn,1 = 1 para cada n N \ {0}.
Adems, dados n, m N, el nmero de soluciones de x1 + + xm+1 = n + 1 con xm+1 = 1
es exactamente Sn,m , salvo en el caso en que n = 0 = m que vale 1. Y si se tiene una solucin
con xm+1 > 1, se tiene una solucin de la ecuacin y1 + + ym+1 = n tomando yi = xi
si i = 1, 2, . . . , m e ym+1 = xm+1 1, y viceversa; por lo tanto el nmero de soluciones con
xm+1 > 1 es Sn,m+1 . En consecuencia se tiene
Sn+1,m+1 = Sn,m + Sn,m+1 .
28 de enero de 2013

Curso 20122013. NOTAS DE TRABAJO, 30

S EC . 17. R ECURRENCIA EN COMBINATORIA

125

en donde n, m N no son ambos iguales a 0, y S1,1 = 1. Todos estos valores podemos escribirlos en la siguiente tabla:
S0,0
S1,0
S2,0

S0,1
S1,1

S2,1

S0,2
S1,2

S2,2

S1,3
S2,3

S0,3
S2,4

S3,2
S3,3
S3,4

S4,0
S4,1
S4,2 S4,3
S4,4
S4,5

S5,0
S5,1
S5,2
S5,3
S5,4
S5,5

S3,0

S3,1

0 0 0

0 1 0 0
0 1 1 0 0
0 1 2 1 0
0 1 33 1 0
0 1 4 6 4 1
0

Los puntos en color azul tienen el valor cero, y los puntos an color rojo o negro reproducen
el tringulo de Tartaglia.
n
En efecto, vamos a probar que Sn+1,m+1 = m
para todo n, m N. Hacemos induccin sobre

n y m. Si n = 0 = m, entonces se tiene S1,1 = 1 = 00 , y el resultado es cierto.

El resultado es cierto para todos los pares (n, 0), ya que Sn+1,1 = 1 = n0 . Vamos a suponer
que el resultado es cierto para todos los pares (n, m) siendon t, y sea cualquiera
el valor
n1
n1
n
de m. Tenemos entonces Sn+1,m+1 = Sn,m + Sn,m+1 = m1 + m = m . Por lo tanto el
resultado es cierto para todos los pares (n, m) N2 .
En consecuencia el nmero de soluciones enteras positivas de la ecuacin x1 + + xm = n
es igual a


n1
Sn,m =
,
m1
si n, m 1 y Sn,m = 0 si n = 0 o m = 0.

Problema. 17.4.
Determinar el nmero de soluciones enteras no negativas de la ecuacin
x1 + + xm = n,
en donde n, m N.
S OLUCIN . Llamamos Sn,m al nmero de soluciones distintas de la ecuacin anterior, con
n, m N. Se tienen los siguientes valores:
(1) Sn,0 = 0 para cada n N.
(2) S0,m = 1 para cada m N \ {0}.
ARITMTICA Y COMBINATORIA
(I. Sucesiones recurrentes)

P. Jara

C AP. VI. M ISCELNEA

126

Dados n, m N consideramos la ecuacin x1 + + xm+1 = n + 1. El nmero de soluciones


con xm+1 = 0 es n+1,m , salvo en el caso en que m = 0 que vale 1. Y si se tiene una solucin
con xm+1 6= 0, entonces tenemos una solucin de la ecuacin y1 + + ym+1 = n tomando
yi = xi si i = 1, . . . , m e ym+1 = xm+1 1, y viceversa; por lo tanto el nmero de soluciones
con xm+1 6= 0 es Sn,m+1 . En consecuencia se tiene
Sn+1,m+1 = Sn,m + Sn,m+1 ,
para cualesquiera n N y m N \ {0}, y Sn,1 = 1 para todo n N.
Agrupamos estos valores en una tabla
S0,0
S1,0
S2,0
S3,0
S4,0

S0,1 S0,2
S1,1 S1,2
S2,1 S2,2
S3,1 y S3,2
S4,1 S4,2
0
0
0
0
0

1 1
1 2
1 3
1 y 4
1 5

S0,3
S1,3
S2,3
S3,3
S4,3
1
3
6
10
15

S0,4
S1,4
S2,4
S3,4
S4,4

1
4
10
20
35

Se trata de calcular el valor exacto de cada Sn,m . Observa que una posible hiptesis es:

 

n+m1
n+m1
Sn,m =
=
,
m1
n
si m 1, y Sn,0 = 0 en otro caso. Consideramos el caso de Sn,m con m 6= 0. Vamos a calcular

S0,m ; sabemos que S0,m = 1 cuando m 6= 0, y se tiene m1
el resultado
m1 = 1; por lo tanto
n
es cierto para estos valores. Para cualquier valor de n se tiene Sn,1 = 1 = n , y por tanto el
resultado es cierto para estos valores.
Supongamos que el resultado es cierto para todos los pare (t, s), s 6= 0, con t + s menor que
un cierto h > 0; vamos a ver que es cierto para todos los pares (n, m), m 6= 0. Se tiene St+1,s =


t+s 
St,s + St+1,s1 = t+s1
+ t+s1
= s1
, y de la misma forma St,s+1 = St1,s+1 + St,s =
s1
s2
t+s1
t+s1
t+s
+ s1 = s .
s
Como consecuencia se tiene


n+m1
Sn,m =
,
m1
para n N, m N \ {0} y Sn,0 = 0 para n N.

Igualdades con nmeros combinatorios.


Lema.
17.5.
Pn
n+1
=
j=0
k+1

j
.
k

28 de enero de 2013

Curso 20122013. NOTAS DE TRABAJO, 30

S EC . 17. R ECURRENCIA EN COMBINATORIA

127

Esta igualdad expresa que en el tringulo de Pascal, el que aparece ms abajo, cada elemento
se consigue como la suma de los elementos de la columna anterior.
0
1
2
3
4
0
1
2
3
4
1
2
3
4
5
0
1
2
3
4
2
3
4
5
6
0
1
2
3
4
3
4
5
6
7
0
1
2
3
4
4
5
6
7
8
0
1
2 3
4
5
6
7
8
9
0
1
2
3
4
6
7
8
9
10
0
1
2
3
4
1
1
1
1
1
1
1
1
1
1

1
2
3
4
5
6
7
8
9
10

1
3
6
10
15
21
28
36
45
55

1
4
10
20
35
56
84
120
165
220

1
5
15
35
70
126
210
330
495
715

1
6
21
56
126
252
462
792
1287
2002

1
7
28
84
210
462
924
1716
3003
5005

1
8
36
120
330
792
1716
3432
6435
11440

1
9
45
165
495
1287
3003
6435
12870
24310

1
10
55
220
715
2002
5005
11440
24310
48620

D EMOSTRACIN . Se parte de la formula de la adicin, Proposicin (17.2.), y se procede por


recurrencia:
n+1
n 
n
=
+
k+1
k+1 
k 
n1
n
= n1
+
+
k+1
k
k
n2
n1
n
= n2
+
+
+
k+1
k
k
 k

0
n
= k+1
+ k0 + k1 + + n1
+
k
k



n
= k0 + k1 + + n1
+
k
k

De forma semejante, trabajando ahora por filas en vez de columnas, se tiene la frmula:
Lema. 17.6.
P
n+k+1
= kj=0
k

n+j
j .
0
0
1
0
2
0
3
0
4
0
5
0
6
0

ARITMTICA Y COMBINATORIA
(I. Sucesiones recurrentes)

1
1
2
1
3
1
4
1
5
1
6
1
7
1

2
2
3
2
4
2
5
2
6
2
7
2
8
2

3
3
4
3
5
3
6
3
7
3 
8
3
9
3

4
4
5
4
6
4
7
4
8
4
9
4
10
4

P. Jara

C AP. VI. M ISCELNEA

128
Otras identidades de inters son:
Lema.
17.7. 
n  m
n
m k = k

nk 
mk .

D EMOSTRACIN .
n  m
n!
m!
m
k = m!(nm)! k!(mk)!
(nk)!
n!
= k!(nk)!
(nm)!(mk)!
 nk 
= nk mk
.


Lema.
17.8.
P
r
k=0 m+k

s 
nk

r+s 
m+n .

D EMOSTRACIN . Comparar los desarrollos de (X + Y )r (X + Y )s y (X + Y )r+s ; los trminos de


la igualdad del enunciado son los coeficientes de X r+s Y r+s(m+n) en cada uno de los desarrollos.


Tringulo de Sierpinski
El tringulo de Sierpinski. Consideramos el tringulo de Tartaglia y reducimos cada entrada mdulo 2; de esta forma si el valor de una entrada es par aparecer un 0 y si es impar
aparecer un 1.
1
1 1
1 0 1
1 1 1 1
1 0 0 0 1
1 1 0 0 1 1
1 0 1 0 1 0 1
1 1 1 1 1 1 1 1
1 0 0 0 0 0 0 0 1
1 1 0 0 0 0 0 0 1 1
1 0 1 0 0 0 0 0 1 0 1
1 1 1 1 0 0 0 0 1 1 1 1
1 0 0 0 1 0 0 0 1 0 0 0 1
1 1 0 0 1 1 0 0 1 1 0 0 1 1
1 0 1 0 1 0 1 0 1 0 1 0 1 0 1
1 1 1 1 1 1 1 1 1 1 1 1 1 1 1 1
28 de enero de 2013

Curso 20122013. NOTAS DE TRABAJO, 30

S EC . 17. R ECURRENCIA EN COMBINATORIA

129
H
H

H
H
H
H
H
H
H
H
H
H
H
H
H
H

H
O

H
O

H
H

O
H

H
H

H
O

H
O

H
O

H
H

H
O

O
H

H
H

O
O

H
H

H
O

O
H

H
H

H
O

H
H

H
O

H
O

H
O

O
H

O
O

H
O

H
H

H
H

H
O

H
H

Problema. 17.9.
Se considera t N \ {0}. Determina la mayor potencia de 2 que divide a 2t !
S OLUCIN . Primero comprobamos que
2 | 21 ,

23 | 22 !,

27 | 23 !
t

Vamos a probar que la mayor potencia de 2 que divide a 2t ! es 22 1 . Tomamos como hiptesis
s
de induccin que para cada s t la mayor potencia de 2 que divide a 2s ! es 22 1 . Vamos a
estudiar 2t+1 ! Consideramos la fraccin
2t+1 !
= (2t + 1)(2t + 2) (2t + 2t ).
2t !
Prescindiendo de los impares tenemos que estudiar la lista
2t + 2, 2t + 4, . . . , 2t + 2t .
Salvo el caso de 2t + 2t = 2t+1 que es divisible por 2t+1 , todos los dems son divisibles como
mximo por 2t , y en cada caso para estudiar la potencia de 2 que divide a 2t + x basta estudiar
la divisibilidad de x; observa que en el caso de 2t+1 tendremos que aadir un factor 2 extra al
final.
Estudiamos la lista
2, 4, . . . , 2t
esta lista tiene 2t1 elementos. Dividimos por 2 cada elemento y obtenemos
1, 2, . . . , 2t1 .
ARITMTICA Y COMBINATORIA
(I. Sucesiones recurrentes)

P. Jara

C AP. VI. M ISCELNEA

130

El nmero de potencias de 2 que dividen a estos elementos es el exponente de la mayor potencia de 2 que divide a 2t1 !, y por hiptesis es 2t1 1. As pues la mayor potencia de 2 que
t+1
t1
t1
t
divide a 22t ! ! es 22 22 1 2 = 22 .
Reuniendo los resultados anteriores se tiene que la mayor potencia de 2 que divide a 2t+1 ! es
t

t+1

22 1 22 = 22

.


Problema. 17.10.
Se considera t N \ {0}. Demuestra que
S OLUCIN . Tenemos

2t 1
i

es impar para cada 0 i 2t 1.

 t

2 1
(2t 1)(2t 2) (2t i)
=
i(i 1) 2
i

Podemos prescindir de los factores impares, y si 2h i es el mayor par menor que i, entonces
tendremos que estudiar la fraccin
(2t 2)(2t 4) (2t 2h)
2h 2
Como solo nos interesan las potencias de 2, las potencias que dividen al numerador y al de
t
nominador son las mismas, por tanto 2 1
es impar.

i
Problema. 17.11.
Se considera el tringulo de Tartaglia. Determina cuanto nmeros pares e impares hay en la

lnea 51 (corresponde a 51
i ).
S OLUCIN . En el tringulo de Tartaglia reducido mdulo 2 la fila 2t est formada solo por
unos, ya que todos elementos de la misma en el tringulo de Tartaglia son impares. En la fila
2t + 1 se inician dos nuevos tringulos, en las posiciones primera y ltima, con 2t filas que
son iguales al tringulo que se inicia en la posicin inicial. Como consecuencia el nmero de
elementos impares en la fila 2t + h es el doble que el de la fila h, siendo 1 h 2t .
Como consecuencia, como 51 = 32 + 19, el nmero de elementos impares es el doble que el
nmero de elementos de la fila 19. Ahora, como 19 = 16+3, el nmero de elementos impares
en la fila 19 es el doble que el de la fila 3. Como la fila 3 tiene 2 elementos impares, la fila 19
tiene 2 2 y la fila 51 tiene 2 2 2 = 8.


28 de enero de 2013

Curso 20122013. NOTAS DE TRABAJO, 30

S EC . 18. A MPLIACIN DE NMEROS COMBINATORIOS

18.

131

Ampliacin de nmeros combinatorios

Nmeros de Stirling de segunda especie





n
El nmero
indica el nmero de formas en que un conjunto de n elementos puede
m
descomponerse en particiones
de m conjuntos se llama un nmero de Stirling se segunda
 
4
especie. Por ejemplo
indica el nmero de particiones de un conjunto de 4 elementos
2
que constan de exactamente dos conjuntos. Si X = {a, b, c, d}, las posibles particiones son:

{a, b, c}
{a, b, d}
{a, c, d}
{b, c, d}

{d};
{c};
{b};
{a};

{a, b}
{a, c}
{a, d}

{c, b}
{b, d}
{b, c}

 
4
y por lo tanto
= 7.
2
Vamos a determinar
una

 forma recursiva para los nmeros de Stirling de segunda especie.
n+1
Para calcular
consideramos el conjunto X = {x0 , . . . , xn }, a partir del conjunto
m+1


n
{x0 , . . . , xn1 } construimos
, ahora el elemento xn podemos colocarlo en cada uno
m+1
de los conjuntos que forman cada particin, obteniendo as particiones distintas con m elementos. Faltan las particionesen m
 + 1 conjuntos, uno de los cuales es {xn }, pero stas san
bemos que son exactamente
. Como consecuencia el nmero de Stirling de segunda
m
especie verifica la relacin:

n+1
m+1

n
= (m + 1)
m+1


+


n
.
m

Los nmeros de Stirling de segunda especie podemos agruparlos en la siguiente tabla:


ARITMTICA Y COMBINATORIA
(I. Sucesiones recurrentes)

P. Jara

C AP. VI. M ISCELNEA

132

 
0
=1
0
1
=0
0
2
=0
0
3
=0
0
4
=0
0
5
=0
0
6
=0
0
7
=0
0

 
1
=1
1
2
=1
1
3
=1
1
4
=1
1
5
=1
1
6
=1
1
7
=1
1

 
2
=1
2
3
=3
2
4
=7
 2
5
= 15
2
6
= 31
2
7
= 63
2

 
3
=1
3
4
=6
 3
5
= 25
 3
6
= 90
3 
7
= 301
3

 
4
=1
 4
5
= 10
4
6
= 65
 4
7
= 350
4

 
5
=1
 5
6
= 15
 5
7
= 140
5

 
6
=1
 6
7
= 21
6

Nmeros de Stirling de primera especie





n
El nmero de Stirling de primera especie
es el nmero de formas en que podemos desm
componer un subconjunto de n elementos en m conjuntos mutuamente disjuntos considerando las ordenaciones cclicas de cada uno de estos conjuntos. Por ejemplo, si consideramos
X = {a, c, b, d}, entonces tenemos las siguientes descomposiciones:
 
4
= 6.
1
[a, b, c, d]

[a, b, c, d]

[a, c, b, d]

[a, d, c, b]

[a, c, d, b]

[a, d, b, c].

 
4
= 11.
2
[a, b, c], [d]
[a, c, d], [b]

[a, c, b], [d]


[a, d, c], [b]
[a, b], [c, d]

[a, b, d], [c]


[b, c, d], [d]
[a, c], [b, d]

[a, d, b], [c]


[b, d, c], [a]
[a, d], [b, c].

 
4
= 6.
3
[a, b], [c], [d]
28 de enero de 2013

[a, c], [b], [d]

[a, d], [b], [c]

[b, c], [a], [d]

[b, d], [a], [c]

[c, d], [a], [b].

Curso 20122013. NOTAS DE TRABAJO, 30

S EC . 18. A MPLIACIN DE NMEROS COMBINATORIOS

133

Nmeros trinomiales
n
Los nmeros combinatorios m
aparecen el desarrollar el binomio (a + b)n segn la Frmula
de Newton. Si se desarrolla el trinomio (a + b + c)n se obtiene una expresin similar:
n

(a + b + c) =

n
X

n
X

i=0 j=0,i+jn

Los nmeros

n
i,j


n
ai bj cnij
i, j

se llaman nmeros trinomiales.

Problema. 18.1.
Determina el valor de

n
i,j .

n
n 
Por comodidad el trinomio i,j
lo representamos por i,j,k
, siendo i + j + k = n. Observa que
ahora la expresin del trinomio es:


n
n!
.
=
i, j, k
i!j!k!
n 
El nmero i,j,k
representa el nmero de particiones ordenadas de un conjunto de n elementos en tres conjuntos con i, j y k elementos.
3 
3!
Por ejemplo, si X = {a, b, c}, entonces 1,1,1
= 6, que corresponde a las particiones:
= 1!1!1!

{{a}, {b}, {c}} ,

En cambio

3 
2,1,0

{{a}, {c}, {b}} ,

3!
2!1!

3
1,0,2

{{c}, {a}, {b}} ,

{{c}, {b}, {a}} .

= 3 corresponde a las particiones:

{{a, b}, {c}, } ,




{{b}, {a}, {c}} ,


{{b}, {c}, {a}} ,

{{a, c}, {b}, } ,

{{b, c}, {a}, } ,

= 3 corresponde a las particiones:


{{c}, , {a, b}} ,

{{b}, , {a, c}} ,

{{a}, , {b, c}} .

De forma similar el tringulo de Tartaglia, se puede construir, en este caso una pirmide, en
la que aparezcan todos los trinomios. el primer nivel de la pirmide es:

0 
0,0,0

El segundo nivel es:


1 
1,0,0

1 
0,0,1

1 
0,1,0

ARITMTICA Y COMBINATORIA
(I. Sucesiones recurrentes)

P. Jara

C AP. VI. M ISCELNEA

134
El tercer nivel es:

2 
2,0,0

2 
1,0,1

2 
1,1,0

2 
0,0,2

2 
0,1,1

2 
0,2,0

El cuarto nivel es:

3 
3,0,0

3 
2,0,1

3 
2,1,0

3 
1,0,2

3 
1,1,1

3 
1,2,0

3 
0,0,3

3 
0,1,2

3 
0,2,1

3 
0,3,0

El quinto nivel es:

4 
4,0,0

4 
3,0,1

4 
3,1,0

4 
2,0,2

4 
2,1,1

4 
2,2,0

4 
1,0,3

4 
1,1,2

4 
1,2,1

4 
1,3,0

4 
0,0,4

4 
0,1,3

4 
0,2,2

4 
0,3,1

4 
0,4,0

El sexto nivel es:

5 
5,0,0

5 
4,0,1

5 
4,1,0

5 
3,0,2

5 
3,1,1

5 
3,2,0

5 
2,1,2

5 
2,2,1

5 
2,3,0

5 
2,0,3

5 
1,4,0

5 
1,1,3

5 
1,2,2

5 
1,3,1

5 
1,0,4

5 
0,0,5

5 
0,1,4

5 
0,2,3

5 
0,3,2

5 
0,4,1

5 
0,5,0

Observa que la regla de formacin es: Los puntos en azul se completan con la ley

 
 

n+1
n
n
=
+
i, j, k
i 1, j, k
i, j 1, k
en sus tres versiones (observa que as se pasa de un nivel al siguiente), y los puntos en rojo
se completan con la ley

 
 

n+1
n+1
n
=
+
.
i, j, k
i + 1, j 1, k
i 1, j + 1, k
Problema. 18.2.
Prueba las leyes enunciadas en el prrafo anterior.
Ejercicio. 18.3.
Completa el sptimo nivel.

28 de enero de 2013

Curso 20122013. NOTAS DE TRABAJO, 30

Bibliografa
[1] S. Lang, Algebra 3rd. ed., Springer, 2002.

ndice alfabtico
combinaciones con repeticin, 124
condiciones iniciales, 20
diferencia, 6, 114
diferencia de orden k > 1, 7
diferencia de orden uno, 7
ecuacin caracterstica, 22
ecuacin de recurrencia, 20
Espiral de Fibonacci, 42

Sucesin de Fibonacci, 20
sucesin recurrente, 20
sucesiones recurrentes no homogneas, 31
suma parcial, 6
trmino general, 6
Tringulo de Sierpinski, 128
Tringulo de Tartaglia, 123

Frmula de Binet, 23
funcin generatriz, 35
Identidad de Cassini, 62
Identidad de Catalan, 62
Identidad de dOcagne, 62
Identidad de GelinCesro, 62
Nmeros de Jacobsthal, 79
Nmeros de Lucas, 61
Nmeros de Padovan, 79
Nmeros de Pell, 79
Nmero de Stirling de primera especie, 132
Nmero de Stirling se segunda especie, 131
Nmero trinomial, 133
Nmeros de Perrin, 44
Principio de Induccin, 112
progresin aritmtica de grado k 0, 7
progresiones aritmticas, 6
progresiones geomtricas, 13
razn, 13
Serie de Fibonacci, 38
Serie de Maclaurin, 38
Sucesin de Pell, 79
137

También podría gustarte